You are on page 1of 196

A 73-year-old patient who sustained a right hip

fracture in a fall requests pain medication from


the nurse. Based on his injury, which type of
pain is this patient most likely experiencing?
1) Phantom
2) Visceral
3) Deep somatic
4) Referred
Answer:
3) Deep somatic

Rationale:
Deep somatic pain originates in ligaments,
tendons, nerves, blood vessels, and bones.
Therefore, a hip fracture causes deep somatic pain.
Phantom pain is pain that is perceived to originate
from a part that was removed during surgery.
Visceral pain is caused by deep internal pain
receptors and commonly occurs in the abdominal
cavity, cranium, and thorax. Referred pain occurs
in an area that is distant to the original site.

Which pain management task can the nurse
safely delegate to nursing assistive personnel?
1) Asking about pain during vital signs
2) Evaluating the effectiveness of pain
medication
3) Developing a plan of care involving
nonpharmacologic interventions
4) Administering over-the-counter pain
medications
Answer:
1) Asking about pain during vital signs

Rationale:
The nurse can delegate the task of asking about
pain when nursing assistive personnel (NAP)
obtain vital signs. The NAP must be instructed to
report findings to the nurse without delay. The
nurse should evaluate the effectiveness of pain
medications and develop the plan of care.
Administering over-the-counter and prescription
medications is the responsibility of the registered
nurse or licensed practical nurse.

Which factor in the patient's past medical
history dictates that the nurse exercise caution
when administering acetaminophen (Tylenol)?

1) Hepatitis B
2) Occasional alcohol use
3) Allergy to aspirin
4) Gastric irritation with bleeding
Answer:
1) Hepatitis B

Rationale:
Even in recommended doses, acetaminophen can
cause severe hepatotoxicity in patients with liver
disease, such as hepatitis B. Patients who consume
alcohol regularly should also use acetaminophen
cautiously. Those allergic to aspirin or other
nonsteroidal anti-inflammatory drugs (NSAIDs)
can use acetaminophen safely. Acetaminophen
rarely causes gastrointestinal (GI) problems;
therefore, it can be used for those with a history of
gastric irritation and bleeding.

Which action should the nurse take before
administering morphine 4.0 mg intravenously to
a patient complaining of incisional pain?

1) Assess the patient's incision.


2) Clarify the order with the prescriber.
3) Assess the patient's respiratory status.
4) Monitor the patient's heart rate.
Answer:
3) Assess the patient's respiratory status.

Rationale:
Before administering an opioid analgesic, such as
morphine, the nurse should assess the patient's
respiratory status because opioid analgesics can
cause respiratory depression. It is not necessary to
clarify the order with the physician because
morphine 4 mg IV is an appropriate dose. It is not
necessary to monitor the patient's heart rate.

Which action should the nurse take when
preparing patient-controlled analgesia for a
postoperative patient?

1) Caution the patient to limit the number of


times he presses the dosing button.
2) Ask another nurse to double-check the setup
before patient use.
3) Instruct the patient to administer a dose only
when experiencing pain.
4) Provide clear, simple instructions for dosing
if the patient is cognitively impaired.
Answer:
2) Ask another nurse to double-check the setup
before patient use.

Rationale:
As a safeguard to reduce the risk for dosing errors,
the nurse should request another nurse to double-
check the setup before patient use. The nurse
should reassure the patient that the pump has a
lockout feature that prevents him from overdosing
even if he continues to push the dose
administration button. The nurse should also
instruct the patient to administer a dose before
potentially painful activities, such as walking.
Patient-controlled analgesia is contraindicated for
those who are cognitively impaired.

The nurse administers codeine sulfate 30 mg
orally to a patient who underwent craniotomy 3
days ago for a brain tumor. How soon after
administration should the nurse reassess the
patient's pain?

1) Immediately
2) In 10 minutes
3) In 15 minutes
4) In 60 minutes
Answer:
4) In 60 minutes

Rationale:
Codeine administered by the oral route reaches
peak concentration in 60 minutes; therefore, the
nurse should reassess the patient's pain 60 minutes
after administration. The nurse should reassess
pain after 10 minutes when administering codeine
by the intramuscular or subcutaneous routes. Drugs
administered by the intravenous (IV) route are
effective almost immediately; however, codeine is
not recommended for IV administration.

Which nonsteroidal anti-inflammatory drug
might be administered to inhibit platelet
aggregation in a patient at risk for
thrombophlebitis?

1) Ibuprofen (Motrin)
2) Celecoxib (Celebrex)
3) Aspirin (Ecotrin)
4) Indomethacin (Indocin)
Answer:
3) Aspirin (Ecotrin)

Rationale:
Aspirin is a unique NSAID that inhibits platelet
aggregation. Low-dose aspirin therapy is
commonly administered to decrease the risk of
thrombophlebitis, myocardial infarction, and
stroke. Ibuprofen, celecoxib, and indomethacin are
NSAIDs, but they do not inhibit platelet
aggregation.

A client who is receiving epidural analgesia
complains of nausea and loss of motor function
in his legs. The nurse obtains his blood pressure
and notes a drop in his blood pressure from the
previous reading. Which complication is the
patient most likely experiencing?

1) Infection at the catheter insertion site


2) Side effect of the epidural analgesic
3) Epidural catheter migration
4) Spinal cord damage
Answer:
3) Epidural catheter migration

Rationale:
The patient is exhibiting signs of epidural catheter
migration, which include nausea, a decrease in
blood pressure, and loss of motor function without
an identifiable cause. Signs of infection at the
catheter site include redness, swelling, and
drainage. Loss of motor function is not a typical
side effect associated with epidural analgesics.
These are common signs of catheter migration, not
spinal cord damage.

Which of the following clients is experiencing
an abnormal change in vital signs? A client
whose (select all that apply):

1) Blood pressure (BP) was 132/80 mm Hg


sitting and is 120/60 mm Hg upon standing
2) Rectal temperature is 97.9°F in the morning
and 99.2°F in the evening
3) Heart rate was 76 before eating and is 60
after eating
4) Respiratory rate was 14 when standing and is
22 after walking
Answer:
1) Blood pressure (BP) was 132/80 mm Hg sitting
and is 120/60 mm Hg upon standing
3) Heart rate was 76 before eating and is 60 after
eating

Rationale:
The BP change is abnormal; a BP change greater
than 10 mm Hg may indicate postural hypotension.
The change in heart rate is abnormal; heart rate
usually increases slightly after eating rather than
decreasing. The temperatures are within normal
range for the rectal route, and temperature
increases throughout the day. It is normal to have
an increased respiratory rate after exercise.

The nurse assesses clients' breath sounds. Which
one requires immediate medical attention? A
client who has:

1) Crackles
2) Rhonchi
3) Stridor
4) Wheezes
Answer:
3) Stridor

Rationale:
Stridor is a sign of respiratory distress, possibly
airway obstruction. Crackles and rhonchi indicate
fluid in the lung; wheezes are caused by narrowing
of the airway. Crackles, rhonchi, and wheezes
indicate respiratory illness and are potentially
serious but do not necessarily indicate respiratory
distress that requires immediate medical attention.

The nurse assesses the client's pedal pulses as
having a pulse volume of 1 on a scale of 0 to 3.
Based on this assessment finding, it would be
important for the nurse to also assess the:

1) Pulse deficit
2) Blood pressure
3) Apical pulse
4) Pulse pressure
Answer:
2) Blood pressure

Rationale:
If the leg pulses are weak, the nurse should assess
the blood pressure in order to further explore the
reason for the low pulse volume. If the blood
pressure is low, then a low pulse volume would be
expected. The pulse deficit is the difference
between the apical and radial pulse. The apical
pulse would not be helpful to assess peripheral
circulation. The pulse pressure is the difference
between the systolic and diastolic pressures.

Which of the following clients has indications of
orthostatic hypotension? A client whose blood
pressure is:

1) 118/68 when standing and 110/72 when lying


down
2) 140/80, HR 82 bpm when sitting and 136/76,
HR 98 bpm when standing
3) 126/72 lying down and 133/80 when sitting,
and reports shortness of breath
4) 146/88 when lying down and 130/78 when
standing, and reports feeling dizzy
Answer:
4) 146/88 when lying down and 130/78 when
standing, and reports feeling dizzy

Rationale:
Orthostatic hypotension is a drop of 10 mm Hg or
more in blood pressure upon moving to a standing
position, with complaints of feeling dizzy and/or
faint.

A client who has experienced prolonged
exposure to the cold is admitted to the hospital.
Which method of taking a temperature would be
most appropriate for this client?

1) Axillary with an electronic thermometer


2) Oral with a glass thermometer
3) Rectal with an electronic thermometer
4) Tympanic with an infrared thermometer
Answer:
3) Rectal with an electronic thermometer

Rationale:
The rectal route is the most accurate for assessing
core temperature, especially when it is critical to
get an accurate temperature. Therefore, in this
situation it is preferred. Temperature is a
particularly relevant data point for this client with
hypothermia as it indicates the patient's baseline
status and response to treatment. The electronic
thermometer is safer than glass and is relatively
accurate. Mercury thermometers are no longer
used in the hospital setting. The accuracy of
tympanic thermometers is debatable.

Which of the following clients would have the
most difficulty maintaining thermoregulation?

1) Young child playing soccer during the


summer
2) Middle-aged adult snow skiing
3) Young adult playing golf on a hot day
4) Older adult raking leaves on a cold day
Answer:
4) Older adult raking leaves on a cold day

Rationale:
Older adults have more difficulty maintaining
body heat because of their slower metabolism, loss
of subcutaneous fat, and decreased vasomotor
control.

Which of the following clients should have an
apical pulse taken? A client who is:

1) Febrile and has a radial pulse of 100 bpm


2) A runner who has a radial pulse of 62 bpm
3) An infant with no history of cardiac defect
4) An elderly adult who is taking antianxiety
medication
Answer:
3) An infant with no history of cardiac defect

Rationale:
An apical pulse should be taken if the radial pulse
is weak and/or irregular, if the rate is <60 or >100,
if the patient is on cardiac medications, or when
assessing children up to 3 years. It is difficult to
palpate a peripheral pulse on infants and young
children.

Which situation requires intrapersonal
communication?

1) Staff meetings
2) Positive self-talk
3) Shift report
4) Wound care committee meeting
Answer:
2) Positive self-talk

Rationale:
The nurse engaging in positive self-talk is using
intrapersonal communication—conscious internal
dialogue. Staff meetings, shift report, and a
committee meeting are all examples of group or
interpersonal communication.

The nurse suspects that a patient is being
physically abused at home. What is the best
environment in which to discuss the possibility
of abusive events?

1) The patient's shared semiprivate room


2) The hallway outside the patient's room
3) An empty corner at the nurse's station
4) A conference room at the end of the hall
Answer:
4) A conference room at the end of the hall

Rationale:
The best environment in which to discuss sensitive
matters is a quiet room where conversation can
occur in private, particularly when the space is
nonthreatening. The patient might be distracted if
conversation takes place in a room where others
(e.g., patients and visitors) are present. The
hallway outside the patient's room and the nurses'
station are public areas and should not be used for
private conversation.

A patient is admitted to the medical surgical
floor with a kidney infection. The nurse
introduces herself to the patient and begins her
admission assessment. Which goal is most
appropriate for this phase of the nurse-patient
relationship? The patient will be able to:

1) Describe how to operate the bed and call for


the nurse.
2) Discuss communication patterns and roles
within the family.
3) Openly express his concerns about the
hospitalization.
4) State expectations related to discharge.
Answer:
1) Describe how to operate the bed and call for the
nurse.

Rationale:
This is the orientation phase of the relationship.
The orientation phase begins when the nurse
introduces herself to the patient and begins to
gather data. In this phase, the nurse and patient are
getting to know each other. As part of the
orientation phase, the nurse will orient the patient
to the hospital room and routines. In the
preinteraction phase, the nurse gathers information
about the patient before she meets him. Discussion
of personal information, particularly if sensitive or
complex, is suitable for the working phase of the
nurse-patient interaction. The patient expressing
feelings and concerns also occurs during the
working phase. During the working phase, care is
communicated, thoughts and feelings are
expressed, and honest verbal and nonverbal
communication occurs. Stating expectations
related to discharge is most appropriate for the
termination phase—the conclusion of the
relationship.

A local church organizes a group for people
who are having difficulty coping with the death
of a loved one. Which type of group has been
organized?

1) Work-related social support group


2) Therapy group
3) Task group
4) Community committee
Answer:
2) Therapy group

Rationale:
Therapy groups are designed to help individual
members cope with issues, such as the death of a
spouse, divorce, or motherhood. Work-related
social support groups help members of a
profession cope with work-associated stress. Task
groups meet to accomplish a specified task.
Community-based committees meet to discuss
community issues.

A mother comes to the emergency department
after receiving a phone call informing her that
her son was involved in a motor vehicle
accident. When she approaches the triage desk,
she frantically asks, "How is my son?" Which
response by the nurse is best?

1) "He's being examined now; he's awake and


talking. We'll take you to see him soon."
2) "Don't worry, I'm sure he'll be fine; we have
an excellent trauma team caring for him."
3) "Everything will be okay; please take a seat
and I'll check on him for you."
4) "Your son is strong and has youth on his side;
I'm sure he'll be fine."
Answer:
1) "He's being examined now; he's awake and
talking. We'll take you to see him soon."

Rationale:
By telling the mother that her son is awake and
talking and being examined by the doctor, the
nurse provides accurate information and helps
reduce the mother's anxiety. Responses such as
"Don't worry, everything will be okay" and "I'm
sure he'll be fine" offer false reassurance and fail to
respect the mother's concern.

During a presentation at a nursing staff meeting,
the unit manager speaks very slowly with a
monotone. She uses medical and technical
terminology to convey her message. Dressed in
business attire, the manager stands erect and
smiles occasionally while speaking. Which
elements of her approach are likely to cause the
staff to lose interest in what she has to say?
Select all answers that apply.

1) Slow speech
2) Monotone
3) Occasional smile
4) Formal dress
Answer:
1) Slow speech
2) Monotone

Rationale:
Speaking slowly with a monotone can contribute to
reduced attention as the listener can think faster
than the speaker is speaking, and the monotone
voice has an almost hypnotizing effect. Smiling
improves personal interest and connection between
the speaker and listener so should not cause a loss
of interest. Wearing formal business attire would
not directly detract from listeners' engagement in
the speaker's message unless it was unusual
enough to distract listeners; nothing in the situation
above indicates that is so.

Which factor(s) in the patient's past medical
history place(s) him at risk for falling? Select all
that apply.

1) Orthostatic hypotension
2) Appendectomy
3) Dizziness
4) Hyperthyroidism
Answer:
1) Orthostatic hypotension
3) Dizziness
Rationale:
Orthostatic hypotension, cognitive impairment,
difficulty with walking or balance, weakness,
dizziness, and drowsiness from certain medications
place the patient at risk for falling. A history of
right appendectomy and hyperthyroidism do not
place that patient at risk for falling.

The nurse is teaching a child and family about
firearm safety. The nurse should instruct the
child to take which step first if he sees a gun at a
friend's house?

1) Leave the area.


2) Do not touch the gun.
3) Stop where he is.
4) Tell an adult.
Answer:
3) Stop where he is.

Rationale:
The child should be instructed to stop where he is.
This allows him to think about the next steps he
has memorized. Next, he should avoid touching the
gun, leave the area, and immediately go tell an
adult.

A patient is agitated and continues to try to get
out of bed. The nurse tries unsuccessfully to
reorient him. What should the nurse do next?

1) Apply a vest restraint.


2) Move the patient to a quieter room.
3) Ask another nurse to care for the patient.
4) Provide comfort measures.
Answer:
4) Provide comfort measures.

Rationale:
Patients sometimes become agitated because they
are uncomfortable or in pain. Providing comfort
measures may decrease agitation. If the patient
continues to be agitated, the nurse should
encourage a family member or friend to sit with
the patient. Applying a physical restraint should be
kept as a last resort for use only when less
restrictive measures fail. The patient should be
placed in a room near the nurses' station so he can
be checked frequently if there is no one available
to provide one-on-one supervision. A quieter room
would probably not help.

While teaching a health promotion group of
adults, the nurse notices one person who is
clutching his throat with both hands. What
should the nurse do first?

1) Call 9-1-1.
2) Encourage the person to cough vigorously.
3) Ask, "Are you choking?"
4) Give five back blows.
Answer:
3) Ask, "Are you choking?"

Rationale:
Clutching the throat is the universal sign of
choking. The first action when you suspect airway
obstruction is to ask, "Are you choking?" If the
person indicates "yes," or if the person cannot
cough, speak, or breathe, that indicates choking.
You must first be certain the person is choking
because you can cause harm when you perform the
choking maneuver. You would not call 9-1-1,
encourage coughing, or give five back blows until
you first establish that the person is choking. The
client appears to be giving the universal sign for
choking, but the nurse must validate the client's
meaning before acting.

What should parents do to promote child safety
in the home?

1) Attach the baby's pacifier to a ribbon so that


it does not fall on the floor.
2) Give a 2-year-old whole grapes instead of
popcorn for a snack.
3) Store firearms unloaded and out of sight in a
location too high for the child to reach.
4) Install window guards; never leave a window
wide open.
Answer:
4) Install window guards; never leave a window
wide open.
Rationale:
To prevent falls, install window guards and never
leave a window wide open. A ribbon can become
entangled around a small child's neck, causing
asphyxiation. Young children can easily choke on
a grape. Firearms should be unloaded, but stored in
a locked cabinet. Children are curious and like to
explore and climb. It would not be too difficult for
a child to find a firearm stored, for example, on a
high closet shelf.

What is the leading cause of unintentional death
for the entire U.S. population?

1) Motor vehicle accidents


2) Poisoning
3) Choking
4) Falls
Answer:
1) Motor vehicle accidents

Rationale:
The leading causes of unintentional death for the
total population, in this order, are automobile
accidents, poisoning, falls, and drowning.

Which change in hygiene practices may be
necessary as the patient ages?

1) Brushing teeth twice a day


2) Bathing every other day
3) Decreasing moisturizer use
4) Increasing soap use
Answer:
2) Bathing every other day
Rationale:
As a person ages, sebaceous glands become less
active, causing skin to dry. Older people may find
it necessary to bathe every 2 days, increase the use
of moisturizers, and decrease soap use to prevent
further drying of skin. Older adults should brush
their teeth after every meal and at bedtime to
prevent tooth decay. It is recommended that people
of all ages brush their teeth at least twice a day, so
that option does not represent a change in an older
adult's hygiene practices.

A woman of Orthodox Jewish faith who
underwent a hysterectomy for cancer is being
cared for on the surgical floor. Which healthcare
team member(s) could be assigned to bathe this
patient? Choose all correct answers.

1) Male nursing assistant


2) Male licensed practical nurse
3) Female graduate nurse
4) Female registered nurse
Answer:
3) Female graduate nurse
4) Female registered nurse

Rationale:
Orthodox Judaism prohibits personal care being
provided by a member of the opposite sex. The
patient who underwent a hysterectomy is female;
therefore, out of respect for her religious beliefs,
she should not be bathed by the male licensed
practical nurse or nursing assistant.

A 75-year-old patient who is 5 feet 7 inches tall
and weighs 170 pounds is admitted with
dehydration. A nursing diagnosis of Risk for
Impaired Skin Integrity is identified for this
patient. Which factor places the client at Risk
for Impaired Skin Integrity?
Answer:
Dehydration
Rationale:
Dehydration places the patient at risk for impaired
skin integrity. Dehydration, caused by fluid
volume deficit, causes the skin to become dry and
crack easily, impairing skin integrity. People who
are very thin or very obese are more likely to
experience impaired skin integrity. This patient is
of normal height and weight; therefore, his body
stature does not place him at risk. There is nothing
to suggest that this patient has an impaired
nutritional status.

The nurse notes a lesion that appears to be
caused by tissue compression on the right hip of
a patient who suffered a stroke 5 days ago. How
should the nurse document this finding?

1) Maceration
2) Abrasion
3) Excoriation
4) Pressure ulcer
Answer:
4) Pressure ulcer

Rationale:
The nurse should document a lesion caused by
tissue compression and inadequate perfusion as a
pressure ulcer. Abrasion, a rubbing away of the
epidermal layer of skin, is commonly caused by
shearing forces that occur when a patient moves or
is moved in bed. Maceration is a softening of skin
from prolonged moisture. Excoriation is a loss of
the superficial layers of the skin caused by the
digestive enzymes in feces.

The charge nurse asks the nursing assistive
personnel (NAP) to give a bag bath to a patient
with end-stage chronic obstructive pulmonary
disease. How should the NAP proceed?

1) Bathe the patient's entire body using 8 to 10


washcloths.
2) Assist the patient to a chair and provide
bathing supplies.
3) Saturate a towel and blanket in a plastic bag,
and then bathe the patient.
4) Assist the patient to the bathtub and provide a
bath chair.
Answer:
1) Bathe the patient's entire body using 8 to 10
washcloths.

Rationale:
A towel bath is a modification of the bed bath in
which the NAP places a large towel and a bath
blanket into a plastic bag, saturates them with a
commercially prepared mixture of moisturizer,
nonrinse cleaning agent, and water; warms in them
in a microwave, and then uses them to bathe the
patient. A bag bath is a modification of the towel
bath, in which the NAP uses 8 to 10 washcloths
instead of a towel or blanket. Each part of the
patient's body is bathed with a fresh cloth. A bag
bath is not given in a chair or in the tub.

For a morbidly obese patient, which intervention
should the nurse choose to counteract the
pressure created by the skin folds?

1) Cover the mattress with a sheepskin.


2) Keep the linens wrinkle free.
3) Separate the skin folds with towels.
4) Apply petrolatum barrier creams.
Answer:
2) Keep the linens wrinkle free.

Rationale:
Separating the skin folds with towels relieves the
pressure of skin rubbing on skin. Sheepskins are
not recommended for use at all. Petrolatum barrier
creams are used to minimize moisture caused by
incontinence.

A client exhibits all of the following during a
physical assessment. Which of these is
considered a primary defense against infection?

1) Fever
2) Intact skin
3) Inflammation
4) Lethargy
Answer:
2) Intact skin

Rationale:
Intact skin is considered a primary defense against
infection. Fever, the inflammatory response, and
phagocytosis (a process of killing pathogens) are
considered secondary defenses against infection.

A client with a stage 2 pressure ulcer has
methicillin-resistant Staphylococcus aureus
(MRSA) cultured from the wound. Contact
precautions are initiated. Which rule must be
observed to follow contact precautions?

1) A clean gown and gloves must be worn when


in contact with the client.
2) Everyone who enters the room must wear a
N-95 respirator mask.
3) All linen and trash must be marked as
contaminated and send to biohazard waste.
4) Place the client in a room with a client with
an upper respiratory infection.
Answer:
1) A clean gown and gloves must be worn when in
contact with the client.

Rationale:
A clean gown and gloves must be worn when any
contact is anticipated with the client or with
contaminated items in the room. A respirator mask
is required only with airborne precautions, not
contact precautions. All linen must be double-
bagged and clearly marked as contaminated. The
client should be placed in a private room or in a
room with a client with an active infection caused
by the same organism and no other infections.

A client requires protective isolation. Which
client can be safely paired with this client in a
client-care assignment? One

1) admitted with unstable diabetes mellitus.


2) who underwent surgical repair of a perforated
bowel.
3) with a stage 3 sacral pressure ulcer.
4) admitted with a urinary tract infection.
Answer:
1) admitted with unstable diabetes mellitus.

Rationale:
The client with unstable diabetes mellitus can
safely be paired in a client-care assignment
because the client is free from infection.
Perforation of the bowel exposes the client to
infection requiring antibiotic therapy during the
postoperative period. Therefore, this client should
not be paired with a client in protective isolation.
A client in protective isolation should not be paired
with a client who has an open wound, such as a
stage 3 pressure ulcer, or with a client who has a
urinary tract infection.

Which action demonstrates a break in sterile
technique?

1) Remaining 1 foot away from nonsterile areas


2) Placing sterile items on the sterile field
3) Avoiding the border of the sterile drape
4) Reaching 1 foot over the sterile field
Answer:
4) Reaching 1 foot over the sterile field

Rationale:
Reaching over the sterile field while wearing
sterile garb breaks sterile technique. While
observing sterile technique, healthcare workers
should remain 1 foot away from nonsterile areas
while wearing sterile garb, place sterile items
needed for the procedure on the sterile drape, and
avoid coming in contact with the 1-inch border of
the sterile drape.

A mother who breastfeeds her child passes on
which antibody through breast milk?
1) IgA
2) IgE
3) IgG
4) IgM
Answer:
3) IgG

Rationale:
The antibody IgG is passed to the child through the
mother's breast milk during breastfeeding. IgA,
IgE, and IgM are produced by the child's body
after exposure to an antigen.

What is the rationale for hand washing? Hand
washing is expected to remove:

1) transient flora from the skin.


2) resident flora from the skin.
3) all microorganisms from the skin.
4) media for bacterial growth.
Answer:
1) transient flora from the skin.

Rationale:
There are two types of normal flora: transient and
resident. Transient flora are normal flora that a
person picks up by coming in contact with objects
or another person (e.g., when you touch a soiled
dressing). You can remove these with hand
washing. Resident flora live deep in skin layers
where they live and multiply harmlessly. They are
permanent inhabitants of the skin and cannot
usually be removed with routine hand washing.
Removing all microorganisms from the skin
(sterilization) is not possible without damaging the
skin tissues. To live and thrive in humans,
microbes must be able to use the body's precise
balance of food, moisture, nutrients, electrolytes,
pH, temperature, and light. Food, water, and soil
that provide these conditions may serve as
nonliving reservoirs. Hand washing does little to
make the skin uninhabitable for microorganisms,
except perhaps briefly when an antiseptic agent is
used for cleansing.

Which of the following incidents requires the
nurse to complete an occurrence report?

1) Medication given 30 minutes after scheduled


dose time
2) Patient's dentures lost after transfer
3) Worn electrical cord discovered on an IV
infusion pump
4) Prescription without the route of
administration
Answer:
2) Patient's dentures lost after transfer

Rationale:
You would need to complete an occurrence report
if you suspect your patient's personal items to be
lost or stolen. A medication can be administered
within a half-hour of the administration time
without an error in administration; therefore, an
occurrence report is not necessary. The worn
electrical cord should be taken out of use and
reported to the biomedical department. The nurse
should seek clarification if the provider's order is
missing information; an occurrence report is not
necessary.

The nurse is orienting a new nurse to the unit
and reviews source-oriented charting. Which
statement by the nurse best describes source-
oriented charting? Source-oriented charting:

1) Separates the health record according to


discipline
2) Organizes documentation around the patient's
problems
3) Highlights the patient's concerns, problems,
and strengths
4) Is designed to streamline documentation
Answer:
1) Separates the health record according to
discipline

Rationale:
In source-oriented charting, each discipline
documents findings in a separately labeled section
of the chart. Problem-oriented charting organizes
notes around the patient's problems. Focus®
charting highlights the patient's concerns,
problems, and strengths. Charting by exception is a
unique charting system designed to streamline
documentation.

When the nurse completes the patient's
admission nursing database, the patient reports
that he does not have any allergies. Which
acceptable medical abbreviation can the nurse
use to document this finding?

1) NA
2) NDA
3) NKA
4) NPO
Answer:
3) NKA

Rationale:
The nurse can use the medical abbreviation NKA,
which means no known allergies, to document this
finding. NA is an abbreviation for not applicable.
NDA is an abbreviation for no known drug
allergies. NPO is an abbreviation that means
nothing by mouth.

The nurse is working on a unit that uses nursing
assessment flow sheets. Which statement best
describes this form of charting? Nursing
assessment flow sheets:

1) Are comprehensive charting forms that


integrate assessments and nursing actions
2) Contain only graphic information, such as
I&O, vital signs, and medication administration
3) Are used to record routine aspects of care;
they do not contain assessment data
4) Contain vital data collected upon admission,
which can be compared with newly collected
data
Answer:
1) Are comprehensive charting forms that integrate
assessments and nursing actions

Rationale:
Nursing assessment flow sheets are organized by
body systems. The nurse checks the box
corresponding to the current assessment findings.
Nursing actions, such as wound care, treatments,
or IV fluid administration, are also included.
Graphic information, such as vital signs, I&O, and
routine care, may be found on the graphic record.
The admission form contains baseline information.

At the end of the shift, the nurse realizes that she
forgot to document a dressing change that she
performed for a patient. Which action should the
nurse take?

1) Complete an occurrence report before


leaving.
2) Do nothing; the next nurse will document it
was done.
3) Write the note of the dressing change into an
earlier note.
4) Make a late entry as an addition to the
narrative notes.
Answer:
4) Make a late entry as an addition to the narrative
notes.

Rationale:
If the nurse fails to make an important entry while
charting, she should make a late entry as an
addition to the narrative notes. An occurrence
report is not necessary in this case. If
documentation is omitted, there is no legal
verification that the procedure was performed. It is
illegal to add to a chart entry that was previously
documented. The nurse can only document care
directly performed or observed. Therefore, the
nurse on the incoming shift would not record the
wound change as performed.

The client asks the nurse why an electronic
health record (EHR) system is being used.
Which response by the nurse indicates an
understanding of the rationale for an EHR
system?

1) It includes organizational reports of unusual


occurrences that are not part of the client's
record.
2) This type of system consists of combined
documentation and daily care plans.
3) It improves interdisciplinary collaboration
that improves efficiency in procedures.
4) This type of system tracks medication
administration and usage over 24 hours.
Answer:
3) It improves interdisciplinary collaboration that
improves efficiency in procedures.

Rationale:
The EHR has several benefits for use, including
improving interdisciplinary collaboration and
making procedures more accurate and efficient. An
occurrence report is an organizational record of an
unusual occurrence or accident that is not a part of
the client's record. Integrated plans of care (IPOC)
are a combined charting and care plan format. A
medication administration record (MAR) is used to
document medications administered and their
usage.

In the United States, the first programs for
training nurses were affiliated with:
1) The military
2) General hospitals
3) Civil service
4) Religious orders
Answer:
4) Religious orders

Rationale:
When the Civil War broke out, the Army used
nurses who had already been trained in religious
orders. Although the Army did provide some
training, it occurred later than in the religious
orders. Although nurses were trained in hospitals,
the training and the hospitals were affiliated with
religious orders. Civil service was not mentioned
in Chapter 1 and was not a factor in the early
1800s. Nursing started with religious orders. The
Hindu faith was the first to write about nursing. In
the United States, all training for nurses was
affiliated with religious orders until after the Civil
War.

Which of the following is/are an example(s) of a
health restoration activity? Select all that apply.

1) Administering an antibiotic every day


2) Teaching the importance of hand washing
3) Assessing a client's surgical incision
4) Advising a woman to get an annual
mammogram after age 50 years
Answer:
1) Administering an antibiotic every day
3) Assessing a client's surgical incision

Rationale:
Health restoration activities help an ill client return
to health. This would include taking an antibiotic
every day and assessing a client's surgical incision.
Hand washing and mammograms both involve
healthy people who are trying to prevent illness.

Which of the following aspects of nursing is
essential to defining it as both a profession and a
discipline?

1) Established standards of care


2) Professional organizations
3) Practice supported by scientific research
4) Activities determined by a scope of practice
Answer:
3) Practice supported by scientific research

Rationale:
The American Nurses Association (ANA) has
developed standards of care, but they are unrelated
to defining nursing as a profession or discipline.
Having professional organizations is not included
in accepted characteristics of either a profession or
a discipline. A profession must have knowledge
that is based on technical and scientific knowledge.
The theoretical knowledge of a discipline must be
based on research, so both are scientifically based.
Having a scope of practice is not included in
accepted characteristics of either a profession or a
discipline.

The charge nurse on the medical surgical floor
assigns vital signs to the nursing assistive
personnel (NAP) and medication administration
to the licensed vocational nurse (LVN). Which
nursing model of care is this floor following?

1) Team nursing
2) Case method nursing
3) Functional nursing
4) Primary nursing
Answer:
3) Functional nursing

Rationale:
With team nursing, an RN or LVN is paired with a
NAP. The pair is then assigned to render care for a
group of patients. In case method nursing, one
nurse cares for one patient during her entire shift.
Private duty nursing is an example of this care
model. This medical surgical floor is following the
functional nursing model of care, in which care is
partitioned and assigned to a staff member with the
appropriate skills. For example, the NAP is
assigned vital signs, and the LVN is assigned
medication administration. When the primary
nursing model is utilized, one nurse manages care
for a group of patients 24 hours a day, even though
others provide care during part of the day.

A patient who suffered a stroke has difficulty
swallowing. Which healthcare team member
should be consulted to assess the patient's risk
for aspiration?

1) Respiratory therapist
2) Occupational therapist
3) Dentist
4) Speech therapist
Answer:
4) Speech therapist

Rationale:
Respiratory therapists provide care for patients
with respiratory disorders. Occupational therapists
help patients regain function and independence.
Dentists diagnose and treat dental disorders.
Speech and language therapists provide assistance
to clients experiencing swallowing and speech
disturbances. They assess the risk for aspiration
and recommend a treatment plan to reduce the risk.

Which of the following is/are an example(s) of
theoretical knowledge as defined in this
chapter? Select all that apply.

1) Antibiotics are ineffective in treating viral


infections.
2) When you take a patient's blood pressure, the
patient's arm should be at heart level.
3) In Maslow's framework, physical needs are
most basic.
4) When drawing medication out of a vial, inject
air into the vial first.
Answer:
1) Antibiotics are ineffective in treating viral
infections.
3) In Maslow's framework, physical needs are
most basic.

Rationale:
Theoretical knowledge consists of research
findings, facts (e.g., "Antibiotics are
ineffective . . ." is a fact), principles, and theories
(e.g., "In Maslow's framework . . ." is a statement
from a theory). Instructions for taking a blood
pressure and withdrawing medications are
examples of practical knowledge—what to do and
how to do it.

Critical thinking and the nursing process have
which of the following in common? Both:

1) Are important to use in nursing practice


2) Use an ordered series of steps
3) Are patient-specific processes
4) Were developed specifically for nursing
Answer:
1) Are important to use in nursing practice

Rationale:
Nurses make many decisions: some require using
the nursing process, whereas others are not client
related but require critical thinking. The nursing
process has specific steps; critical thinking does
not. Neither is linear. Critical thinking applies to
any discipline.

In which step of the nursing process does the
nurse analyze data and identify client problems?

1) Assessment
2) Diagnosis
3) Planning outcomes
4) Evaluation
Answer:
2) Diagnosis
Rationale:
In the assessment phase, the nurse gathers data
from many sources for analysis in the diagnosis
phase. In the diagnosis phase, the nurse identifies
the client's health status. In the planning outcomes
phase, the nurse formulates goals and outcomes. In
the evaluation phase, which occurs after
implementing interventions, the nurse gathers data
about the client's responses to nursing care to
determine whether client outcomes were met.

In which phase of the nursing process does the
nurse decide whether her actions have
successfully treated the client's health problem?
Answer:
Evaluation

Rationale:
In the assessment phase, the nurse gathers data
from many sources for analysis in the diagnosis
phase. In the diagnosis phase, the nurse identifies
the client's health status. In the planning outcomes
phase, the nurse and client decide on goals they
want to achieve. In the intervention planning
phase, the nurse identifies specific interventions to
help achieve the identified goal. During the
implementation phase, the nurse carries out the
interventions or delegates them to other health care
team members. During the evaluation phase, the
nurse judges whether her actions have been
successful in treating or preventing the identified
client health problem.

What is the most basic reason that self-
knowledge is important for nurses? Because it
helps the nurse to:

1) Identify personal biases that may affect his


thinking and actions
2) Identify the most effective interventions for a
patient
3) Communicate more efficiently with
colleagues, patients, and families
4) Learn and remember new procedures and
techniques
Answer:
1) Identify personal biases that may affect his
thinking and actions

Rationale:
The most basic reason is that self-knowledge
directly affects the nurse's thinking and the actions
he chooses. Indirectly, thinking is involved in
identifying effective interventions, communicating,
and learning procedures. However, because
identifying personal biases affects all the other
nursing actions, it is the most basic reason.

Arrange the steps of the nursing process in the
sequence in which they generally occur. A.
Assessment
B. Evaluation
C. Planning outcomes
D. Planning interventions
E. Diagnosis

1) E, B, A, D, C
2) A, B, C, D, E
3) A, E, C, D, B
4) D, A, B, E, C
Answer:
3) A, E, C, D, B

Rationale:
Logically, the steps are assessment, diagnosis,
planning outcomes, planning interventions, and
evaluation. Keep in mind that steps are not always
performed in this order, depending on the patient's
needs, and that steps overlap.

How are critical thinking skills and critical
thinking attitudes similar? Both are:

1) Influences on the nurse's problem solving and


decision making
2) Like feelings rather than cognitive activities
3) Cognitive activities rather than feelings
4) Applicable in all aspects of a person's life
Answer:
1) Influences on the nurse's problem solving and
decision making

Rationale:
Cognitive skills are used in complex thinking
processes, such as problem solving and decision
making. Critical thinking attitudes determine how
a person uses her cognitive skills. Critical thinking
attitudes are traits of the mind, such as independent
thinking, intellectual curiosity, intellectual
humility, and fair-mindedness, to name a few.
Critical thinking skills refer to the cognitive
activities used in complex thinking processes. A
few examples of these skills involve recognizing
the need for more information, recognizing gaps in
one's own knowledge, and separating relevant
from irrelevant data. Critical thinking, which
consists of intellectual skills and attitudes, can be
used in all aspects of life.

The nurse is preparing to admit a patient from
the emergency department. The transferring
nurse reports that the patient with chronic lung
disease has a 30+ year history of tobacco use.
The nurse used to smoke a pack of cigarettes a
day at one time and worked very hard to quit
smoking. She immediately thinks to herself, "I
know I tend to feel negatively about people who
use tobacco, especially when they have a serious
lung condition; I figure if I can stop smoking,
they should be able to. I must remember how
physically and psychologically difficult that is,
and be very careful not to let be judgmental of
this patient." This best illustrates:

1) Theoretical knowledge
2) Self-knowledge
3) Using reliable resources
4) Use of the nursing process
Answer:
2) Self-knowledge

Rationale:
Personal knowledge (2) is self-understanding—
awareness of one's beliefs, values, biases, and so
on. That best describes the nurse's awareness that
her bias can affect her patient care. Theoretical
knowledge consists of information, facts,
principles, and theories in nursing and related
disciplines; it consists of research findings and
rationally constructed explanations of phenomena.
Using reliable resources is a critical thinking skill.
The nursing process is a problem-solving process
consisting of the steps of assessing, diagnosing,
planning outcomes, planning interventions,
implementing, and evaluating. The nurse has not
yet met this patient, so she could not have begun
the nursing process.

Which organization's standards require that all
patients be assessed specifically for pain?

1) American Nurses Association (ANA)


2) State nurse practice acts
3) National Council of State Boards of Nursing
(NCSBN)
4) The Joint Commission
Answer:
4) The Joint Commission

Rationale:
The Joint Commission has developed assessment
standards, including that all clients be assessed for
pain. The ANA has developed standards for
clinical practice, including those for assessment,
but not specifically for pain. State nurse practice
acts regulate nursing practice in individual states.
The NCSBN asserts that the scope of nursing
includes a comprehensive assessment but does not
specifically include pain.

Which of the following is an example of data
that should be validated?

1) The urinalysis report indicates there are white


blood cells in the urine.
2) The client states she feels feverish; you
measure the oral temperature at 98°F.
3) The client has clear breath sounds; you count
a respiratory rate of 18.
4) The chest x-ray report indicates the client has
pneumonia in the right lower lobe.
Answer:
2) The client states she feels feverish; you measure
the oral temperature at 98°F.

Rationale:
Validation should be done when subjective and
objective data do not make sense. For instance, it is
inconsistent data when the patient feels feverish
and you obtain a normal temperature. The other
distractors do not offer conflicting data. Validation
is not usually necessary for laboratory test results.

Which of the following is an example of
appropriate behavior when conducting a client
interview?

1) Recording all the information on the agency-


approved form during the interview
2) Asking the client, "Why did you think it was
necessary to seek health care at this time?"
3) Using precise medical terminology when
asking the client questions
4) Sitting, facing the client in a chair at the
client's bedside, using active listening
Answer:
4) Sitting, facing the client in a chair at the client's
bedside, using active listening

Rationale:
Active listening should be used during an
interview. The nurse should face the patient, have
relaxed posture, and keep eye contact. Asking
"why" may make the client defensive. Note-taking
interferes with eye contact. The client may not
understand medical terminology or health care
jargon.

The nurse wishes to identify nursing diagnoses
for a patient. She can best do this by using a
data collection form organized according to
(select all that apply):

1) A body systems model


2) A head-to-toe framework
3) Maslow's hierarchy of needs
4) Gordon's functional health patterns
Answer:
3) Maslow's hierarchy of needs
4) Gordon's functional health patterns

Rationale:
Nursing models produce a holistic database that is
useful in identifying nursing rather than medical
diagnoses. Body systems and head-to-toe are not
nursing models, and they are not holistic; they
focus on identifying physiological needs or
disease. Maslow's hierarchy is not a nursing
model, but it is holistic, so it is acceptable for
identifying nursing diagnoses. Gordon's functional
health patterns are a nursing model.

The nurse is recording assessment data. She
writes, "The patient seems worried about his
surgery. Other than that, he had a good night."
Which errors did the nurse make? Select all that
apply.

1) Used a vague generality


2) Did not use the patient's exact words
3) Used a "waffle" word (e.g., appears)
4) Recorded an inference rather than a cue
Answer:
1) Used a vague generality
3) Used a "waffle" word (e.g., appears)
4) Recorded an inference rather than a cue
Rationale:
The nurse recorded a vague generality: "he has had
a good night." The nurse did not use the patient's
exact words, but she did not quote the patient at
all, so that is not one of her errors. The nurse used
the "waffle" word, "seems" worried instead of
documenting what the patient said or did to lead
her to that conclusion. The nurse recorded these
inferences: worried and had a good night.

A patient is admitted with shortness of breath,
so the nurse immediately listens to his breath
sounds. Which type of assessment is the nurse
performing?
1) Ongoing assessment
2) Comprehensive physical assessment
3) Focused physical assessment
4) Psychosocial assessment
Answer:
3) Focused physical assessment

Rationale:
The nurse is performing a focused physical
assessment, which is done to obtain data about an
identified problem, in this case shortness of breath.
An ongoing assessment is performed as needed,
after the initial data are collected, preferably with
each patient contact. A comprehensive physical
assessment includes an interview and a complete
examination of each body system. A psychosocial
assessment examines both psychological and social
factors affecting the patient. The nurse conducting
a psychosocial assessment would gather
information about stressors, lifestyle, emotional
health, social influences, coping patterns,
communication, and personal responses to health
and illness, to name a few aspects.

The nurse is assessing vital signs for a patient
just admitted to the hospital. Ideally, and if there
are no contraindications, how should the nurse
position the patient for this portion of the
admission assessment?
1) Sitting upright
2) Lying flat on the back with knees flexed
3) Lying flat on the back with arms and legs
fully extended
4) Side-lying with the knees flexed
Answer:
1) Sitting upright

Rationale:
If the patient is able, the nurse should have the
patient sit upright to obtain vital signs in order to
allow the nurse to easily access the anterior and
posterior chest for auscultation of heart and breath
sounds. It allows for full lung expansion and is the
preferred position for measuring blood pressure.
Additionally, patients might be more comfortable
and feel less vulnerable when sitting upright
(rather than lying down on the back) and can have
direct eye contact with the examiner. However,
other positions can be suitable when the patient's
physical condition restricts the comfort or ability
of the patient to sit upright.

For all body systems except the abdomen, what
is the preferred order for the nurse to perform
the following examination techniques?
A. Palpation
B. Auscultation
C. Inspection
D. Percussion
1) D, B, A, C
2) C, A, D, B
3) B, C, D, A
4) A, B, C, D
Answer:
2) C, A, D, B

Rationale:
Inspection begins immediately as the nurse meets
the patient, as she observes the patient's
appearance and behavior. Observational data are
not intrusive to the patient. When performing
assessment techniques involving physical touch,
the behavior, posture, demeanor, and responses
might be altered. Palpation, percussion, and
auscultation should be performed in that order,
except when performing an abdominal assessment.
During abdominal assessment, auscultation should
be performed before palpation and percussion to
prevent altering bowel sounds.

The nurse is assessing a patient admitted to the
hospital with rectal bleeding. The patient had a
hip replacement 2 weeks ago. Which position
should the nurse avoid when examining this
patient's rectal area?
1) Sims'
2) Supine
3) Dorsal recumbent
4) Semi-Fowler's
Answer:
1) Sims'

Rationale:
Sims' position is typically used to examine the
rectal area. However, the position should be
avoided if the patient has undergone hip
replacement surgery The patient with a hip
replacement can assume the supine, dorsal
recumbent, or semi-Fowler's positions without
causing harm to the joint. Supine position is lying
on the back facing upward. The patient in dorsal
recumbent is on his back with knees flexed and
soles of feet flat on the bed. In semi-Fowler's
position, the patient is supine with the head of the
bed elevated and legs slightly elevated.

How should the nurse modify the examination
for a 7-year-old child?
1) Ask the parents to leave the room before the
examination.
2) Demonstrate equipment before using it.
3) Allow the child to help with the examination.
4) Perform invasive procedures (e.g., otoscopic)
last.
Answer:
2) Demonstrate equipment before using it.

Rationale:
The nurse should modify his examination by
demonstrating equipment before using it to
examine a school-age child. The nurse should
make sure parents are not present during the
physical examination of an adolescent, but they
usually help younger children feel more secure.
The nurse should allow a preschooler to help with
the examination when possible, but not usually a
school-age child. Toddlers are often fearful of
invasive procedures, so those should be performed
last in this age group. It is best to perform invasive
procedures last for all age groups; therefore, this
does not represent a modification.

The nurse must examine a patient who is weak
and unable to sit unaided or to get out of bed.
How should she position the patient to begin
and perform most of the physical examination?
1) Dorsal recumbent
2) Semi-Fowler's
3) Lithotomy
4) Sims'
Answer:
2) Semi-Fowler's

Rationale:
If a patient is unable to sit up, the nurse should
place him lying flat on his back, with the head of
the bed elevated. Dorsal recumbent position is
used for abdominal assessment if the patient has
abdominal or pelvic pain. The patient in dorsal
recumbent is on his back with knees flexed and
soles of feet flat on the bed. Lithotomy position is
used for female pelvic examination. It is similar to
dorsal recumbent position, except that the patient's
legs are well separated and thighs are acutely
flexed. Feet are usually placed in stirrups. Fold
sheet or bath blanket crosswise over thighs and
legs so that genital area is easily exposed. Keep
patient covered as much as possible. The patient in
Sim's position is on left side with right knee flexed
against abdomen and left knee slightly flexed. Left
arm is behind body; right arm is placed
comfortably. Sims' position is used to examine the
rectal area. In semi-Fowler's position, the patient is
supine with the head of the bed elevated and legs
slightly elevated.

The nurse should use the diaphragm of the
stethoscope to auscultate which of the
following?
1) Heart murmurs
2) Jugular venous hums
3) Bowel sounds
4) Carotid bruits
Answer:
3) Bowel sounds

Rationale:
The bell of the stethoscope should be used to hear
low-pitched sounds, such as murmurs, bruits, and
jugular hums. The diaphragm should be used to
hear high-pitched sounds that normally occur in
the heart, lungs, and abdomen.

The nurse calculates a body mass index (BMI)
of 18 for a young adult woman who comes to
the physician's office for a college physical.
This patient is considered:
1) Obese
2) Overweight
3) Average
4) Underweight
Answer:
4) Underweight

Rationale:
For adults, BMI should range between 20 and 25;
to 29.9 is overweight; and BMI greater than 30 is
considered obese.

Answer: D

The code of ethics is the philosophical ideals of


right and wrong that define the principles the
nurse will use to provide care to clients. A code
of ethics does not ensure identical care to all
clients (which would not be acceptable). The
nursing code of ethics does not protect clients
from harm or improve self-health care.
The nurse practices nursing in conformity with the
code of ethics for professional registered nurses.
This code:

A) Improves self-health care


B) Protects the client from harm
C) Ensures identical care to all clients
D) Defines the principles by which nurses' provide
care to their clients

Answer: C

The nurse is assessing the client. Diagnosis


occurs after all assessments are completed. Then
a plan is developed and implemented. The
process is completed with evaluation.
An 18-year-old woman is in the emergency
department with fever and cough. The physician
asks the nurse to measure vital signs, auscultate
lung sounds, listen to heart sounds, determine the
level of comfort, and collect blood and sputum
samples for analysis. The nurse is performing what
aspect of practice?

A) Diagnosis
B) Evaluation
C) Assessment
D) Implementation

Answer: D

Implementation is the actual delivery of care.


Assessment is data gathering. Then the
information is developed into a diagnosis and
the planning occurs with the diagnosis.
Evaluation is the final step of the nursing
process.
A client is wheezing and short of breath. The
physician orders a medicated nebulizer treatment
now and in 4 hours. The nurse is providing what
aspect of care?

A) Planning
B) Evaluation
C) Assessment
D) Implementation

Answer: C

An advocate helps speak for the client,


communicating the client's concerns and wishes
to family and other caregivers. A caregiver
assists in meeting all health care needs of the
client, including taking measures to restore
emotional, spiritual, and social well-being. A
manager coordinates all the activities of the
members of the nursing staff in delivering
nursing care and has personnel, policy, and
budgetary responsibilities for a specific nursing
unit or agency. An educator explains concepts
and facts about health, demonstrates procedures
such as self-care activities, reinforces learning
or client behavior, and evaluates the client's
progress in learning.
The nurse is caring for a client with end-stage lung
disease. The client wants to go home on oxygen
therapy and be comfortable. The family wants the
client to undergo a new surgical procedure. The
nurse explains the risk and benefits of the surgery
to the family and discusses the client's wishes with
the family. The nurse is acting as the client's:

A) Manager
B) Educator
C) Advocate
D) Caregiver

Answer: C

Evidence-based practice is a problem-solving


approach to clinical practice that uses the best
available evidence, along with the nurse's
expertise and the client's preference and values,
in making decisions about care. The other
answers are incorrect.
Evidence-based practice is defined as:

A) Nursing care based on tradition


B) Scholarly inquiry embodied in the nursing and
biomedical research literature
C) A problem-solving approach to clinical practice
based on best practices
D) Quality nursing care provided in an efficient
and economically sound manner

Answer: D

The RN licensure examination provides a


minimum standard of knowledge for nurses. The
examination cannot guarantee or ensure care for
clients.
The examination for the registered nurse (RN)
licensure is exactly the same in every state in the
United States. This examination:

A) Guarantees safe nursing care for all clients


B) Ensures standard nursing care for all clients
C) Ensures that honest and ethical care is provided
D) Provides a minimal standard of knowledge for
practice

Answer: B

APNs are generally the most independently


functioning nurses. An APN can work in a
primary, acute, or restorative care setting. The
setting may be a private, public, or university
facility. The APN may function as a clinician,
educator, case manager, consultant, or
researcher.
Advanced practice nurses (APNs) generally:

A) Work in acute care settings


B) Function independently
C) Function as unit directors
D) Work in the university setting

Answer: A

The client is the correct choice. The health care


facility is where the client goes to receive
treatment. The nursing process is how nurses
proceed to plan care for the client. Cultural
diversity is not the correct choice.
Nursing practice in the twenty-first century is an
art and science that is centered on:

A) The client
B) The nursing process
C) Cultural diversity
D) The health care facility

Answer: D

Florence Nightingale is the correct choice.


Barton founded the Red Cross. Dix organized
hospitals, nurses, and supply lines to support the
troops of the Union Army. Wald opened the first
community health service for the poor.
Who acted to decrease mortality by improving
sanitation in the battlefields, which resulted in a
decline in illness and infection?
A) Dorothea Dix
B) Lillian Wald
C) Clara Barton
D) Florence Nightingale

Answer: B

Mary Mahoney, the first African American


professional nurse, worked to bring respect to
individuals regardless of race, color,
background, or religion. Tubman assisted slaves
to freedom during the Civil War. Hampton
founded the Nurses Associated Alumnae of the
United States and Canada, which later became
the American Nurses Association (ANA).
Nutting was instrumental in the affiliation of
nursing education with universities.
The professional nurse responsible for increasing
respect for the individual and awareness of cultural
diversity was:

A) Harriet Tubman
B) Mary Mahoney
C) Isabel Hampton
D) Mary Adelaide Nutting

Answer: C

Healthy People 2010, a federal document,


outlines goals for the public. Notes on Nursing
set forth Nightingale's first nursing philosophy.
The Last Acts Campaign has developed
standards and policies for end-of-life care.
Nursing Principles and Practice 2010—current
readings in journals are necessary for all nurses
in practice.
The document that developed goals and objectives
to meet the health of the public is known as:

A) Notes on Nursing
B) Last Acts Campaign
C) Healthy People 2010
D) Nursing Principles and Practice 2010

Answer: B

This process may be carried out with other


members of the health care team, and client and
family members may be included. All nurses
use critical thinking. An advanced care nurse
has advanced educational preparation. An
evidence-based practitioner draws on research
findings as well as clinical expertise and client
values. A multidisciplinary practice includes
health care members from various fields of
activity, such as physical therapy and dietary
therapy, along with nursing.
A nurse who uses critical thinking in the decision-
making process to provide effective quality care to
individuals is known as:

A) An advanced care nurse


B) A clinical decision maker
C) A member of a multidisciplinary practice
D) An evidence-based practitioner

Answer: A

Standards of care describe the competency level


of nursing care as described by the ANA. The
Nurse Practice Act regulates the licensing and
practice of nursing; it describes the scope of
practice. Accreditation allows the facility,
school, or hospital to operate and be recognized
in good standing according to standards set by
peers. National council licensure is the
standardized national examination that assess
for a minimum knowledge base relevant to the
client population that the nurse serves.
Which of the following assures clients that they
will receive quality care from a competent nurse?

A) Standards of care
B) Nurse Practice Act
C) Accreditation certification
D) National council licensure

Answer: B

The Nurse Practice Act regulates the license and


practice of nursing; it describes the scope of
practice and is the correct answer. The NCLEX-
RN national licensure examination is
administered in each state to test that candidates
have the minimum knowledge level required for
practice. Passage of an examination and
requirements for certification signify additional
knowledge and competence in a specific area.
The ANA Congress for Nursing is an
organization that addresses legal aspects of
nursing practice.
The licensure and practice of nursing is regulated
by:

A) The NCLEX-RN
B) The Nurse Practice Act
C) The certification examination
D) The ANA Congress for Nursing

Answer: C

The nurse who has held the same position for 2


to 3 years and understands the specific area and
client population is termed a competent nurse.
The expert is a nurse with diverse experience
who can focus on a specific problem and offer
multidimensional solutions. The proficient nurse
has more than 2 to 3 years' experience and
applies knowledge and experience to a situation.
The advanced beginner nurse has at least some
level of experience.
A nurse who has filled a position on the same unit
for 2 years understands the unit's organization and
the care of the clients on that nursing unit. Benner
defines this nurse as able to anticipate nursing care
and to formulate long-range goals; this nurse is
given the title:
A) Expert nurse
B) Proficient nurse
C) Competent nurse
D) Advanced beginner

Answer: A

Care provider is a staff position, a nurse who


provides direct care. The nurse specialist has
clinical expertise in a specific area. The nurse
practitioner has advanced training in assessment
and pharmacology and is able to provide health
care in specific settings. The case manager has
additional experience and is able to coordinate
activities of other members of the health care
team.
An APN is the most independently functioning of
all professional nurses. All of the following are
examples of a clinically focused APN except:

A) Care provider
B) Case manager
C) Nurse specialist
D) Nurse practitioner

Answer: D

Additional training in anesthesia medicine


would be required to be a certified registered
nurse anesthetist.
An APN is pursuing a job change. Which of the
following positions would the APN be unable to
fill without meeting additional criteria?

A) Case manager
B) Nurse manager
C) Nurse educator
D) Certified registered nurse anesthetist

Answer: A
National League for Nursing (NLN) is the
correct answer. The master of science in nursing
(MSN) degree is earned through advanced
educational preparation in nursing. Public
Health Administration (PHA) is concerned with
areas of public health. The National Institutes of
Health (NIH) addresses health on a national
level.
Which of the following professional organizations
was created to address concerns of members in the
nursing profession?

A) NLN
B) MSN
C) PHA
D) NIH

Answer: A, B, C, D

Each of the options is an example of a


professional role or responsibility of the
professional nurse.
Contemporary nursing requires that the nurse
possess knowledge and skills to carry out a variety
of professional roles and responsibilities.
Examples include which of the following? (Select
all that apply.)

A) Autonomy and accountability


B) Advocacy
C) Provision of bedside care
D) Health promotion and illness prevention

Answer: C

The federal government, which pays for the


Medicare and Medicaid programs, is the biggest
consumer of health care. The other options are
incorrect.
Which of the following is the biggest consumer of
health care?
A) Hospitals
B) Businesses
C) Federal government
D) Private insurance companies

Answer: C

The prospective payment system is one of the


most significant factors influencing payment for
health care. The prospective payment system
groups payments into diagnosis-related groups
for Medicare and Medicaid clients. Managed
care organizations are systems in which there is
administrative control over primary health care
services for a defined client population.
Which of the following was most significant in
influencing competition in health care costs?

A) Medicare and Medicaid


B) Diagnosis-related groups
C) Prospective payment system
D) Managed care organizations

Answer: B, C, D

Evidence-based practice helps nurses to solve


dilemmas in the clinical setting because it
combines scientific research with clinical
expertise and local values. Evidence-based
practice does require nurses to review and
critique research and practice findings. Nurses
are expected to always meet the standards of
practice.
Which of the following statements is true about
evidence-based practice? (Select all that apply.)
Evidence-based practice:

A) Is based only on the results of research


B) Assists nurses in meeting standards of practice
C) Helps nurses solve dilemmas in the clinical
setting
D) Requires nurses to review and critique research
and practice findings

Answer: D

Evidence-based practice draws on both research


and clinical experience. Competencies are
evidence that skills have been demonstrated.
Critical thinking is the questioning thought
process that nurses need to use in practice.
Primary care is health care provided in the
community by one caregiver who takes
responsibility for managing a client's care.
The nurse found that using tympanic thermometers
was quick, easy, and yielded temperatures as
reliable as those obtained using oral thermometers.
This finding represents:

A) Primary care
B) Critical thinking
C) Competency testing
D) Evidence-based practice

Answer: A

In the staff model of an MCO, the physicians


are salaried employees. In the group model, the
MCO contracts with a single group practice. An
independent practice association is a group of
physicians who are under contact to the
organization but are not members of it and
whose practices include fee-for-service and
capitated clients. The MCO contracts with
multiple group practices and/or integrated
organizations in the network model.
A client is receiving health care from a health care
provider who is a salaried employee. Which model
is being followed by the managed care
organization (MCO) to which the client belongs?
(Select all that apply.)

A) Staff model
B) Group model
C) Network model
D) Independent practice association

Answer: D

The utilization review committee reviews


admissions, diagnostic procedures, and
treatments ordered by physicians. Review of the
quality, quantity, and cost of care is more
similar to the functions of a professional
standards review organization. Review of
reimbursement fees and appropriation of funds
involves review of diagnosis-related groups.
Reviewing the utilization of the payment
mechanism is similar to capitation.
The purpose of a utilization review committee is
to:

A) Review quality, quantity, and cost of care


B) Review the utilization of the payment
mechanism
C) Review reimbursement fees and appropriation
of funds
D) Review admissions, diagnostic tests, and
treatments ordered by physicians

Answer: C

This is the description of an MCO. In a PPO,


choice of care providers is limited to those listed
in the group. Medicare is a federally funded
national health insurance program. Private
insurance is a traditional fee-for-service plan.
The client's health insurance changed, and instead
of having a limited number of physicians from
whom to choose, the client is voluntarily enrolled
in a plan in which medical care is provided by a
special group of caregivers. This arrangement is
known as:

A) Medicare
B) Private insurance
C) Managed care organization (MCO)
D) Preferred provider organization (PPO)

Answer: A

A positive benefit of a professional nursing staff


is a decreased length of stay. The diagnosis-
related group has greater influence on the rate of
readmission. The ancillary personnel need to
remain so that registered nurses can spend the
necessary time to assess and manage clients.
Nosocomial infections decrease with a
professional nursing staff.
Recent research provided evidence that a
professional nursing staff affects health care
financing. These results indicated that the positive
benefit of a professional nursing staff is:

A) Decreased length of stay


B) Decreased rate of readmission
C) Increased rate of nosocomial infections
D) Decreased need to hire ancillary personnel

Answer: C

Health promotion includes dietary counseling.


Blood glucose monitoring at the pharmacy is an
example of illness prevention. Restorative care
is care of a client who, for instance, is
recovering from complications of diabetes. Any
diagnostic procedure or tests completed in the
hospital would be examples of such care.
The nurse is giving discharge instructions to a
client with newly diagnosed diabetes. The nurse
discusses with the client what the dietary intake
should be. This is an example of which health care
service?

A) Tertiary care
B) Restorative care
C) Health promotion
D) Illness prevention

Answer: D

Taking blood pressure measurements is illness


prevention. Health promotion includes activities
like exercise classes. Secondary care is often
known as traditional care. It would include
rehabilitation after a stroke in an individual with
a history of elevated blood pressure.
A nurse volunteers to take blood pressure
measurements after church services. This is an
example of which level of health care service?

A) Secondary care
B) Restorative care
C) Health promotion
D) Illness prevention

Answer: C

A critical pathway is a multidisciplinary


treatment plan with interventions prescribed
within a structured framework. A discharge plan
includes an assessment and anticipation of the
client's needs. Medicare is a federal health
insurance plan for those 65 years of age and
older. Standard nursing care is the minimum
care to be given to a client.
The nurse completes the standard orders on a
client's first day postoperatively. The instrument
that is used to coordinate the client's care is:

A) A Medicare plan
B) A discharge plan
C) A critical pathway
D) Standard nursing care

Answer: C

Case management is a model of organizing care


in which the case manager monitors, directs, and
advises the nursing care personnel on specific
care issues and the progress of a client. In team
nursing, care might be provided by groups
composed of registered nurses, licensed
practical nurses, and possibly assistive
personnel. Nursing process is used to plan the
nursing care for a client. Interdisciplinary care is
care provided by a team whose members come
from a variety of disciplines.
The multidisciplinary care model used to move
clients efficiently from admission to discharge is
known as:

A) Team nursing
B) Nursing process
C) Case management
D) Interdisciplinary care

Answer: D

Restorative care assists an individual in


regaining the maximum possible level of
functioning. Home care includes professional
and paraprofessional services that are rendered
in the home setting. Extended care is
intermediate medical or nursing care for
individuals with an acute or chronic illness or
disability. Assisted care is a setting in which the
client is able to function at a higher level of
autonomy within a homelike environment but in
which care can be given when needed.
A client discharged after suffering a stroke is
transferred from a tertiary care facility to another
facility for additional care to help the client
recover and continue to regain function. This type
of care facility is known as:

A) Home care
B) Assisted care
C) Extended care
D) Restorative care

Answer: A

Day care is an example of respite care because it


allows the family to maintain normalcy while
the client is under their care. A nursing home
client receives 24-hour care in the facility.
Home care is an intermittent service in which
only certain tasks are performed. Nurse
extenders may be hired to perform a specific
task, such as bathing.
Which of the following is an example of respite
care?

A) Day care
B) Home care
C) Nursing home
D) Nurse extender

Answer: C

Obtaining a sterile specimen requires insertion


of a catheter, a procedure that must be
performed by a licensed nurse. Therefore, this
would not be an appropriate task to delegate to
an assistive person. Assistive personnel would
be able to ambulate a client, give a bed bath, and
add to the I&O record.
Which task is it not appropriate for a professional
nurse to delegate to assistive personnel?

A) Ambulate a client
B) Complete a bed bath
C) Obtain a sterile urine specimen
D) Complete the intake and output (I&O) record

Answer: B, C

The case manager coordinates the efforts of all


disciplines to achieve the most efficient and
appropriate plan of care for the client, with a
focus on discharge planning. Therefore,
coordination of transfer to a step-down
rehabilitation unit and follow-up after discharge
to evaluate that needs have been met are the
correct answers.
A nurse is working in an acute care hospital that
uses a case management model. About which of
the following activities should the nurse
communicate with the case manager? (Select all
that apply.)

A) Management of a client transfer to the


radiology department
B) Coordination of a client transfer to the step-
down rehabilitation unit
C) Follow-up after a client's discharge to evaluate
whether needs have been met
D) Permission for a family to bring in special food
for a client

Answer: A, C

Clients being discharged home need education


regarding how to take their medication and
when to call their health care provider. There is
not enough information here to determine if
options 2 and 4 are appropriate, although hand
hygiene after toileting is always important.
A nurse is planning a client's discharge from a
subacute care unit to home. Education should be
provided on which of the following topics? (Select
all that apply.)

A) Medication administration
B) Stress reduction techniques with blood pressure
assessment
C) Circumstances in which the client should call
the health care provider
D) Hand-washing hygiene when assisting with
transfer to the bathroom

Answer: B, D

Extended care encompasses intermediate


medical, nursing, or custodial care for clients
recovering from acute illness or clients with
chronic illnesses or disabilities. Extended care
facilities include intermediate care facilities and
skilled nursing facilities.
Which of the following clients should be cared for
in an extended care facility with skilled nursing?
(Select all that apply.)

A) Client who had a stroke, can talk, and has lost


bowel and bladder control
B) Severely brain injured client on a ventilator
who is receiving intravenous medications
C) Client with Alzheimer's disease who is abusive,
combative, and a threat to self and others
D) Young child who recently had a spinal cord
injury and is living with quadriplegia and needs to
learn a new way of life

Answer: D

Healthy People 2010 was established to create


ongoing health care goals, including increasing
life expectancy and quality of life, and eliminate
health disparities through improved delivery of
health care services. Gathering information,
assessing needs, and developing and
implementing public health policies are steps in
achieving the goals set forth by Healthy People
2010.
Healthy People 2010's overall goals are to:

A) Assess the health care needs of individuals,


families, or communities
B) Develop and implement public health policies
and improve access to care
C) Gather information on incident rates of certain
diseases and social problems
D) Increase life expectancy and quality of life and
eliminate health disparities

Answer: C

Substance abusers avoid health care for fear of


judgmental attitudes by health care providers
and concern about being turned in to the
criminal authorities. Options 1, 2, and 4 are not
primary concerns that result in avoidance of
health care.
Substance abusers frequently avoid health care
providers because of:

A) Fear of the cost of health care


B) Fear of institutions and people
C) Fear of being turned in to the criminal
authorities
D) Fear of being without the recreational drug of
choice

Answer: D

Vulnerable population are defined as clients


who are more likely to develop health problems
as a result of excess risks, who have limits in
access to health care services, or who are
dependent on others for care.
Vulnerable populations of clients are those who are
more likely to develop health problems as a result
of:

A) Chronic diseases, homelessness, and poverty


B) Poverty and limits in access to health care
services
C) Lack of transportation, dependence on others
for care, and homelessness
D) Excess risks, limits in access to health care
services, and dependence on others for care

Answer: A

Secondary intervention includes disease


prevention after a health issue has been
identified. Primary intervention is prevention of
a health problem that has not yet occurred in the
community. Tertiary intervention occurs after a
problem has occurred and aims at preventing
long-term negative impacts or recurrences in a
population.
The local health department received information
from the Centers for Disease Control and
Prevention that the flu was expected to be very
contagious this season. The nurse is asked to set up
flu vaccine clinics in local churches and senior
citizen centers. This activity is an example of
which level of prevention?

A) Primary intervention
B) Tertiary intervention
C) Nursing intervention
D) Secondary intervention

Answer: A

An educator helps clients, families, and


communities gain greater skills and knowledge
to provide their own care. An advocate is
someone who helps clients walk through the
system, identifies services, and plans for
accessing appropriate resources. A collaborator
is an individual who engages in a combined
effort with other individuals to develop a
mutually acceptable plan that will achieve
common goals. A case manager develops and
implements a plan of care.
The local school has an increasing number of
adolescent parents. The nurse works with the
school district to design and teach classes about
infant care, child safety, and time management.
These are examples of which nursing role?

A) Educator
B) Advocate
C) Collaborator
D) Case manager

Answer: B

A counselor helps clients identify and clarify


health problems and choose appropriate courses
of action to solve those problems. An educator
helps the community gain greater skills,
including through the presentation of
educational programs. A collaborator is an
individual who engages in a combined effort
with other individuals to develop a mutually
acceptable plan that will achieve common goals.
A nurse is practicing in an occupational health
setting. There are a large number of employees
who smoke, and the nurse designs an employee
assistance program for smoking cessation. This is
an example of which nursing role?

A) Educator
B) Counselor
C) Collaborator
D) Case manager

Answer: C

The community has three components: structure


or locale, people, and social systems. To
develop a complete community assessment, the
nurse must take a careful look at each of the
three components to begin to identify needs for
health policy, health programs, and health
services.
What are the three elements included in a
community assessment?

A) Environment, families, and social systems


B) People, neighborhoods, and social systems
C) Structure or locale, people, and social systems
D) Health care systems, geographic boundaries,
and people

Answer: C

Option 3 defines the focus of community health


nursing. Community health nursing focuses on
the individual, family, and community.
Educational requirements for community-based
nurses are not as clearly defined as those for
public health nurses. An advanced degree is not
always required.
The focus of community health nursing differs
from that of public health nursing because the
nursing care:

A) Is directed at the individual client only


B) Is provided by nurses with a graduate degree in
community health nursing
C) Provides direct care to subpopulations who
make up the community as a whole
D) Is administered to a collection of individuals
who have in common one or more personal or
environmental characteristics

Answer: B

In Healthy People 2010, the assurance role of


public health is defined as making essential
community-wide health services available and
accessible. In Healthy People 2010, public
development and implementation refer to the
role of health professionals in providing
leadership in development of policies that
support the population's health. Population-
based public health programs focus on disease
prevention, health promotion, and health
protection. A healthy environment for each
individual, family, and community is the overall
goal of Healthy People 2010.
In Healthy People 2010, assurance refers to the
role of public health in:

A) Providing disease prevention, health protection,


and health promotion
B) Making essential community-wide health
services available and accessible
C) Providing leadership in developing policies that
support the population's health
D) Achieving a healthy environment for each
individual, family, and community

Answer: B

Assessing the learner's needs and readiness to


learn are important to increase the success of the
learning process. Options A and D are negative
responses and would block the learning process.
Repeating the old teaching plan is
nonproductive and an inefficient application of
the nursing process.
A home care nurse educator has repeatedly
counseled a 33-year-old male diabetic client
concerning the need for dietary compliance. In
writing an effective teaching plan the nurse will
first:

A) Reprimand the client for noncompliant


behavior
B) Assess the client's learning needs and readiness
to learn
C) Repeat the old teaching plan to ensure the
client's comprehension
D) Provide a detailed description of complications
associated with the disease process

Answer: A

Vulnerable populations are defined as specific


populations with unique health care problems.
Vulnerable populations are not limited to the
very young or older adults. Such individuals are
those living in poverty, homeless persons,
abused clients, substance abusers, and so on.
Members of most vulnerable populations come
from different cultures and have different beliefs
and values. Vulnerable populations are at risk of
experiencing poorer outcomes in response to
interventions because of the multiple stressors
that affect their daily lives.
Vulnerable populations are more likely to develop
health problems. Which of the following is true of
these populations?

A) They are specific populations with unique


health care problems.
B) They are limited to the very young and older
adult age groups.
C) They live in communities with similar cultures,
beliefs, and values.
D) They frequently experience positive outcomes
in response to community health interventions.

Answer: C

A case manager's competency is defined as the


ability to establish an appropriate plan of care
that is based on assessment of clients and
families and coordinates the provision of needed
resources and services across a continuum of
care. A collaborator's competency is described
as engaging in a combined effort with all those
involved in care delivery. A change agent's
competency is to implement new and more
effective approaches to problems. A client
advocate presents the client's point of view so
that appropriate resources can be obtained.
A competent community-based nurse must be
skilled in fulfilling a variety of roles. The ability to
establish an appropriate plan of care that is based
on assessment of clients and families and
coordinates the provision of needed resources and
services across a continuum of care defines the
competency of:

A) Collaborator
B) Change agent
C) Case manager
D) Client advocate

Answer: D

No individual client assessment should occur in


isolation from the environment and conditions
of the client's community. Industrial
development, types of pollution, and cultural
and religious groups are individual elements in
the community.
When completing an individual total assessment of
a client, the community-based nurse will include
consideration of:

A) The type of pollution present in the community


B) The amount of industrial development in the
past 5 years in the community
C) The predominant cultural and religious groups
found in the community
D) The community structures, the population, and
the local social system in which the client lives

Answer: A

When dealing with clients who are at risk for or


may have suffered abuse, it is important to
provide protection. Educating the mother on the
developmental issues of her infant is important
but provides no protection for the victim.
Providing protection and eliminating the fear of
retribution is a priority upon discovery of abuse.
By disregarding the mother's situation, the nurse
has failed to intervene for the family in crisis in
the community.
During a well-baby visit, the community-based
nurse observed patterned bruises and skin
abrasions on the face, arms, and throat of the
infant's 21-year-old mother. In questioning the
mother, the nurse discovers that she is a recent
victim of spousal abuse. An important principle in
dealing with this client is:

A) Ensuring the protection of the mother


B) Informing the authorities of the attack
C) Educating the mother on well-baby
developmental issues
D) Continuing with the well-baby examination and
disregarding the mother's situation

Answer: D

Change must be perceived as advantageous,


compatible with existing values, and easily
adaptable to be successful and accepted. Up-
front cost, managerial framework, building
plans, contractors, compliance with building
codes, and regulations for governmental
agencies are all incorporated in proposals but do
not provide convincing reasoning that leads to
change.
A proposal written by a community-based nurse
for a new, higher quality older adult care center
will have increased probability of acceptance if the
proposal includes:

A) All building plans and a list of contractors to


complete the job
B) Compliance with the codes and building
requirements of local government agencies
C) The up-front cost and managerial framework of
the new older adult center
D) Description of how advantageous, realistic,
compatible, and adaptable the change will be when
implemented

Answer: D

The homeless person's lack of a storage site for


medication and inability to obtain nutritious
meals are factors that contribute to poor
management of chronic disease. Homeless
people are often stereotyped as having a lack of
concern for their situations. Poor attire and lack
of hygiene are not causes of chronic illness
exacerbation. They are signs of the client's
status as a member of an at-risk population. It is
incorrect for the nurse to assume that the client
lacks education and the ability to read.
A nurse is caring for a 64-year-old homeless
woman with a chronic respiratory disease in the
local community-based clinic. The nurse realizes
that the client is at risk of experiencing
exacerbation of the disease process related to:
A) Poor attire and cleanliness practices
B) The client's lack of education and ability to read
C) The individual's lack of concern about the
disease
D) The client's lack of a storage site for medication
and the inability to obtain nutritious meals

Answer: A, B, C, D

In this case, all four options are correct. The


community health nurse is providing
information for the community and helping its
members learn to access the help that is
available, but not dictating the steps that need to
be taken.
A community health nurse is caring for members
of a Bosnian community. The nurse determines
that the children are undervaccinated and that the
community is unaware of this resource. As the
nurse assesses the community, the nurse
determines that there is a health clinic within 5
miles. The nurse meets with the community
leaders and explains the need for immunizations,
the location of the clinic, and the process for
accessing the health care resources. Which of the
following is the nurse doing? (Select all that
apply.)

A) Improving children's health care


B) Teaching the community about illness
C) Educating about community resources
D) Promoting autonomy in decision making

Answer: A, B, C, D

All are factors that will impact the client's


potential to change.
A nurse at the community clinic nurse cares for a
40-year-old woman who takes insulin to manage
diabetes. She is having increasing difficulty
controlling the disease, and the nurse wants her to
try a new insulin pump to help her manage her
diabetes. Which of the following change factors
increase the likelihood that she will accept this
new insulin pump? (Select all that apply.)

A) The innovation or change must be perceived as


more advantageous than other alternatives.
B) The innovation or change must be compatible
with existing needs, values, and past experiences.
C) The innovation must be tried on a limited basis.
D) Simple innovations or changes are more readily
adopted than those that are complex.

Answer: D

Nursing's paradigm includes four linkages: the


person, health, environment/situation, and
nursing.
Nursing's paradigm includes:

A) Health, person, environment, and theory


B) Concepts, theory, health, and environment
C) Nurses, physicians, models, and client needs
D) The person, health, environment/situation, and
nursing

Answer: C

Prescriptive theories address nursing


interventions for a phenomenon and predict the
consequence of a specific nursing intervention.
Descriptive theories describe the phenomena,
speculate on the reason the phenomena occur,
and predict nursing phenomena. Grand theories
are broad and complex and provide a structural
framework for broad, abstract ideas about
nursing.
Which of the following statements about
prescriptive theories is accurate?

A) They describe phenomena.


B) They have the ability to explain nursing
phenomena.
C) They reflect practice and address specific
phenomena.
D) They provide a structural framework for broad
abstract ideas.

Answer: B

A theory is a set of concepts, definitions,


relationships, and assumptions that explains a
phenomenon. Theories do not formulate
legislation, measure nursing functions, or reflect
any domain of nursing practice.
A theory is a set of concepts, definitions,
relationships, and assumptions that:

A) Formulates legislation
B) Explains a phenomenon
C) Measures nursing functions
D) Reflects the domain of nursing practice

Answer: C

Theories will be tested to describe or predict


client outcomes as nursing is addressed as a
science and an art. Scientists will not make
nursing decisions, and nursing will base client
care on the practice of nursing science, which
will be guided by multiple theories.
There is a contemporary move toward addressing
nursing as a science or as evidenced-based
practice. This suggests that:

A) One theory will guide nursing practice.


B) Scientists will make nursing decisions.
C) Theories will be tested to describe or predict
client outcomes.
D) Nursing will base client care on the practice of
other sciences.

Answer: C
Interdisciplinary theories provide a systematic
view of a phenomenon. Developmental theories,
health and wellness theories, and systems
theories are examples of other types of theories.
To practice in today's health care environment,
nurses need a strong scientific knowledge base in
nursing and other disciplines, such as the physical,
social, and behavioral sciences. This relates to
which of the following?

A) Systems theories
B) Developmental theories
C) Interdisciplinary theories
D) Health and wellness model

Answer: B

Developmental theories discuss human growth


from conception to death. The other options are
incorrect.
Which theories describe an orderly process
beginning with conception and continuing through
death?

A) Systems theories
B) Developmental theories
C) Interdisciplinary theories
D) Stress and adaptation theories

Answer: C

The first level of Maslow's hierarchy of needs


includes the need for air, food, and water—basic
elements of survival. Love and belonging are on
the second level, esteem and self-esteem are on
the fourth level, and self-actualization is the
final level.
Maslow's hierarchy of needs is useful to nurses,
who must continually prioritize a client's nursing
care needs. The most basic or first-level needs
include:
A) Self-actualization
B) Love and belonging
C) Air, water, and food
D) Esteem and self-esteem

Answer: A

The goal of Leininger's theory is to provide the


client with culturally specific nursing care, in
which the nurse integrates the client's cultural
traditions, values, and beliefs into the plan of
care.
Leininger's theory of cultural care diversity and
universality specifically addresses:

A) Caring for clients from unique cultures


B) Understanding the humanistic aspects of life
C) Identifying variables affecting a client's
response to a stressor
D) Caring for clients who cannot adapt to internal
and external environmental demands

Answer: D

As a science, nursing draws on scientifically


tested knowledge applied in the practice setting.
As an art, nursing relies on knowledge gained from
practice and reflection on past experiences. As a
science, nursing relies on:

A) Experimental research
B) Nonexperimental research
C) Physician-generated research
D) Scientifically tested knowledge

Answer: C

The domain contains the subject, central


concepts, values and beliefs, phenomena of
interest, and the central problems of the
discipline. A paradigm is a model that explains
the linkage of science, philosophy and theory
that is accepted and applied by the discipline.
Each science has a domain, which is the
perspective of the discipline. This domain:

A) Represents the recipients of the benefits of the


science or discipline
B) Is a model that explains the linkage of science,
philosophy, and theory that is accepted and applied
by the discipline
C) Describes the subject, central concepts, values
and beliefs, phenomena of interest, and central
problems of the discipline
D) Is a dynamic state of being in which the
developmental and behavioral potential of the
individual is realized to the fullest

Answer: D

Describing, explaining, predicting, and/or


prescribing interrelationships among concepts
are stated purposes of research.
A theory is a set of concepts, definitions,
relationships, and assumptions or propositions to
explain a phenomenon. The purposes of the
components of a theory are to:

A) Describe concepts or connect two concepts that


are factual
B) Formulate a perceptual experience to describe
or label a phenomenon
C) Express the global view about the individual,
situations, or factors of interest to a specific
discipline
D) Describe, explain, predict, and/or prescribe
interrelationships among the concepts that define
the phenomenon

Answer: A
Phenomena are defined as aspects of reality that
can be consciously sensed or experienced.
Nursing theories focus on the phenomena of
nursing and nursing care. Which of the following
is true of phenomena?

A) They are aspects of reality that can be


consciously sensed or experienced.
B) They convey the general meaning of concepts
in a manner that fits the theory.
C) They are statements that describe concepts or
connect two concepts that are factual.
D) They are mental formulations of an object or
event that come from individual perceptual
experience.

Answer: A

Grand theories are described as broad and


complex. Middle-range theories are limited in
scope, less abstract, address specific phenomena
or concepts, and reflect practice. Descriptive
theories describe phenomena, speculate as to
why the phenomena occur, and describe the
consequences of phenomena. Prescriptive
theories address nursing interventions and
predict the consequence of a specific
intervention.
Theories that are broad and complex are:

A) Grand theories
B) Descriptive theories
C) Middle-range theories
D) Prescriptive theories

Answer: D

Middle-range theories are limited in scope, less


abstract than grand theories, address specific
phenomena or concepts, and reflect practice.
Grand theories are described as broad and
complex. Prescriptive theories address nursing
interventions and predict the consequence of a
specific nursing intervention. Descriptive
theories describe phenomena, speculate as to
why the phenomena occur, and describe the
consequences of phenomena.
Mishel's theory of uncertainty in illness focuses on
the experience of clients with cancer who live with
continual uncertainty. The theory provides a basis
for nurses to assist clients in appraising and
adapting to the uncertainty and illness response
and can be described as:

A) A grand theory
B) A descriptive theory
C) A prescriptive theory
D) A middle-range theory

Answer: C

Prescriptive theory addresses nursing


interventions and predicts the consequence of a
specific nursing intervention. Middle-range
theories are limited in scope, less abstract than
grand theories, address specific phenomena or
concepts, and reflect practice. Descriptive
theories describe phenomena, speculate as to
why the phenomena occur, and describe the
consequences of phenomena. Grand theories are
broad and complex.
The type of theory that tests the validity and
predictability of nursing interventions is:

A) A grand theory
B) A descriptive theory
C) A prescriptive theory
D) A middle-range theory

Answer: B
Identified linkages of a nursing paradigm are the
person, health, environment/situation, and
nursing itself. Concepts, definitions,
relationship, and assumptions are components of
a theory. The individuals, groups, situations, and
interests specific to nursing are potential
subjects for middle-range theories. Description,
explanation, prediction, and prescription of an
interrelationship of nursing are purposes of
nursing theory.
The nursing paradigm identifies four linkages of
interest to the nursing profession. These four
linkages are:

A) Concepts, definitions, relationships, and


assumptions
B) The person, health, environment/situation, and
nursing
C) The individual, groups, situations, and interests
specific to nursing
D) Description, explanation, prediction, and
prescription of an interrelationship of nursing

Answer: C

An open system is defined as a system that


interacts with the environment, exchanging
information between the system and the
environment.
The nursing process is an example of an open
system. An open system:

A) Is universal and dynamic


B) Represents a relationship between two concepts
C) Interacts with the environment by exchanging
information
D) Is a process through which information is
returned to the system

Answer: D
The result of theory-generating or theory-testing
research is to increase the knowledge base of
nursing. As these research activities continue,
clients become the recipients of evidence-based
nursing care.
Evidence-based nursing practice is the end result
of:

A) Prescriptive theory
B) Use of practical knowledge
C) Application of theoretical knowledge
D) Theory-generating and theory-testing research

Answer: B

The question describes the nursing theory


developed by Fay Abdellah and others. Rogers'
theory considered the individual as an energy
field existing within the universe. Henderson's
theory defines nursing as "assisting the
individual, sick, or well, in the performance of
those activities that will contribute to health,
recovery, or a peaceful death." Nightingale
viewed nursing as providing fresh air, light,
warmth, cleanliness, quiet, and adequate
nutrition.
The nursing theory that emphasizes the delivery of
nursing care for the whole person to meet the
physical, emotional, intellectual, social, and
spiritual needs of the client and family is:

A) Rogers' theory
B) Abdellah's theory
C) Henderson's theory
D) Nightingale's theory

Answer: B

A qualitative study involves inductive reasoning


to develop generalizations or theories from
specific observations or interviews. Historical
research establishes facts and relationships
concerning past events. Correlational research
explores the interrelationships among variables
of interest without any intervention by the
researcher. An experimental study used tightly
controlled subject groups, variables, and
procedures to eliminate bias and ensure that
findings can be generalized to similar groups of
subjects.
A nurse researcher interviews senior oncology
nurses, asking them to describe how they deal with
the loss of a client. The analysis of the interviews
yields common themes describing the nurses' grief.
This is an example of which type of study?

A) Historical study
B) Qualitative study
C) Correlational study
D) Experimental study

Answer: B

Surgical clients are the client population of


interest (P) in the PICO (population,
intervention, comparison, outcome) question.
Chlorhexidine use is the comparison of interest,
and povidone-iodine use is the intervention of
interest. The operating room nurse is not an
element of the PICO question.
An operating room nurse is talking with colleagues
during a meeting and asks, "I wonder if we would
see fewer wound infections if we used
chlorhexidine instead of povidone-iodine to clean
the skin of our surgical clients? In this example of
a PICO question, the P is:

A) Povidone-iodine use
B) Surgical clients
C) Chlorhexidine use
D) Operating room nurses

Answer: A

Because the clients at the clinic are allowed


their choice of the traditional versus the new
exercise program, the sampling in this study is
not random sampling and can bias study results.
A nurse researcher is designing an exercise study
that involves 100 clients who attend a wellness
clinic. As the clients come to the clinic, each has a
choice as to whether they want to be in the new
exercise program or remain in the traditional
program. The nurse plans to measure the clients'
self-report of exercise before and 6 months after
the program begins. What factor might influence
the results of this study in an unfavorable way?

A) Bias
B) Anonymity
C) Sample size
D) Sampling method

Answer: D

The scientific method is the foundation of


research and the most reliable and objective of
all methods of gaining knowledge. Experience,
critical thinking, and evidence are not the
foundation of research.
The foundation of research is which of the
following?

A) Evidence
B) Experience
C) Critical thinking
D) Scientific method

Answer: D

Informed consent means that the research


subjects are given full and complete information
about the purpose of the study, procedures, data
collection, potential harm and benefits, and
alternate methods of treatment. Confidentiality
rules guarantee that any information the subject
provides will not be reported to people outside
the research team. Bias is any personal opinion
or judgment that may be interjected into the
results.
A researcher gives a subject full and complete
information about the purpose of a study. This is
an example of:

A) Bias
B) Anonymity
C) Confidentiality
D) Informed consent

Answer: D

A knowledge-focused trigger is a question


regarding new information available on a topic.
A problem-focused trigger is one faced while
caring for a client or noting a trend. The PICO
(population, intervention, comparison, outcome)
format is a way to phrase a question to help
clarify the question and the parts. A hypothesis
is a prediction about the relationship between
study variables.
A new nurse on an orthopedic unit is assigned to
care for a client undergoing skeletal traction. The
nurse asks a colleague, "What is the best practice
for cleaning pin sites in skeletal traction?" This
question is an example of which of the following?

A) Hypothesis
B) PICO question
C) Problem-focused trigger
D) Knowledge-focused trigger

Answer: A
The "Do" step consists of selecting an
intervention based on a data review and
implementing the change, plus studying the
results of the change. The "Plan" step includes
reviewing the available data to understand
existing practice conditions or problems to
identify the need for change. The results of the
change are evaluated in the "Study" step. The
"Act" step is the incorporation of the findings
into current practice.
The nurses on a medical unit have seen an increase
in the number of pressure ulcers developing in
their clients. The nurses decide to initiate a quality
improvement project using the PDSA (plan, do,
study, act) model. Which of the following is an
example of the "Do" step of that model?

A) Implement a new skin care protocol on all


medical units.
B) Review the data collected on clients cared for
using the new protocol.
C) Review the quality improvement reports on the
six clients who developed ulcers over the previous
3 months.
D) Based on findings from clients who developed
ulcers, implement an evidence-based skin care
protocol.

Answer: C

The conduct of research must meet ethical


standards in which the rights of human subjects
are protected. The research participants must be
told about the study's purpose and procedure,
and their roles in the study. The researcher is
always legally responsible for his or her actions.
Control of variables is related to the study
design, not to informed consent. Confidentiality
is part of the ethical nature of research but is not
the focus of informed consent.
The nurse researcher obtains informed consent
from participants in a study primarily to:
A) Release the researcher from legal liability.
B) Control variables that might affect the study.
C) Ensure that the study subjects understand their
roles in the study.
D) Maintain the confidentiality of the researcher
and the participants.

Answer: B

Quality client care is always the primary focus


of nursing practice. Cost control would be a
benefit but is not the primary focus. Research is
not about technology. Many "old" procedures
can be improved through research. Although
research is a professional function of nursing, it
is not done to serve the profession.
A priority goal for nursing research is:

A) Controlling cost for hospitals


B) Improving client care
C) Keeping up with technological advances
D) Maintaining the professional climate in nursing

Answer: C

Review of the literature is the first step in the


orderly research process to determine what is
already known about the problem. Recruiting
clients occurs later in the process, after
identifying the problem, researching the
literature, and designing the study.
Experimenting with new nursing procedures that
have not been tested or approved is a risk to
clients. Surveys are designed to obtain
information from large study populations and
would not be a first step in the research process.
A clinical nurse develops a better way to secure an
intravenous access device in a client and wants to
see if it would benefit other clients. The first step
in initiating a study should be to:
A) Recruit clients to participate in the study.
B) Use the new technique and gather client
feedback.
C) Review current literature related to the clinical
problem.
D) Survey clients regarding their preferences and
feelings regarding the procedure.

Answer: B

Publication of research results provides other


nurses with the scientific background of the
study before they apply its findings in practice.
Study subjects and setting should be similar to
duplicate a study. Nurses should not change
from accepted to unproven ways of providing
care without careful research and collaboration
with colleagues. Experimenting with new
nursing measures is inappropriate and may place
a client at risk.
The nurse researcher who gains new knowledge
regarding a procedure can most effectively share
the information with the nursing profession by:

A) Duplicating the study using different clients in


different settings
B) Communicating the research findings in a
professional journal
C) Recruiting clients who are willing to
demonstrate the new technique
D) Asking individual nurses to report their
experiences related to the new procedure

Answer: A

A nurse who is new to practice has not


developed the experience required for research
but can begin at the less complicated level of
data collection. An experienced researcher is
more qualified to identify problems for formal
research, although input from all levels of
nursing is valuable. Nurses with doctoral-level
training are typically prepared for obtaining
financial backing. An American Nurses
Association position paper cites a master's
degree as qualification for implementing
research-based change in nursing practice.
Nurses who are new to practice can best contribute
to nursing research by:

A) Assisting with data collection


B) Identifying clinical problems in nursing
C) Obtaining financial backing and public interest
D) Implementing research-based change in nursing
practice

Answer: C

Evaluation research is aimed at finding out how


well a program, practice, policy, or procedure is
working. A survey studies a large group to
identify general information, opinions, attitudes,
or perceptions. A grounded theory is a theory
developed through the collection and analysis of
qualitative data. Experimental research collects
information about human subjects who are
divided into a control group and a comparison
group.
A nurse manager wants to determine how well a
new policy is working in the clinical area. It would
be appropriate to use:

A) Survey methods
B) Grounded theory
C) Evaluation research
D) Experimental research

Answer: B

Critical thinking involves analyzing the data,


learning, and problem solving to come up with a
course of action. Tradition limits the ability to
learn new ways and overlooks what research has
to offer. The advice of experienced practitioners
may limit research because experience may
mean doing things the same way they have been
done for years. Using personal opinion
overlooks the objective data that are available.
The nurse involved in scientific research
effectively analyzes the information collected and
determines a course of nursing action by:

A) Depending on tradition
B) Using critical thinking
C) Seeking the advice of experienced practitioners
D) Relying on personal perspective or opinion

Answer: B

Effective qualitative research can be carried out,


because through narrative interviews the
participants' perceptions can be compared and
common characteristics can be discovered. It is
difficult to collect data about perceptions or
feelings without talking to those involved.
Quantitative research involves precise
measurements and would not be of use in this
study of perceptions. Although obtaining
suggestions for possible solutions could be
useful, it does not help to identify the problems
on this specific unit. The data must be collected
first.
A nurse manager is researching the effects of staff
shortages on job satisfaction among new graduates.
It would be most effective for the nurse to gather
data by:

A) Directly observing the nursing behaviors on the


unit
B) Interviewing staff nurses on the unit regarding
their perceptions
C) Setting up an experimental group and a control
group for the study
D) Calling on other nurses in the facility to suggest
ways of handling the problem

Answer: D

Shortage of staff could mean less time and


personnel to conduct and participate in research.
Nursing teams that have teamwork skills can aid
research. The desire to change is an incentive
for research. Pressure from higher levels in the
organization is also an incentive to research.
Which of the following could be a barrier to
nursing research?

A) Presence of teams in nursing


B) Pressure from the administration
C) Staff wishes to change a policy
D) Shortage of professional nursing staff

Answer: B

Hypertension is often asymptomatic until


pressure is very high. Headache (usually
occipital), facial flushing, nosebleed, and fatigue
are common symptoms of hypertension.
Restlessness and dusky or cyanotic skin that is
cool to the touch, dizziness, mental confusion,
and mottled extremities are all signs and
symptoms of hypotension. Unexplained pain
and hyperactivity are very vague complaints.
Besides high blood pressure values, what other
signs and symptoms may the nurse observe if
hypertension is present?

A) Unexplained pain and hyperactivity


B) Headache, flushing of the face, and nosebleed
C) Dizziness, mental confusion, and mottled
extremities
D) Restlessness and dusky or cyanotic skin that is
cool to the touch

Answer: D

An oxygen saturation of 89% should be


addressed first, because this indicates that a
client needs oxygen. The high respiratory rate
may be a result of hypoxemia and may decrease
as the oxygen saturation climbs. The blood
pressure is high, but this might be attributed to
hypoxemia or anxiety. The heart rate and
temperature are within normal limits.
Which of the following values for vital signs
would the nurse address first?

A) Heart rate = 72 beats per minute


B) Respiration rate = 28 breaths per minute
C) Blood pressure = 160/86
D) Oxygen saturation by pulse oximetry = 89%
E) Temperature = 37.2° C (99° F), tympanic

Answer: D

This client has a fever, potentially secondary to


the pneumonia previously diagnosed. His blood
pressure is within normal limits. His oxygen
saturation is at 92%, so this will need to be
addressed second. His respiratory rate is high,
which can be a result of the fever.
An 82-year-old widower brought via ambulance is
admitted to the emergency department with
complaints of shortness of breath, anorexia, and
malaise. He recently visited his health care
provider and was put on an antibiotic for
pneumonia. The client indicates that he also takes a
diuretic and a beta blocker, which helps his "high
blood." Which vital sign value would take priority
in initiating care?

A) Respiration rate = 20 breaths per minute


B) Oxygen saturation by pulse oximetry = 92%
C) Blood pressure = 138/84
D) Temperature = 39° C (102° F), tympanic

Answer: C

Since the "up ad lib" orders are new and the


client has been on bed rest, checking orthostatic
blood pressure before allowing the client to
ambulate is the correct answer. If no sign of
orthostatic hypotension is present, then a
nursing assistant could assist him to the
bathroom. Giving the client a urinal is not a
good choice if the client is asymptomatic when
orthostatic blood pressure is checked.
The client, who has been on bed rest for 2 days,
asks to get out of bed to go to the bathroom. He
has new orders for "up ad lib." What action should
the nurse take?

A) Give him some slippers and tell him where the


bathroom is located.
B) Ask the nursing assistant to assist him to the
bathroom.
C) Obtain orthostatic blood pressure
measurements.
D) Tell him it is not a good idea and provide a
urinal.

Answer: A

Although early morning temperatures are


routinely low, the best practice is for the nurse
to check the client's previous temperatures.
Clients may routinely have a low temperature.
Depending on the client's temperature history,
the nurse may retake the temperature with
another thermometer to check for a malfunction.
If everything seems satisfactory, the nurse
should chart the temperature and check the
client for signs of hypothermia.
Using an oral electronic thermometer, the nurse
checks the early morning temperature of a client.
The client's temperature is 36.1° C (97° F). The
client's remaining vital signs are in the normally
acceptable range. What should the nurse do next?

A) Check the client's temperature history.


B) Document the results; temperature is normal.
C) Recheck the temperature every 15 minutes until
it is normal.
D) Get another thermometer; the temperature is
obviously an error.

Answer: B

The apical pulse gives the nurse the most


information and accuracy when assessing
irregular cardiac rhythm. The carotid or femoral
pulses are usually used to assess a client in
shock. The radial pulse is adequate for
determining routine postoperative vital signs
and for checking changes in orthostatic heart
rate.
The nurse decides to take an apical pulse instead of
a radial pulse. Which of the following client
conditions influenced the nurse's decision?

A) The client is in shock.


B) The client has an arrhythmia.
C) The client underwent surgery 18 hours earlier.
D) The client showed a response to orthostatic
changes.

Answer: D

Postponing this assessment is definitely a


judgment call by the nurse. Postponing is
appropriate unless the assessment of respiration
is a critical aspect of the test and the client is
leaving for the test immediately. Otherwise, it is
probably not necessary to invade the client's
privacy and disrupt the visitation. Agency policy
will dictate whether the respiration rate should
be documented as "deferred" or whether
documentation can wait until the rate is
obtained. Respirations should be counted when
the client is "at rest"; therefore, counting
respirations during the visitation would not be
appropriate. Waiting at the bedside until the
visitor leaves is an invasion of privacy for the
client and a waste of the nurse's time.
The nurse is to measure vital signs as part of the
preparation for a test. The client is talking with a
visiting pastor. How should the nurse handle
measuring the rate of respiration?

A) Count respirations during the time the client is


not talking to the visitor.
B) Wait at the client's bedside until the visit is over
and then count respirations.
C) Tell the client it is very important to end the
conversation so the nurse can count respirations.
D) Document the respiration rate as "deferred" and
measure the rate later, since the talking client is
obviously not in respiratory distress.

Answer: D

The nurse may delegate vital signs measurement


to unlicensed assistive personnel when the client
is in stable condition, the results are predictable,
and the technique is standard. The preoperative
client is the only client listed who meets these
guidelines.
Delegation of some tasks may become one of the
decisions the nurse will make while on duty. For
which of the following clients would it be most
appropriate for unlicensed assistive personnel to
measure the client's vital signs?

A) A client who recently started taking an


antiarrhythmic medication
B) A client with a history of transfusion reactions
who is receiving a blood transfusion
C) A client who has frequently been admitted to
the unit with asthma attacks
D) A client who is being admitted for elective
surgery who has a history of stable hypertension

Answer: D

Therapies such as tepid water or alcohol sponge


baths should be avoided because they lead to
shivering, which stimulates body heat.
Antipyretics, not analgesics, are the medications
that lower body temperature.
The client has an oral temperature of 39.2° C
(102.6° F). What are the most appropriate nursing
interventions?

A) Provide an alcohol sponge bath and monitor


laboratory results.
B) Remove excess clothing, provide a tepid sponge
bath, and administer an analgesic.
C) Provide fluids and nutrition, keep the client's
room warm, and administer an analgesic.
D) Reduce external coverings and keep clothing
and bed linens dry; administer antipyretics as
ordered.

Answer: D

The anterior hypothalamus controls heat loss by


initiating the mechanisms of sweating and
vasodilation of blood vessels. Blood is
redistributed to surface vessels (flushing of the
skin) to promote heat loss, not heat retention.
The posterior hypothalamus controls heat
production by initiating the mechanisms of
shivering, vasoconstriction of blood vessels, and
reduction of blood flow to the skin and
extremities.
The hypothalamus controls body temperature. The
anterior hypothalamus controls heat loss, and the
posterior hypothalamus controls heat production.
What heat conservation mechanisms will the
posterior hypothalamus initiate when it senses that
the client's body temperature is lower than
comfortable?

A) Vasodilation and redistribution of blood to


surface vessels
B) Sweating, vasodilation, and redistribution of
blood to surface vessels
C) Vasoconstriction, sweating, and reduction of
blood flow to extremities
D) Vasoconstriction, reduction of blood flow to
extremities, and shivering

Answer: C

To measure pulse deficit the nurse and a


colleague assess the radial and apical pulse rates
simultaneously and subtract the radial from the
apical pulse rate. The result is the pulse deficit.
Tachycardia and bradycardia are assessed by
measuring the pulse rate for 1 minute. A rate of
more than 100 beats per minute is categorized as
tachycardia, whereas a rate of less than 60 beats
per minute constitutes bradycardia.
The nurse's documentation indicates that a client
has a pulse deficit of 14 beats. The pulse deficit is
measured by:

A) Subtracting 60 (bradycardia) from the client's


pulse rate and reporting the difference
B) Subtracting the client's pulse rate from 100
(tachycardia) and reporting the difference
C) Assessing the apical pulse and the radial pulse
for the same minute and subtracting the difference
D) Assessing the apical pulse and 30 minutes later
assessing the carotid pulse and subtracting the
difference

Answer: D

Cheyne-Stokes respiration is an irregular


respiratory rate and depth with alternating
periods of apnea and hyperventilation; it begins
with slow breaths and climaxes in apnea before
respiration resumes.
The nurse observes that a client's breathing pattern
represents Cheyne-Stokes respiration. Which
statement best describes the Cheyne-Stokes
pattern?

A) Respirations cease for several seconds.


B) Respirations are abnormally shallow for two to
three breaths followed by irregular periods of
apnea.
C) Respirations are labored, with an increase in
depth and rate (more than 20 breaths per minute);
the condition occurs normally during exercise.
D) Respiration rate and depth are irregular, with
alternating periods of apnea and hyperventilation;
the cycle begins with slow breaths and climaxes in
apnea.

Answer: D

All questionable blood pressure readings should


be rechecked. Ensuring the client's safety is a
necessary safeguard, because low blood
pressure is generally accompanied by weakness.
For the majority of people, low blood pressure
(systolic pressure of 90 mm Hg or below) is an
abnormal finding and should be reported.
Giving a client orange juice may raise blood
glucose level but is not recommended to elevate
blood pressure. Ambulating a client with
hypotension would not be following safety
precautions.
The nurse finds that the systolic blood pressure of
an adult client is 88 mm Hg. What are the
appropriate nursing interventions?

A) Check other vital signs.


B) Recheck the blood pressure and give the client
orange juice.
C) Recheck the blood pressure after ambulating the
client safely.
D) Recheck the blood pressure, make sure the
client is safe, and report the findings.

Answer: A

The general survey focuses on general


appearance and behavior, including gender and
race, age, signs of distress, body type, posture,
gait, hygiene and grooming, dress, affect, mood,
and speech. The other actions are carried out in
different parts of the assessment.
The nurse conducts a general survey of an adult
client, which includes:

A) Checking appearance and behavior


B) Measuring vital signs
C) Observing specific body systems
D) Conducting a detailed health history

Answer: C

The nurse accurately assesses temperature by


palpating the skin with the dorsum or back of
the hand, because this area of the hand is more
sensitive to temperature than is the base of the
hands, the fingertips, or the palmar surface.
To correctly palpate the client's skin for
temperature, the nurse uses which of the
following?

A) Base of the hands


B) Fingertips of the hands
C) Dorsal surface of the hands
D) Palmar surface of the hands

Answer: D

Superficial lymph nodes are gently palpated


using the pads of the index and middle fingers.
The lymph nodes are small, and any other
method would not be helpful.
To assess a client's superficial lymph nodes, the
nurse:

A) Deeply palpates using the entire hand


B) Deeply palpates using a bimanual technique
C) Lightly palpates using a bimanual technique
D) Gently palpates using the pads of the index and
middle fingers

Answer: C

Pigmented skin lesions that are asymmetrical,


have irregular borders, have variegated colors,
and are larger than 6 mm in diameter are lesions
that are suspect and should be reported to a
medical provider.
The nurse teaches the client to inspect all skin
surfaces and to report pigmented skin lesions that:

A) Are symmetrical
B) Are uniform in color
C) Have irregular borders
D) Are smaller than 6 mm in diameter

Answer: A

The sounds of lung fields on one side of the


body are compared with the sounds of the same
fields on the opposite side of the body. The
other answers would provide incorrect
comparisons.
To auscultate the client's lung fields, the nurse uses
a systematic pattern comparing:

A) Side to side
B) Top to bottom
C) Anterior to posterior
D) Interspace to interspace

Answer: B

Rhonchi are loud, low-pitched, rumbling, coarse


sounds heard most often during inspiration or
expiration. Crackles are moist sounds heard
during inspiration that are not cleared with
coughing. Normal lungs produce no sounds such
as that described. Wheezes are high-pitched
continuous muscles sounds such as a squeak
heard continuously during inspiration and
expiration.
The client's respiratory assessment reveals loud,
low-pitched, rumbling, coarse sounds heard during
inspiration and expiration. The nurse interprets
these sounds as:

A) Normal
B) Rhonchi
C) Crackles
D) Wheezes

Answer: C

The S2 (dub) sound is the second heart sound


and indicates closure of the aortic and pulmonic
valves. The closing of the mitral and tricuspid
valves is the S1 sound.
While auscultating heart sounds, the nurse
documents that S2 is best heard at the base. This
sound (S2) correlates with closure of which of the
following?

A) Aortic and mitral valves


B) Mitral and tricuspid valves
C) Aortic and pulmonic valves
D) Tricuspid and pulmonic valves

Answer: D

The dorsalis pedis pulse is felt on the top of the


foot in line with the groove between the
extensor tendons of the great toe and the first
toe. The popliteal pulse is felt behind the knee.
The posterior tibial pulse is felt in the groove
behind the medial malleolus. Typically pulses
are not palpated behind the lateral malleolus.
To assess the client's dorsalis pedis pulse, the nurse
palpates:

A) Behind the knee


B) Over the lateral malleolus
C) In the groove behind the medial malleolus
D) Lateral to the extensor tendon of the great toe

Answer: A

Lying in the supine position with the ipsilateral


arm behind the head helps the breast tissue to
flatten evenly against the chest wall. The other
options do not allow the tissue to spread on the
chest wall.
So that breast tissue will be spread evenly over the
chest wall during an examination, the nurse asks
the client to lie supine with:

A) The ipsilateral arm behind the head


B) Hands clasped just above the umbilicus
C) Both arms overhead with palms upward
D) The dominant arm straight alongside the body

Answer: B

Painless, pea-sized nodules should be checked


by a health care provider. Testicular self-
examination should be performed monthly and
should be done after a bath or shower. The
testes feel smooth, rubbery, and free of nodules.
The nurse is teaching a client how to perform a
testicular self-examination. The nurse tells the
client which of the following?
A) "The testes are normally round, moveable, and
have a lumpy consistency."
B) "Contact your health care provider if you feel a
painless pea-sized nodule."
C) "The best time to do a testicular self-
examination is before your bath or shower."
D) "Perform a testicular self-examination weekly
to detect signs of testicular cancer."

Answer: D

Adduction is movement toward the body.


Abduction is movement away from the body.
Flexion is movement that decreases the angle of
the joint, whereas extension is movement that
increase the angle of the joint.
The client is being assessed for range of joint
movement. The nurse asks the client to move the
arm toward the body to evaluate:

A) Flexion
B) Extension
C) Abduction
D) Adduction

Answer: D

This common adage embodies an abstract idea,


and explaining it indicates the client's ability to
perform abstract reasoning. Judgment involves
comparison and evaluation of facts and ideas to
understand their relationships and to form
appropriate conclusions. Knowledge is
understanding or awareness of information
gained through learning or experience.
Association involves finding similarities
between concepts.
The nurse asks the client to interpret the saying,
"Don't count your chickens before they're
hatched." The client's response provides
information about the client's:
A) Judgment
B) Knowledge
C) Association
D) Abstract reasoning

Answer: D

The movement of the head and shoulders is


controlled by cranial nerve XI, the spinal
accessory nerve. The facial nerve innervates the
face. The hypoglossal nerve innervates portions
of the tongue. The trigeminal nerve is a sensory
and motor nerve enervating the side of the face
and jaw.
The nurse asks the client to shrug the shoulders
and turn the head side to side against the resistance
of the examiner's hand. These actions allow the
nurse to evaluate which cranial nerve?

A) VII—Facial
B) V—Trigeminal
C) XII—Hypoglossal
D) XI—Spinal accessory

Answer: D

The nurse begins with inspection and then


follows with auscultation. It is important to
perform auscultation before palpation and
percussion, because palpation and percussion
may alter the frequency and character of bowel
sounds.
The techniques of physical assessment are
inspection, palpation, percussion, and auscultation.
The order in which these techniques are used is
slightly different during abdominal examination
than during examination of other body areas. The
nurse should perform which two of the following
first?
A) Palpation and inspection
B) Inspection and percussion
C) Palpation and auscultation
D) Inspection and auscultation

Answer: B

Sitting upright provides full expansion of the


lungs and provides better visualization of the
symmetry of upper body parts. The lateral
recumbent position aids in detecting heart
murmurs. The dorsal recumbent position is used
for abdominal assessment because it allows
relaxation of abdominal muscles. The supine
position provides easy access to pulse sites.
The nurse should assist the client to a sitting
position to provide the best position to examine
which of the following?

A) Heart
B) Lungs
C) Abdomen
D) Pulse sites

Answer: D

To assess skin turgor, a fold of skin on the back


of the forearm or sternal area is grasped with the
fingertips and released. When turgor is good the
skin lifts easily and snaps back immediately.
The skin stays pinched when turgor is poor. The
hands and neck are not the best places to test
turgor, because the skin is normally loose and
thin in those areas.
Turgor is the skin's elasticity, which can be
diminished by edema or dehydration. Which is the
best place for the nurse to assess skin turgor in the
older adult?

A) Side of the neck


B) Back of the hand
C) Palm of the hand
D) Over the sternal area

Answer: C

Resonance is the low, hollow sound of normal


lungs. Hyperresonance can be heard over
emphysematous lungs as a booming sound.
Tympany is the high-pitched, drumlike sound
heard over a gastric air bubble. Dullness is the
soft, thudlike sound that is heard over dense
organ tissue.
In assessing the client's lungs, the nurse notes that
the lungs are normal upon percussion. This means
that the nurse detected:

A) Dullness
B) Tympany
C) Resonance
D) Hyperresonance

Answer: C

Pallor would appear as yellowish brown in


brown-skinned people. Pallor would manifest as
bluish skin in light-skinned people. Pallor would
appear as ashen gray skin in black-skinned
people. Shiny skin indicates edema.
A common abnormality encountered during
inspection of the skin is pallor. Pallor is easily seen
in the face, mucosa of the mouth, and nail beds.
How would pallor appear in a brown-skinned
client?

A) As shiny skin
B) As bluish skin
C) As yellowish skin
D) As ashen gray skin

Answer: D
A normal tympanic membrane is translucent,
shiny, and pearly gray. Dark yellow and sticky
describes normal moist cerumen (earwax) in
front of the tympanic membrane. A white color
indicates pus behind the membrane. A pink or
red bulging membrane is an indication of
inflammation.
Using an otoscope, the nurse can inspect the
tympanic membrane. A normal tympanic
membrane appears:

A) Round and white


B) Pink and bulging
C) Dark yellow and sticky
D) Translucent, shiny, and pearly gray

Answer: C

Wheezes are adventitious breath sounds that are


high-pitched, continuous musical sounds, such
as a squeak heard continuously during
inspiration or expiration. Pleural friction rub has
a dry, grating quality and is heard best during
inspiration. Crackles can be fine, high-pitched,
short, interrupted crackling sounds; moist, low
sounds in the middle of inspiration; or coarse,
loud, bubbly sounds. Rhonchi are loud, low-
pitched, rumbling, coarse sounds heard during
inspiration.
In assessing the client's lungs the nurse hears
adventitious breath sounds that are high-pitched,
continuous musical sounds, such as a squeak heard
continuously during inspiration or expiration,
usually louder on expiration. These adventitious
breath sounds are known as:

A) Crackles
B) Rhonchi
C) Wheezes
D) Pleural friction rub

Answer: D

Aortic, pulmonic, tricuspid, mitral areas are the


sites for auscultation assessment of cardiac
function. Auscultation of the carotid arteries is
not the same as auscultation of the heart itself.
The sternal region is not an appropriate site.
There are no costal sites where heart sounds can
be heard well. Anterior, posterior, and lateral are
too vague.
The nurse should use which anatomical sites for
the auscultatory assessment of cardiac function?

A) Inner costal, outer costal, and sternal


B) Aortic, carotid, coronary, and jugular
C) Apical, lateral, anterior, and posterior
D) Aortic, pulmonic, tricuspid, and mitral

Answer: C

Complaints of tenderness in the calf during


palpation may indicate phlebitis. Other
characteristics of phlebitis are swelling, warmth,
redness, and sometimes a positive Homans' sign.
Cyanosis, pallor, and brown pigmentation
around the ankles as well as ulceration and
reduced hair growth are indications of venous or
arterial insufficiency, which would not cause
tenderness on palpation. Venous distention may
be indicative of varicosities, which also are not
associated with tenderness.
While the nurse was palpating the calf muscles of
the client's right leg, the client complained of
tenderness. Further assessment by the nurse should
include which of the following?

A) Observation for reduced hair growth and


ulceration
B) Observation for venous distention while the
client is standing
C) Observation of the area for swelling, warmth,
redness, and a positive Homans' sign
D) Observation for cyanosis, pallor, and change in
pigmentation around the ankles

Answer: D

Kyphosis (hunchback) is an exaggeration of the


posterior curvature of the thoracic spine and is
common in older adults. Lordosis (swayback) is
increased lumbar curvature. Scoliosis is lateral
spinal curvature. Hypotonic muscle has little
tone and feels flabby, usually because of
atrophy of muscle mass.
During general inspection of the musculoskeletal
system of an older client, the nurse notes kyphosis.
Kyphosis is:

A) Lateral spinal curvature


B) Loss of or decrease in muscle tone
C) Increased lumbar curvature
D) Exaggeration of the posterior curvature of the
thoracic spin

Maslow's Hierarchy of basic human needs is


useful when planning and implementing nursing
care as it provides a structure for :
a. Making accurate nursing diagnoses
b. Establishing priorities of care
c. Communicating concerns more concisely
d. Integrating science into nursing care
b. Establishing priorities of care

Which of the following levels of basic human
needs is most basic?
a. Physiologic
b. Safety & Security
c. Love & belonging
d. Self-actualization
a. Physiologic

Of all the physiologic needs, which one is the
most essential?
a. Food
b. Water
c. Elimination
d. Oxygen
d. Oxygen
(Oxygen is the most essential of all needs because
all body cells require oxygen for survival)

Careful hand-washing and using sterile
techniques are ways in which nurses meet which
basic human need?
a. Physiologic
b. Safety & Security
c. Self-esteem
d. Love & belonging
b. Safety & Security
(By carrying out careful hand-washing and using
sterile technique, nurses provide safety from
infection)

Of the following statements, which one is true
of self-actualization?
a. Humans are born with fully developed self-
actualization
b. Self-actualization needs are met by having
confidence and independence
c. The self-actualization process continues
throughout life
d. Loneliness and isolation occur when self-
actualization needs are unmet
c. The self-actualization process continues
throughout life.
(Self-actualization, or reaching one's full potential,
is a process that continues through life.)

What is the best broad definition of a family?
a. A father, a mother, and children
b. Members are biologically related
c. Includes aunts, uncles, and cousins
d. A group of people who live together
d. a group of people who live together
(Although all the responses may be true, the best
definition is a group of people who live together.)

Where do individuals learn their health beliefs
and values?
a. In the family
b. In school
c. From school nurses
d. from peers
a. In the family
(Healthcare activities, heal beliefs, and health
values are learned within one's family)

John and Mary, each parents of one child, are
both divorced. When they marry, the family
structure that is formed will be:
a. Nuclear family
b. Extended family
c. Blended family
d. Cohabiting family
c. Blended family

One of the developmental tasks of the older
adult family is to:
a. Maintain a supportive home base
b. Prepare for retirement
c. Cope with loss of energy and privacy
d. Adjust to loss of spouse
d. Adjust to loss of spouse
(A developmental task of the older adult family is
adjusting to the loss of a spouse)

One element of a healthy community is that it:
a. Meets all the needs of its inhabitants
b. Offers access to healthcare services
c. Has mixed residential and industrial areas
d. Is little concerned with air and water quality
b. Offers access to healthcare services

Of the following statements, which is most true
of health and illness?
a. Health and illness are the same for all people
b. Health and illness are individually defined by
each person
c. People with acute illness are actually healthy
d. People with chronic illnesses have poor
health beliefs
b. Health and illness are individually defined by
each person

Of the following terms, which would be defined
as a disease?
a. Excess fluid volume
b. Risk for infection
c. Rheumatoid arthritis
d. Altered body image
c. Rheumatoid arthritis

Your neighbor, Alan, asks you to come over
because he has a high temperature, feels
"awful," and did not go to work. What stage of
illness behavior is Alan exhibiting?
a. Experiencing symptoms
b. Assuming the sick role
c. Assuming the dependent role
d. Achieving recovery and rehabilitation
b. Assuming the sick role
(When people assume the sick role, they define
themselves as ill, seek validation of this experience
from others, and give up normal activities)

Of the following characteristics, which one is
not a part of chronic illness?
a. Permanent change in body structure or
function
b. Self-treatment that relieves symptoms
c. Long period of treatment and care
d. Often has remission and exacerbation
b. Self-treatment that relieves symptoms

The agent-host-environment model of health
and illness is based on the concept of:
a. risk factors
b. infectious diseases
c. behaviors to promote health
d. stages of illness
a. risk factors
(The interaction of the agent-host-environment
creates risk factors that increase the probability of
disease)

What do both the health-illness continuum and
the high-level wellness models demonstrate?
a. Illness as a fixed point in time
b. The importance of family
c. Wellness as a passive state
d. Health as a constantly changing state
d. Health as a constantly changing state
(both these models view health as a dynamic,
constantly changing state)

Following the birth of his first child and after
reading about the long-term effects of nicotine,
John decides to stop smoking. This behavior
change is most likely based on John's
perceptions of all but one of the following.
Which one is not true?
a. His susceptibility to lung cancer
b. How serious lung cancer would be
c. What benefits his stopping smoking will have
d. Personal choice and economic factors
d. Personal choice and economic factors
(the others are components of the health-belief
model)

Of the following clinic patients, which one is
most likely to have annual breast examinations
and mammograms based on the physical human
dimension?
a. Jane, because her best friend had a benign
breast lump removed.
b. Sarah, who lives in a low-income
neighborhood.
c. Tricia, who has a family history of breast
cancer.
d. Nancy, because her family encourages regular
physical examinations
c. Tricia, who has a family history of breast cancer.

You are asked to teach a group of preschool
parents about poison control in the home. This
activity is an example of what level of
preventive care?
a. Lowest
b. Tertiary
c. Primary
d. Secondary
c. Primary
(teaching poison control in the home is an example
of primary preventive care)

As a nurse, you follow the guidelines for a
healthy lifestyle. How can this promote health in
others?
a. By being a role model for healthy behaviors.
b. By not requiring sick days from work.
c. By never exposing others to any type of
illness
d. By not being overweight
a. By being a role model for healthy behaviors.
(Good personal health enables the nurse to serve as
a role model for patients and families.)

Which of the following statements about the
nursing process is most accurate?
a. The nursing process is a four-step procedure
for identifying and resolving patient problems.
b. Beginning in Florence Nightingale's days,
nursing students learned and practiced the
nursing process
c. Use of the nursing process is optional for
nurses, since there are many ways to accomplish
the work of nursing.
d. The state board examinations for professional
nursing practice now use the nursing process
rather than medical specialties as an organizing
concept
d. The state board examinations for professional
nursing practice now use the nursing process rather
than medical specialties as an organizing concept
(a.The nursing process is a five-step process, b.
The term nursing process was first used by Hall in
1955, c. Standards demand the use of the nursing
process, so it is not optional.)

The nursing process ensures that nurses are
patient centered rather than task centered.
Rather than simply approaching a patient to take
vital signs, the nurse thinks "How is Mrs.
Barclay today? Are our nursing actions helping
her to achieve her goals? How can we better
help her?" This demonstrates which
characteristics of the nursing process?
a. Systematic
b. Interpersonal
c. Dynamic
d. Universally applicable in nursing situatins
b. Interpersonal
(Each of the other options are characteristics of the
nursing process, but the conversation and thinking
quoted best illustrates the interpersonal dimension
of the nursing process)

An experienced nurse tells you not to bother
studying too hard, since most clinical reasoning
becomes "second nature" and "intuitive" once
you start practicing. What thinking below
should underline your response?
a. When intuition is used alone, there are
increased risks and fewer benefits. Intuition
often moves problem-solving forward quickly,
but it might result in a lot of trial-and-error
approaches.
b. For nursing to remain a science, nurses must
continue to be vigilant about stamping out
intuitive reasoning.
c. The emphasis on logical, scientific, evidence-
based reasoning has held nursing back for years.
It's time to champion intuitive, creative
thinking!
d. It's simply a matter of preference. Some of us
are logical, scientific thinkers, and some are
intuitive, creative thinkers
a. When intuition is used alone, there are increased
risks and fewer benefits. Intuition often moves
problem-solving forward quickly, but it might
result in a lot of trial-and-error approaches.

This text is based upon a notion of blended
skills. .Simply described, this means:
a. Nursing works best when nurses competently
use the intellectual and technical skills that
achieve patient outcomes. Nursing has been held
back by outdated notions of care and
compassion (interpersonal skills), which can be
done by anyone.
b. Nursing works best when each nurse
competently uses the intellectual, interpersonal,
technical, and ethical/legal skills demanded by
each situation.
c. All of the blended skills are important, but not
every nurse has to be skilled in each area.
d. Every nursing situation demands the same
blend of basic nursing skills, intellectual,
technical, interpersonal, and ethical/legal skills.
b. Nursing works best when each nurse
competently uses the intellectual, interpersonal,
technical, and ethical/legal skills demanded by
each situation.

The best description of critical thinking
indicators (CTIs) is which of the following:
a. Evidence-based descriptions of behaviors that
demonstrate the knowledge that promotes
critical thinking in clinical practice.
b. Evidence-based descriptions of behaviors that
demonstrate the knowledge and skills that
promote critical thinking in clinical practice.
c. Evidence-based descriptions of behaviors that
demonstrate the knowledge, characteristics and
skills that promote critical thinking in clinical
practice.
d. Evidence-based descriptions of behaviors that
demonstrate the knowledge, characteristics,
standards, and skills that promote critical
thinking in clinical practice.
c. Evidence-based descriptions of behaviors that
demonstrate the knowledge, characteristics and
skills that promote critical thinking in clinical
practice.

Although the nursing process is presented as an
orderly progression of steps, in reality there is
great interaction and overlapping among the five
steps. This characteristic of the nursing process
is described as:
a. Systematic
b. Dynamic
c. Interpersonal
d. Outcome oriented
b. Dynamic

While administering a medication to relieve a
patient's pain, you wonder if there are some
nonpharmacologic interventions that would
enhance relief by complementing the pain
medication. When you discuss this with your
instructor you are most likely to hear:
a. "You should wait until after you evaluate the
effect of the medication you just administered
before planning a different intervention"
b. "One step at a time, dear. Don't start planning
a new intervention until you evaluate the old."
c. "Lets talk about this... we often get new
information that we can incorporate successfully
into the plan of care. Sometimes the steps of the
process interact or overlap."
d. "Think about this patient. Nonpharmacologic
interventions wouldn't be effective with her."
c. "Lets talk about this... we often get new
information that we can incorporate successfully
into the plan of care. Sometimes the steps of the
process interact or overlap."

When a patient you are admitting to the unit
asks you why you are doing a history and exam
since the doctor just did one, your best reply is:
a. "In addition to providing us with valuable
information about your health status, the nursing
assessment will allow us to plan and deliver
individualized, holistic nursing care that draws
on your strengths."
b. "It's hospital policy. I know it must be
tiresome, but I will try to make this quick."
c. "I'm a student nurse and need to develop the
skill of assessing your health status and need for
nursing care. This information will help me
develop a plan of care individualized to your
unique needs."
d. "We want to make sure that your responses
are consistent and that all our data are accurate."
a. "In addition to providing us with valuable
information about your health status, the nursing
assessment will allow us to plan and deliver
individualized, holistic nursing care that draws on
your strengths."

When you receive shift report, you learn that
your patient has no special skin care needs. You
are surprised during the bath to observe
reddened areas over body prominences. You
should:
a. Correct the initial assessment form.
b. Redo the initial assessment and document
current findings.
c. Conduct and document an emergency
assessment.
d. Perform and document a focused assessment
on skin integrity
d. Perform and document a focused assessment on
skin integrity

Fearful of attempting your first nursing history,
you ask your instructor how anyone ever learns
everything you have to ask to get good baseline
data. You are most likely to hear:
a. "There's a lot to learn at first, but once it
becomes part of you, you just keep asking the
same questions over and over in each situation
until you can do it in your sleep!"
b. " You make the basic questions a part of you
and then learn to modify them for each unique
situation, asking yourself how much you need to
know to plan good care."
c. "No one ever really learns how to do this well
because each history is different."
d. "Don't worry about learning all of the
questions to ask. Every agency has its own
assessment for you must use."
b. " You make the basic questions a part of you
and then learn to modify them for each unique
situation, asking yourself how much you need to
know to plan good care."

A patient complains about feeling nauseated
after lunch. This is an example of what type of
data?
a. Subjective
b. Objective
c. Signs and symptoms
d. Overt
a. Subjective

When you enter the patient's room to begin your
nursing history, the patient's wife is there. You
should:
a. Introduce yourself to both and thank the wife
for being present.
b. Introduce yourself to both and ask the wife if
she wants to remain.
c. Introduce yourself and ask the wife to leave.
d. Introduce yourself and ask the patient if the
would like the wife to stay.
d. introduce yourself and ask the patient if he
would like the wife to stay.

The patient is Vietnamese and does not speak
English. Her son is with her and does speak
English. How should you respond?
a. Ask the son if he is willing to translate and be
sure to thank him if he says yes.
b. Determine if the son can translate medical
information, and if so, begin.
c. After determining that the son can translate,
evaluate if he can do so objectively and if the
patient wants him to serve in this capacity.
d. Explain to the son that hospital policy forbids
using family members as translators and find a
hospital approved translator.
c. After determining that the son can translate,
evaluate if he can do so objectively and if the
patient wants him to serve in this capacity.

You are surprised to detect and elevated
temperature (102 F) in a patient scheduled for
surgery. The patient has been afebrile and shows
no other signs of being febrile. The first thing
you do is to:
a. Inform the charge nurse
b. Inform the surgeon
c. Validate your finding
d. Document your finding.
c. Validate your finding

You tell your instructor that your patient is fine
and has "no complaints." You are likely to hear:
a. You made an inference that she is fine
because she has no complaints.How did you
validate this?
b. She probably just doesn't trust you enough to
share what she is feeling. I'd work on
developing a trusting relationship.
c. Sometimes everyone gets lucky. Why don't
you try to help another patient?
d. Maybe you should reassess the patient. He
has to have a problem. Why else would he be
here?
a. You made an inference that she is fine because
she has no complaints.How did you validate this?

Identify all of the following that are purposes of
diagnosing. The purpose of diagnosing is to
identify:
1. How an individual, group, or community
responds to actual or potential health and life
processes.
2. Factors that contribute to or cause health
problems (etiologies)
3. Strengths the patient can draw on to prevent
or resolve problems.
4. Nursing interventions to resolve health
problems
a. 1 & 2
b. 3 & 4
c. 1, 2, & 3
d. All of the above
c. 1, 2, & 3

The terms diagnose and diagnosis have legal
implications. they imply that there is a specific
problem that requires management by a
qualified expert. Which of the following
statements is false?
a. If you make a diagnosis, it means that you
accept accountability for accurately naming and
managing the problem
b. If you treat a problem or allow a problem to
persist without ensuring that the correct
diagnosis has been made, you may cause harm
and be accused of negligence.
c. You are accountable for detecting,
identifying, or recognizing signs and symptoms
that may indicate problems beyond your
expertise.
d. When nurses diagnose a medical problem,
they are just as accountable as physicians for
detecting, identifying, and managing the signs
and symptoms of disease.
d. When nurses diagnose a medical problem, they
are just as accountable as physicians for detecting,
identifying, and managing the signs and symptoms
of disease.

Which was the first state to identify diagnosing
as part of the legal domain of professional
nursing?
a. New Jersey
b. New York
c. North Carolina
d. North Dakota
b. New York

Which group is responsible for the promotion
and organization of activities to continue the
development, classification, and scientific
testing of nursing diagnoses?
a. American Nurses Association
b. National Nursing Diagnosis Association
c. North American Nursing Diagnosis
Association
d. Clearinghouse for Nursing Diagnoses
c. North American Nursing Diagnosis Association

Altered Health Maintenance is an example of:
a. Collaborative problem
b. Interdisciplinary problem
c. Medical problem
d. Nursing problem
d. Nursing problem
(because it describes a problem that can be treated
by nurses within their scope of independent
nursing practice)

To determine the significance of a blood-
pressure reading of 148/100, it is first necessary
to:
a. Compare this reading to standards
b. Check the taxonomy of nursing diagnoses for
a pertinent label.
c. Check a medical text for the signs and
symptoms of high blood pressure
d. Consult with collleagues.
a. Compare this reading to standards

A clinical judgment that an individual, family,
or community is more vulnerable to develop the
problem than others in the same or similar
situation is what type of nursing diagnosis?
a. Actual
b. Risk
c. Possible
d. Wellness
e. Syndrome
b. Risk

During the outcome identification and planning
step of the nursing process, the nurse works in
partnership with the patient and family to do
which of the following?
1) Formulate and validate prioritized nursing
diagnoses
2) Identify expected patient outcomes
3) Select evidence-based nursing interventions
4) Communicate the plan of nursing care
2, 3 & 4

Mr Price tells the nurse he fears becoming
"hooked on drugs" and consequently waits until
his pain becomes unbearable before requesting
his prn analgesic. The nurse plans to be more
attentive to Mr Price and to assess his needs for
pain management more closely. Which of the
following consequences of informal planning
ought to be the major concern for this nurse?
a. The lack of a coordinated plan known by
everyone will result in uneven pain
management.
b. Faulty prioritization of patient needs
c. Inability to evaluate the patient's responses to
nursing care
d. Lack of a record for reimbursement purposes
a. The lack of a coordinated plan known by
everyone will result in uneven pain management.

When helping Mr Price turn in bed, the nurse
notices that his heels are reddened and plans to
place him on precautions for skin breakdown.
This is an example of:
a. Initial Planning
b. Standardized planning
c. Ongoing planning
d. Discharge planning
c. Ongoing planning

Use Maslow's hierarchy of human needs to
prioritize the following patient problems from
highest priority (#1) to lowest priority (#4):
1) Disturbed Body Image
2) Ineffective Airway Clearance
3) Spiritual Distress
4) Impaired Social Interaction
2, 4, 1, 3

From which of the following are outcomes
derived?
a. The problem statement of the nursing
diagnosis
b. The etiology of the problem of the nursing
diagnosis
c. The defining characteristics of the problem
d. The evaluative statement
a. The problem statement of the nursing diagnosis
(Outcomes are derived from the problem statement
of the nursing diagnosis. For each nursing
diagnosis in the plan of care, at least one outcome
should be written that, if achieved, demonstrates a
direct resolution of the problem statement)

Which one of the following is an example of an
affective outcome?
a. within 1 day after teaching, the patient will
list three benefits of continuing to apply moist
compresses to leg ulcer after discharge.
b. By 6/12/12, the patient will correctly
demonstrate application of wet-to-dry dressing
on leg ulcer.
c. By 6/19/12, the patient's ulcer will begin to
show signs of healing (eg, size shrinks form 3"
to 2.5").
d. By 6/12/12, the patient will verbalize valuing
health sufficiently to practice new health
behaviors to prevent recurrence of leg ulcer.
d. By 6/12/12, the patient will verbalize valuing
health sufficiently to practice new health behaviors
to prevent recurrence of leg ulcer.
(Affective outcomes describe changes in patient
values, beliefs, and attitudes. Cognitive outcomes
describe increases in patient knowledge of
intellectual behaviors; psychomotor outcomes
describe the patient's achievement of new skills. )

Which of the following is an optional element in
a measurable outcome?
a. Subject
b. Verb
c. Performance criteria
d. Conditions
e. target time
d. Conditions
(Conditions specify the particular circumstances in
or by which the outcome is to be achieved. Not
every outcome specifies conditions.)

Which of he following outcomes are correctly
written?
1. Offer Mr Myer 60 mL fluid every 2 hours
while awake.
2. During the next 24-hr period, the patient's
fluid intake will total at least 2,000 mL.
3. By discharge Mrs Gaston will know how to
bathe her newborn.
4. At the next visit, 12/23/12, the patient will
correctly demonstrate relaxation exercises.
2&4

Which of the following guidelines for outcome
writing are correct?
1. At least one of the outcomes shows a direct
resolution of the problem statement in the
nursing diagnosis.
2. The patient (and family) values the outcomes.
3. The outcomes are supportive of the total
treatment plan.
4. Each outcome is brief and specific (clearly
describes one observable, measurable patient
behavior/manifestation), is phrased positively,
and specifies a time line.
All of the above

Which of the following are examples of well-
stated nursing interventions?
1. Offer patient 60 mL water or juice (prefers
orange or cranberry juice) every 2 hours while
awake for a total minimum PO intake of 500
mL.
2. Teach patient the necessity of carefully
monitoring fluid intake and output; remind
patient each shift to mark off fluid intake on
record at bedside.
3. Walk with patient to bathroom for toileting
every 2 hours (on even hours) while patient is
awake.
4. Manage patient's pain.
1, 2, 3

A school nurse notices that Jill is losing weight
and wants to perform a focused assessment on
Jill's nutritional status, fearing that she might
have an eating disorder. How should the nurse
proceed?
a. Perform the focused assessment. This is an
independent nurse-initiated intervention.
b. Request an order from Jill's physician since
this is a physician-initiated intervention.
c. Request an order from Jill's physician since
this is a collaborative intervention
d. Request an order from the nutritionist since
this is a collaborative intervention
a. Perform the focused assessment. This is an
independent nurse-initiated intervention

Which of the following would you expect to
find in the Nursing Interventions Classification
Taxonomy?
a. Case studies illustrating a complete set of
activities that a nurse performs to carry out
nursing interventions.
b. Nursing interventions, each with a label, a
definition, and a set of activities that a nurse
performs to carry it out, with a short list of
background readings.
c. A complete list of nursing diagnoses,
outcomes, and related nursing activities for each
nursing intervention.
d. A complete list of reimbursable charges for
each nursing intervention.
b. Nursing interventions, each with a label, a
definition, and a set of activities that a nurse
performs to carry it out, with a short list of
background readings

You are a brand new RN. When you orient to a
new nursing unit that is currently understaffed,
you are told that the UAPs have been trained to
obtain the initial nursing assessment. What is
the best response?
a. Allow the UAPs to do the admission
assessment and report the findings to you.
b. Do your own admission assessments but don't
interfere with the practice if other professional
RNs seem comfortable with the practice.
c. Tell the charge nurse that you are choosing
not to delegate the admission assessment at this
time until you can get further clarification from
administration.
d. Contact your labor representative and
complain.
c. Tell the charge nurse that you are choosing not
to delegate the admission assessment at this time
until you can get further clarification from
administration.

Jeanne is a college student who wants to lost 20
pounds. She meets with the student health nurse
and develops a plan to increase her activity level
and decrease the consumption of the wrong
types of foods and excess calories. The nurse
plans to evaluate her weight loss monthly. When
Jeanne arrives for her first "weigh-in", the nurse
discovers that instead of the projected weight
loss of 5 pounds, Jeanne has only lost 1 pound.
Which is the best nursing response?
a. Congratulate Jeanne and continue the plan of
care.
b. Terminate the plan of care since it is not
working.
c. Try giving Jeanne more time to reach the
targeted outcome.
d. Modify the plan of care after discussing
possible reasons for Jeanne's partial success
d. Modify the plan of care after discussing possible
reasons for Jeanne's partial success.

The following are all classic elements of
evaluation. What is the correct sequence?
1. Interpreting and summarizing findings
2. Collecting data to determine whether
evaluative criteria and standards are met.
3. Documenting your judgment
4. Terminating, continuing, or modifying the
plan
5. Identifying evaluative criteria and standards
(what you are looking for when you evaluate,
eg, expected pt outcomes)

52134
Which of the following laboratory values would
you expect in a client experiencing prolonged
immobility?
1. Elevated calcium
2. Decreased sodium
3. Elevated hemoglobin
4. Elevated potassium
1. Elevated calcium

A client has been on bed rest for several days.
The client stands, and the nurse notes that the
client's systolic pressure drops 20 mm Hg.
Which of the following should the nurse
document in the medical record?
1. Rebound hypotension
2. Positional hypotension
3. Orthostatic hypotension
4. Central venous hypotension
3. Orthostatic hypotension

The nurse puts elastic stockings on a client
following major abdominal surgery. The nurse
teaches the client that the stockings are used
after a surgical procedure to:
1. prevent varicose veins
2. prevent muscular atrophy
3. ensure joint mobility and prevent
contractures
4. facilitate the return of venous blood to the
heart
4. facilitate the return of venous blood to the heart

You are caring for a client who has
osteoporosis. The nurse is teaching her about
ways to prevent fractures. Which of the
following client statements reflects a need for
further education?
1. "I usually go swimming with my family at the
YMCA 3 times a week."
2. "I need to ask my doctor if i need to have a
bone mineral density check this year."
3. "If i don't drink milk at dinner, i will eat
broccoli or cabbage to get the calcium that i
need in my diet."
4. "The more frequently i walk the more likely i
will be to fall and break my leg. I think i will get
a wheelchair so i don't have to walk any more."
4. "The more frequently i walk, the more likely i
will be to fall and break my leg. I think i will get a
wheelchair so i don't have to walk any more."

The client at greatest risk for developing adverse
effects of immobility is a:
1. 3-year-old child with a fractured femur
2. 78-year-old man in traction for a broken hip
3. 48-year-old woman following a
thyroidectomy
4. 38-year-old woman undergoing a
hysterectomy
2. 78-year-old man in traction for a broken hip

A client who was in a car accident and broke his
femur has been immobilized for 5 days. When
the nurse gets this client out of bed for the first
time, a nursing diagnosis related to the safety of
this client will be:
1. Pain
2. Impaired skin integrity
3. Altered tissue perfusion
4. Risk for activity intolerance
4. Risk for activity intolerance

A client had a left- sided cerebral vascular
accident 3 days ago and is receiving 5000 units
of heparin subcutaneously every 12 hours to
prevent thrombophlebitis. The client is receiving
enternal feedings through a small-bore
nasogastric tube because of dysphagia. Which of
the following symptoms requires the nurse to
call the health care provider immediately?
1. Hematuria
2. Unilateral neglect
3. Limited ROM in the right hip
4. Coughing up moderate amount clear, thin
sputum
1. Hematuria

A home care nurse is preparing the home for a
client who is going home following a left hip
replacement. The client is cooperative and can
partially bear weight. What should the nurse
order from the home medical supply company to
help the client move from the bed to the chair?
1. A trapeze bar
2. A small transfer board
3. A powered standing-assist device
4. An ankle foot orthotic (AFO) for the affected
foot
2. A small transfer board

The nurse is caring for a client who has right-
sided weak-ness. The nurse needs to help the
client walk. What should the nurse order from
the home medical supply company to help the
client move from the bed to the chair?
1. Hold the client's left hand while walking
2. Hold the client's right hand while walking
3. Put a gait belt on the client and provide
support on the left side
4. Put a gait belt on the client and provide
support on the right side
4. Put a gait belt on the client and provide support
on the right side

Before transferring a client from the bed to a
stretcher, which assessment data does the nurse
need to gather?
(choose all that apply)
1. The client's weight
2. How cooperative the client is
3. The client's nutritional status
4. The presence of intravenous (IV) tubes
1. The client's weight
2. How cooperative the client is
4. The presence of intravenous (IV) tubes

THE NURSE SHOULD USE EXTREME
CAUTION WHEN APPLYING HEAT
THERAPY TO WHICH OF THE
FOLLOWING PATIENTS:
A UNCONSCIOUS
B HIGH PAIN SENSITIVITY
C VENOUS ULCER
D RECEIVING STEROIDS
A

WHEN ADMINISTERING ORAL
MEDICATIONS, WHICH OF THE
FOLLOWING PRACTICES SHOULD THE
NURSE FOLLOW(SELECT ALL THAT
APPLYS)
A DISPENSE MULTIPLE LIQUID
MEDICATIONS INTO A SINGLE CUP TO
REDUCE THE NUMBER OF CONTAINERS
THE PATIENT MUST HANDLE
B PERFORM HAND HYGIENE BEFORE
AND AFTER MEDICATION
ADMINISTRATION
C STAY AT THE BEDSIDE UNTIL THE
PATIENT HAS FINISHED ALL
MEDICATIONS
D KEEP THE PATIENTS MAR AT THE
BEDSTIME AT ALL TIMES
E VERIFY THE PATIENTS RESPONSE TO
THE MEDICATION 30 MINUTES AFTER
ADMINISTRATION, OR AS APPROPRIATE
FOR THE DRUG
BCE

THE NURSE IS PREPARING TO
ADMINISTER A MEDICATION VIA NG
TUBE. WHAT GUIDELINE IS
APPROPRIATE FOR THE NURSE TO
FOLLOW WHEN ADMINISTERING A
DRUG VIA THIS ROUTE?
FLUSH THE TUBE WITH WATER
BETWEEN EACH MED
...

THE NURSE WOULD RECOGNIZE THAT
AN OBESE MALE PATIENT WHO HAS
BEEN DIAGNOSED WITH OBSTRUCTIVE
SLEEP APNEA FACES AN INCREASED
RISK OF WHICH OF THE FOLLOWING?
A DEPRESSION
B RESPIRATORY ACIDOSIS
C HEART DISEASE
D SEIZURES
C

A GRADUATE NURSE IS ADMINISTERING
SEVERAL MEDICATIONS TO A NEWLY
ADMITTED PATIENT. WHO IS LEGALLY
RESPONSIBLE FOR THE DRUGS
ADMINISTERED BY THIS NURSE?
A THE NURSE ADMINISTERING THE
DRUG
B PHARMACIST WHO DISPENSED
C NURSE MANAGER
D PHYSICIAN WHO WROTE THE ORDER
A

WHICH ONE OF THE NUTRITIONAL
GUIDELINES SHOULD THE NURSE GIVE
A WOMAN IN HER 2ND TRIMESTER OF
PREGNANCY
A EAT NORMAL NUMBER OF CALORIES
BUT INCREASE FRUITS AND
VEGETABLES
B MAINTAIN REG CALORIE INTAKE, BUT
TAKE SUPPLEMENTS
C EAT AS MUCH AS YOU CAN
D MORE CALORIES AND HIGH IN
NUTRIENTS
D

PATIENT TELLS NURSE "I CANT GET ANY
SLEEP AROUND HERE" NURSES FIRST
RESPONSE:
A ADD MORE CARBS TO DINNER
B ASSESS FACTORS THAT PATIENT
BELIEVES TO BE PROBLEM
C TEACH PATIENT RELAXATION
TECHNIQUES AND REDUCE NOISE ON
THE UNIT
D OBTAIN PRN ODER FOR SEDATIVE
B

THE DRESSING CHANGE ON A DEEP
UPPER-ARM WOUND IS PAINFUL FOR
THE PATIENT. WHEN PREPARING A
CARE PLAN FOR THE PATIENT, THE
NURSE WILL INCORPORATE WHICH OF
THE FOLLOWING MEASURES:
A ADMINISTER ANALGESIC
IMMEDIATELY BEFORE DRESSING
CHANGE
B PERFORM DRESSING CHANGE WHEN
PATIENT IS FATIGUED FROM PT
C PERFORM DRESSING CHANGE DURING
MEALTIME SO PATIENT IS DISTRACTED
D ADMINISTER ANALGESIC 30-45 MIN
BEFORE DRESSING CHANGE
D

THE PHYSICIANS ADMITTING ORDERS
INDICATE THAT THE PATIENT IS TO BE
PLACED IN A FOWLERS POSITION, UPON
POSITIONING THIS PATIENT, HOW MUCH
WILL THE NURSE ELEVATE THE HEAD?
A 15
B 90
C 45-60
D 30
C

PRIOR TO STARTING A TUBE FEEDING,
THE NURSE ASSESSES THE PH AND
COLOR OF THE PATIENT'S GASTRIC
CONTENTS AND RECEIVES A PH
READING OF 6.2 AND THE ASPIRATE IS
OFF-WHITE COLOR.
A STOMACH
B SMALL INTESTINE
C COLON
D RESPIRATORY TRACT
D

DURING A SKIN ASSESSMENT, THE
NURSE RECOGNIZES THE 1ST
INDICATION THAT A PRESSURE ULCER
MAY BE DEVELOPING WHEN SHE
NOTICES THE SKIN IS WHICH COLOR?
A BLUE
B WHITE
C YELLOW
D RED
B

WHICH MEDICATION WILL DELAY
HEALING OF A POST-OP WOUND
A LAXATIVE
B ANTIHYPERTENSIVE
C CORTICOSTEROID
D K+ SUPPLEMENT
C

THE NURSE WOULD RECOGNIZE WHICH
OF THE FOLLOWING PATIENTS TO HAVE
IMPAIRED WOUND HEALING
A NPO FOLLOWING SURGERY
B OBESE WOMAN WITH TYPE 1
DIABETES
C MAN WITH SEDENTARY LIFESTYLE
AND LIFELONG SMOKER
D A WOMAN WHO'S BREAST
RECONSTRUCTION SURGERY REQUIRED
NUMEROUS INCISION
B

UPON RESPONDING TO A PATIENTS
CALL BELL, THE NURSE DISCOVERS
THAT THE PATIENT'S WOUND HAS
DEHISCED. INITIAL NURSING
MANAGEMENT INCLUDES WHICH OF
THE FOLLOWING
A COVERING THE WOUND AREA WITH
STERILE TOWELS MOISTENED WITH
STERILE 0.9% SALINE
B CLOSING WOUND WITH STERI STRIPS
C HOLDING WOULD TOGETHER AND
COVER WITH BLANKET
D POURING H202 INTO ABDOMINAL
CAVITY AND PACKING WITH GAUZE
A

AT WHAT POINT SHOULD THE NURSE DO
THE 3 CHECKS OF MEDICATION
ADMINISTRATION?
A AS THE NURSE REACHES FOR THE
DRUG PACKAGE
B WHEN REVIEWING THE PATIENT;S
MAR
C AT THE BEGINNING OF SHIFT
D AFTER RETRIEVING THE DRUG
A

What are the 10 Rights of Medication
Administration
Medication
Assessment
Dose
Documentation
Route
Patient
Education
Timing
Evaluation
Refusal
(MADDRPETER)

blood+water
Serosanguinous

blood
Sanguinous

brown green or yellow
Purulent

True or False
The character of the exudate, in amount, color
and odor, can help to identify the exact nature of
the infection
True

Stage I pressure Ulcer
Nonblanchable erythema of intact skin, the
heralding lesion of skin ulceration. In individuals
with darker skin, discoloration of the skin, warmth,
edema, induration, or hardness may also be
indicators

Stage II pressure Ulcer
Partial thickness skin loss involving epidermis,
dermis, or both. The ulcer is superficial and
presents clinically as an abrasion, blister, or
shallow crater.

Stage III pressure Ulcer
Full thickness skin loss involving damage to or
necrosis of subcutaneous tissue that may extend
down to, but not through, underlying fascia. The
ulcer presents clinically as a deep crater with or
without undermining of adjacent tissue.

Stage IV pressure Ulcer
Full thickness skin loss with extensive destruction,
tissue necrosis, or damage to muscle, bone, or
supporting structures (e.g., tendon, joint capsule).
Undermining and sinus tracts also may be
associated with this type of pressure ulcer.

All of the following are examples of increased
risk for pressure ulcers (select all that apply)
A Wheelchair Bound
B Peripheral Vascular Disease
C Diabetes
D Malnourishment
E Incontinence
ABCDE

A nurse is performing wound care. Which of the
following practices violates surgical asepsis?
a. Holding sterile objects above the waist
b. Considering a 1″ edge around the sterile field
as being contaminated
c. Pouring solution onto a sterile field cloth
d. Opening the outermost flap of a sterile
package away from the body
C

Nurse Kate is changing a dressing and providing
wound care. Which activity should she perform
first?
a. Assess the drainage in the dressing.
b. Slowly remove the soiled dressing
c. Wash hands thoroughly.
d. Put on latex gloves.
C

Which of the following clients would least
likely be at risk of developing skin breakdown?
a. A client incontinent of urine feces
b. A client with chronic nutritional deficiencies
c. A client with decreased sensory perception
d. A client who is unable to move about and is
confined to bed
C

The evening nurse reviews the nursing
documentation in the male client's chart and
notes that the day nurse has documented that the
client has a stage II pressure ulcer in the sacral
area. Which of the following would the nurse
expect to note on assessment of the client's
sacral area?
a. Intact skin
b. Full-thickness skin loss
c. Exposed bone, tendon, or muscle
d. Partial-thickness skin loss of the dermis
D

The nurse has delegated administration of 10am
medications to an LPN/LVN. At 10:15am, the
nurse notes none of the medications have been
administer yet. Which is the best action for the
nurse to take?

a) ask another LPN/LVN assigned to the unit to


help administer medications
b) begin administering the medications
c) report he situation to the head nurse
d) ask the LPN/LVN to give the nurse a status
report
A

A nurse on a pediatric unit is preparing the
assignment for the evening shift. The unit
employs unlicensed assistive personnel (UAP).
Which task is most appropriate for the nurse to
delegate to the UAP?

a) setting up Bryant's traction


b) completing the FACES pain scale for a child
with sickle cell crisis
c) obtaining post-operative vital signs on a client
status post-tonsillectomy
d) setting up an intravenous therapy pump
A

An operating room nurse has just finished
setting up a sterile field for a kidney transplant
surgery. She gets word that the donor organ will
not be available for another thirty minutes.
Which of the following is the best course of
action?

A)
Personally watch the sterile field to ensure that
it is not broken.
B)
Place cones or barriers in front of the main OR
doors.
C)
Place sterile drapes over all surfaces.
D)
Thirty minutes is too long. The sterile field will
need to be broken and reestablished later
A

When teaching a patient about wound healing,
the nurse should tell the patient:
A) Inadequate nutrition delays wound healing
and increases risk of infection.
B) Chronic wounds heal more efficiently in a
dry, open environment, so leave them open to
air when possible.
C) Long-term steroid therapy diminishes the
inflammatory response and speeds wound
healing.
D) Fat tissue heals more readily because there is
less vascularization.
A

The nurse is caring for a patient who had knee
replacement surgery 5 days ago. The patient's
knee appears red and is very warm to the touch.
The patient requests pain medication. Which of
the following would be a correct explanation of
what the nurse is noticing?
A) These are expected findings for this
postoperative time period.
B) The patient may becoming dependent upon
pain medication.
C) The nurse should observe the patient more
closely for wound dehiscence.
D) The patient is demonstrating signs of a
postoperative wound infection.
D

The nurse is caring for a patient after major
abdominal surgery. Which of the following
demonstrates correct understanding in regard to
wound dehiscence?
A) The nurse should be alert for an increase in
serosanguineous drainage from the wound.
B) Wound dehiscence is most likely to occur
during the first 24 to 48 hours after surgery.
C) The nurse should administer cough
suppressant to prevent wound dehiscence.
D) The condition is an emergency that requires
surgical repair.
A

A contaminated or traumatic wound may show
signs of infection within 24 hours. A surgical
wound infection usually develops
postoperatively within 14 days.
True
False
false

Healing by primary intention is expected when
the edges of a clean surgical incision are sutured
or stapled together, tissue loss is minimal or
absent, and the wound is uncontaminated by
microorganisms.
True
False
true

Which of the following patients have risk
factors for developing a wound infection?
(Select all that apply.)
A) An 80-year-old man who has a burn
B) A 17-year-old patient who has a metal
fragment lodged in his thigh
C) A 30-year-old female who had an episiotomy
after childbirth
D) A patient receiving chemotherapy who has a
surgical incision
E) A patient with peripheral vascular disease
and an ulcer on the heel
ABDE

How can you determine a patient's history of
allergies? (Select all that apply.)
A) By looking at the patient's allergy bracelet
B) By looking at the MAR
C) By asking the patient
D) By looking at the front of the chart
E) By administering a dose and monitoring the
patient's response
ABCD

Heat or Cold?
Increased Capillary Permeability
Heat

Heat or Cold?
Relieves Pain
Cold

Heat or Cold?
Increased Blood Viscosity
Cold

Heat or Cold?
Decreased Blood Viscosity
Heat

Heat or Cold?
Reduced cell metabolism
Cold

Heat or Cold?
Vasodilation
Heat

Heat or Cold?
Reduced muscle tension
Heat

Heat or Cold?
Vasoconstriction
Cold

Heat or Cold?
Local Anesthesia
Cold

Heat or Cold?
Increased Tissue metabolism
Heat

Heat or Cold?
Promotes movement of wastes and nutrients
Heat

Heat or Cold?
Promotes Muscle Relaxation
Heat

Heat or Cold?
Reduces Inflammation
Cold

Heat or Cold?
Improves delivery of leukocytes to wound
Heat

Heat or Cold?
Decreased blood flow to injured site
Cold

Heat or Cold?
Helps prevent edema from forming
Cold

Heat or Cold?
Increases Blood flow
Heat

Heat or Cold?
Reduces O2 needs of tissues
Cold

Heat or Cold?
Promotes blood coagulation at injury site
Cold

Heat or Cold?
Improves delivery of antibiotics to wound
Heat

Heat or Cold?
Decreases Spasmodic Pain
Heat

Advantages of Oral, Buccal and Sublingual
Routes
--Conveinent and comfortable
--Economical
--Sometimes produce local or systemic effects
--rarely cause anxiety

Disadvantages of Oral, Buccal and Sublingual
Routes
--GI irritation

Advantages of Parenteral Routes
--can be used when oral drugs are contraindicated
--more rapid absorption
--epidural provides excellent pain control

Disadvantages of Parenteral Routes
--introducing infection
--tissue damage
--more expensive
--quicker absorption=quicker adverse reactions
--more painful

Advantages for Skin
--local effect
--painless
--limited side effects

Disadvantages for Skin
--absorption occurs too rapid over abrasions
--medications overall absorb slowly through this
route

Advantages for MM
--local application provides therapeutic effects
--aqueous solutions readily absorbed and capable
of causing systemic effects
--potential ROA when oral drugs are
contraindicated

Disadvantages for MM
--highly sensitive
--awkward(vaginal and rectal)

Advantages for Inhalation
--rapid relief

Disadvantages for Inhalation
--serious systemic effects

Position of Ear for child
Down and Back

Position of ear for adult
Up and Back

Type of Syringe and Needle for ID
1ml
25-27 g
3/8-5/8"

Type of Syringe and Needle for SQ
3ml
25-29 g
1/2-5/8"

Type of Syringe and Needle for IM
3ml
20-25g
1-1.5"

Administration of intraocular disk
position convex side on fingertip
place on conjuctival sac btw iris and lower lid
gently pull eyelid over disk
carefully pinch disk to remove from patient's eye

What is the purpose of a spacer?
it helps the medication reach the lungs
used in children and elderly
helps avoid mouth fungus,nervousness and other
side effects

NREM stage 1
A few minutes, light sleep, easily aroused, gradual
reduction in vital signs

NREM stage 2
10-20 min, can be awakened w/effort, deeper
relaxation

NREM stage 3
15-30 min, early phase of deep sleep, snoring,
relaxed muscle tone, little/no physical movement,
difficult to arouse

NREM stage 4
15-30 min, shortens toward morning, deep sleep,
sleep-walking, sleep-talking, bed-wetting may
occur

REM sleep
a recurring sleep state during which dreaming
occurs

Circadian Rhythm
the biological clock; regular bodily rhythms that
occur on a 24-hour cycle

Functions of Sleep
Restoration, reducing fatigue,stabilizing mood,
improving blood flow to the brain, increasing
protein synthesis, maintaining the disease-fighting
mechanisms of the immune system,promoting
cellular growth and repair, improving the capacity
for learning and memory storage

Factors affecting sleep
physical illness
drugs and substances
emotional stress
environment
lifestyle
exercise and fatigue
food and caloric intake
sound

transduction of pain
process that begins in the periphery when pain-
producing stimulus send an impulse across a
peripheral nerve fiber

transmission of pain
movement of pain impulses from the periphery to
the spinal cord & then to the brain

perception of pain
protects the body from damage, and is stimulated
by extremes of pressure and temperature, as well
as chemicals released from damaged
tissues(physical component)

pain modulation
hindering the transmission of pain by release of
inhibitory neurotransmitters
(endorphins&enkephalins) that produce an
analgesic effect

NSAIDs inhibit the synthesis of
prostaglandins

A patient's age, gender,anxiety, culture, and
__________ influence the pain experience
previous experience&meaning of pain

Evaluation of pain therapy requires the
consideration of the ________ character of pain,
____ to therapy, ___ to function, and patient's
perception of a therapy's effectivness
changing
response
ability

hypostatic pneumonia
inflammation of the lung from stasis or pooling of
secretions from lack of movement and exercise

reactive hyperemia
a bright red flush on the skin occurring after
pressure is relieved

blanchable hyperemia
redness of the skin due to dilation of the superficial
capillaries. When pressure is applied to the skin,
the area blanches, or turns a lighter color

nonblanchable hyperemia
redness of the skin due to dilation of the superficial
capillaries. The redness persists when pressure is
applied to the area, indicating tissue damage

Unstageable
full thickness tissue loss in which the base of the
ulcer is covered by slough and/or eschar

partial thickness wound repair
- inflammatory response (24hrs.)
- epithelial proliferation/migration
- reestablishment of epidermal layers

full thickness wound repair
- inflammatory (up to 3 days)
- proliferative (3-24 days)
- remodeling (up to 1 yr.)

evisceration
wound separation with protrusion of organs

fistula
abnormal passageway between two organs or
between an internal organ and the body surface

dihiscence
rupture separion of one or more layers of a wound.

incision
surgical cut or wound produced by a sharp
instrument

abrasion
Scraping or wearing away of the skin by friction;
irritation

contusion
an injury to underlying tissues without breaking
the skin and is characterized by discoloration and
pain

open wound
an injury in which the skin is interrupted, exposing
the tissue beneath

closed wound
wound that involves underlying tissue without
break in the skin

puncture wound
An open wound that tears through the skin and
destroys underlaying tissues. A penetrating
puncture wound can be shallow or deep. A
perforating puncture wound has both an entrance
and an exit wound.

hematoma
the collection of blood under the skin as the result
of blood escaping into the tissue from damaged
blood vessels. bruise

crushing injury
an injury caused by compression that involves both
direct tissue injury caused by circulation
disturbance resulting from pressure on blood
vessels

primary intention
wounds that heal under conditions of minimal
tissue loss(partial thickness)

secondary intention
complex healing of a larger wound involving
sealing of the wound through scab formation,
granulation or filling of the wound, and
constriction of the wound. (full thickness)

Which phrase best describes the science of
nursing?
a. The skilled application of knowledge
b. The knowledge base for care
c. Hands-on care, such as giving a bath
d. Respect for each individual patient
b. the science of nursing is the knowledge base for
care that is provided. In contrast, the skilled
application of that knowledge is the art of nursing.

Which nurse in history is credited with
establishing nursing education?
a. clara barton
b. lilian wald
c. lavinia dock
d. florence nightingale
d. Florence Nightingale established nursing
education

What historic event in the 20th century led to an
increased emphasis on nursing and broadened
the role of nurses?
a. religious reform
b. crimean war
c. world war II
d. Vietnam War
c. World War II

Which of the following phrases describes one of
the purposes of the ANA's nursing's social
policy statement?
a. to describe the nurse as a dependent caregiver
b. To provide standards for nursing educational
programs
c. to regulate nursing research
d. to describe nursing's values and social
responsiblity
d. The nursing's social policy statement describes
the values and social responsibility of nursing

A school nurse is teaching a class of junior-high
students about the effects of smoking. This
educational program will meet which of the
aims of nursing?
a. promoting health
b. preventing illness
c. restoring health
d. facilitating coping with disability or death
b. Educational programs can reduce the risk of
illness by teaching good health habits

Which of the following nursing degrees
prepares a nurse for advanced practice as a
clinical specialist or nurse practitioner?
a. LPN
b. ADN
c. BSN
d. Master's
d. A Master's degree prepares advanced practice
nurses.

Which nursing organization was the first
international organization of professional
women?
a. ICN
b. ANA
c. NLN
d. NSNA
a. The ICN, founded in 1899, was the first
international organization of professional women.

What is the purpose of the ANA's Scope and
Standards of Practice?
a. To describe the ethical responsibility of
nurses
b. To define the activities that are special and
unique to nursing
c. To establish nursing as an independent and
free standing profession
d. To regulate the practice of nursing
b. The ANA's Scope and Standards of Practice
define the activities of nurses that are specific and
unique to nursing.

What type of authority regulates the practice of
nursing?
a. International standards and codes
b. Federal guidelines and regulations
c. State nurse practice acts
d. Institutional policies
c. Nurse practice act are established in each state to
regulate the practice of nursing

Who are the largest group of healthcare
providers in the United States?
a. Registered Nurses
b. Physicians
c. Physical therapists
d. Social Workers
a. Registered nurses are the largest group of
healthcare providers in the United States

Which of the following phrases best defines
culture?
a. A dominant group within a society
b. A shared system of beliefs, values, and
behaviors
c. One's values are replaced by the values of the
dominant culture
d. Categories are based on specific physical
characteristics
b. Culture may be defined as a shared system of
beliefs, values, and behavioral expectations that
provide social structure for daily living

Minority groups living within a dominant
culture may lose the cultural characteristics that
made them different. What is this process
called?
a. cultural diversity
b. cultural imposition
c. cultural assimilation
d. ethnocentrism
c. When minority groups live within a dominant
group, many members lose the cultural
characteristics that once made them different

Which of the following terms is defined as the
sense of identification with a collective cultural
group?
a. ethnicity
b. race
c. cultural acquisition
d. culture shock
a. Ethnicity is the sense of identification with a
collective cultural group, largely based on the
group's common heritage.

A nurse states, that woman is 78 years old-too
old to learn how to change a dressing. What is
the nurse demonstrating?
a. cultural imposition
b. clustering
c. cultural competency
d. stereotyping
d. stereotyping is assuming that all members of a
group are alike.

A young hispanic mother comes to the local
clinic because her baby is sick. She speaks only
Spanish and the nurse speaks only English.
What should the nurse do?
a. Use short words and talk more loudly?
b. Ask an interpreter for help
c. Explain why care can't be provided
d. Provide instructions in writing.
b. Many agencies have a qualified interpreter who
understands the healthcare system and can reliably
provide assistance.

A nurse is interviewing a newly admitted
patient. Which question would be considered
culturally sensitive?
a. do you think you will be able to eat the food
we have here?
b. Do you understand that we can't prepare
special meals?
c. What types of food do you eat for meals?
d. Why cant you just eat our food while you are
here?
c. Asking patients what types of foods they eat for
meals is culturally sensitive

What group is the largest subculture of the
healthcare system?
a. nurses
b. physicians
c. social workers
d. physical therapists
a. nurses are the largest subculture of the
healthcare system

A nurse states, I know I am cleaner than most of
my patients. What does this statement indicate?
a. cultural assimilation
b. racism
c. ethnocentrism
d. sterotyping
c. Ethnocentrism occurs when one believes that
one's own ideas and practices are superior to those
of others.

A nurse wants to acquire knowledge of a
specific culture. What could be done first?
a. talk to coworkers
b. review literature
c. talk to family members of the patient
d. ask others with more experience for help
b. reviewing literature about a specific culture can
provide the nurse with a starting point for
information about cultural values, dietary
practices, family lines of authority, and helth and
illness beliefs and practices.

Although all of the following are important to
culturally competent nursing care, which one is
the most basic?
a. learning another language
b. having signifigant information
c. treating each person as an individual
d. recognizing the importance of family
c. In all aspects of nursing, it is important to treat
each patient as an individual. This is also true in
providing culturally competent care.

A nurse is providing care based on Maslow's
hierarch of basic human needs. For which of the
following nursing activities is the approach
useful?
a. making accurate nursing diagnoses
b. establishing priorities of care
c. communicating concerns more concisely
d. integrating science into nursing care
b. Maslow's hierarch of basic human needs is
useful for establishing priorities of care.

Which of the following levels of basic human
needs is most basic?
a. physiologic
b. safety and security
c. love and belonging
d. self-actualization
a. physiologic

Of all physiologic needs, which one is the most
essential?
a. food
b. water
c. elimination
d. oxygen
d. oxygen

Practicing careful hand hygiene and using sterile
techniques are ways in which nurses meet which
basic human need?
a. physiologic
b. safety and security
c. self esteem
d. love and belonging
b. safety and security

Of the following statements, which one is true
of self actualization?
a. Humans are born with fully developed self-
actualization
b. self-actualization needs are met by having
confidence and indepence
c. The self-actualization process continues
throughout life.
d. loneliness and isolation occur when self-
actualization needs are unmet
c. self actualization, or reaching one's full potential
is a process that continues through life

What is the best broad definition of family?
a. a father, a mother, and children
b. a gruop whose members are biologically
related
c. a unit that includes aunts, uncles, and cousins
d. a group of people who live together
d. although all of the responses may be true, the
best definition is a group of people who live
together

Where do individuals learn their health beliefs
and values?
a. in the family
b. in school
c. from school nurses
d. from peers
a. healthcare activities, health beliefs, and values
are learned within one's family.
See More
B
besides high blood pressure values, what other
signs and symptoms may the nurse observe if
hypertension is present?
A) Unexplained pain and hyperactivity
B) Headache, flushing of the face, and nosebleed
C) Dizziness, mental confusion, and mottled
extremities
D) Restlessness and dusky or cyanotic skin that is
cool to the touch

D
Which of the following vlues for vital signs would
the nurse address first?
A) Heart rate = 72 beats per minute
B) Respiration rate = 28 breaths per minute
C) Blood pressure = 160/86
D) Oxygen saturation by pulse oximetry = 89%
E) Temperature = 37.2° C (99° F), tympanic

D
An 82-year-old widower brought via ambulance is
admitted to the emergency department with
complaints of shortness of
breath, anorexia, and malaise. He recently visited
his health care provider and was put on an
antibiotic for pneumonia. The client indicates that
he also takes a diuretic and a beta blocker, which
helps his "high blood." Which vital sign value
would take priority in initiating care?
A) Respiration rate = 20 breaths per minute
B) Oxygen saturation by pulse oximetry = 92%
C) Blood pressure = 138/84
D) Temperature = 39° C (102° F), tympanic

C
The client, who has been on bed rest for 2 days,
asks to get out of bed to go to the bathroom. He
has new orders for "up ad lib." What action should
the nurse take?
A) Give him some slippers and tell him where the
bathroom is located.
B) Ask the nursing assistant to assist him to the
bathroom.
C) Obtain orthostatic blood pressure
measurements.
D) Tell him it is not a good idea and provide a
urinal.

A
Using an oral electronic thermometer, the nurse
checks the early morning temperature of a client.
The client's temperature is 36.1° C (97° F). The
client's remaining vital signs are in the normally
acceptable range. What should the nurse do next?
A) Check the client's temperature history.
B) Document the results; temperature is normal.
C) Recheck the temperature every 15 minutes until
it is normal.
D) Get another thermometer; the temperature is
obviously an error.

B
The nurse decides to take an apical pulse instead of
a radial pulse. Which of the following client
conditions influenced the nurse's decision?
A) The client is in shock.
B) The client has an arrhythmia.
C) The client underwent surgery 18 hours earlier.
D) The client showed a response to orthostatic
changes.

D
The nurse is to measure vital signs as part of the
preparation for a test. The client is talking with a
visiting pastor. How should the nurse handle
measuring the rate of respiration?
A) Count respirations during the time the client is
not talking to the visitor.
B) Wait at the client's bedside until the visit is over
and then count respirations.
C) Tell the client it is very important to end the
conversation so the nurse can count respirations.
D) Document the respiration rate as "deferred" and
measure the rate later, since the talking client is
obviously not in respiratory distress.

D
Delegation of some tasks may become one of the
decisions the nurse will make while on duty. For
which of the following clients would it be most
appropriate for unlicensed assistive personnel to
measure the client's vital signs?
A) A client who recently started taking an
antiarrhythmic medication
B) A client with a history of transfusion reactions
who is receiving a blood transfusion
C) A client who has frequently been admitted to
the unit with asthma attacks
D) A client who is being admitted for elective
surgery who has a history of stable hypertension

D
The client has an oral temperature of 39.2° C
(102.6° F). What are the most appropriate nursing
interventions?
A) Provide an alcohol sponge bath and monitor
laboratory results.
B) Remove excess clothing, provide a tepid sponge
bath, and administer an analgesic.
C) Provide fluids and nutrition, keep the client's
room warm, and administer an analgesic.
D) Reduce external coverings and keep clothing
and bed linens dry; administer antipyretics as
ordered.

D
The hypothalamus controls body temperature. The
anterior hypothalamus controls heat loss, and the
posterior hypothalamus controls heat production.
What heat conservation mechanisms will the
posterior hypothalamus initiate when it senses that
the client's body temperature is lower than
comfortable?
A) Vasodilation and redistribution of blood to
surface vessels
B) Sweating, vasodilation, and redistribution of
blood to surface vessels
C) Vasoconstriction, sweating, and reduction of
blood flow to extremities
D) Vasoconstriction, reduction of blood flow to
extremities, and shivering

C
The nurse's documentation indicates that a client
has a pulse deficit of 14 beats. The pulse deficit is
measured by:
A) Subtracting 60 (bradycardia) from the client's
pulse rate and reporting the difference
B) Subtracting the client's pulse rate from 100
(tachycardia) and reporting the difference
C) Assessing the apical pulse and the radial pulse
for the same minute and subtracting the difference
D) Assessing the apical pulse and 30 minutes later
assessing the carotid pulse and subtracting the
difference

D
The nurse observes that a client's breathing pattern
represents Cheyne-Stokes respiration. Which
statement best describes the Cheyne-Stokes
pattern?
A) Respirations cease for several seconds.
B) Respirations are abnormally shallow for two to
three breaths followed by irregular periods of
apnea.
C) Respirations are labored, with an increase in
depth and rate (more than 20 breaths per minute);
the condition occurs normally during exercise.
D) Respiration rate and depth are irregular, with
alternating periods of apnea and hyperventilation;
the cycle begins with slow breaths and climaxes in
apnea.

D
The nurse finds that the systolic blood pressure of
an adult client is 88 mm Hg. What are the
appropriate nursing interventions?
A) Check other vital signs.
B) Recheck the blood pressure and give the client
orange juice.
C) Recheck the blood pressure after ambulating the
client safely.
D) Recheck the blood pressure, make sure the
client is safe, and report the findings.

C
52 year old woman admitted with dyspnea and
discomfort in her left chest with deep breaths. SHe
smoked for 35 years and recently lost over 10
pounds. What vital sign should not be delegated to
a nursing assistant:
a) temperature
b) radial pulse
c) respiratory rate
d) oxygen saturation

1, 5, 2, 4, 3
Place the vital signs in order of priority for your
nursing interventions:
1) SpO2= 89%
2) BP= 160/86 mmHG
3) Temperature= 37.3 (99.4)
4) HR= 72 BPM
5) RR= 28 BrPM

1, 2, 4, 7
82 yr old admitted via ambulance to ER with s

hortness of breath, anorexia, and malaise. He


recently visited the health care center and is on
antibiotic for pneumonia. He is also on a diuretic,
beta-adrergic blocker, which helps his "high
blood".
He has a temperature of 38.2 (100.8) via temporal
artery. What additional assessment data is needed
in planning intervention for the patients infection ?
(choose all that apply)
1. HR
2. Skin turgor
3. Smoking history
4. Allergies to antibiotics
5. Recent BM's
6. BP in right arm
7. Client's normal temperature
8. BP in distal extremity
Please allow access to your computer’s microphone to use Voice
Recording.
Having trouble? Click here for help.

Combo with Fundamentals Nclex- Ch. 47 and 1 other

Created by chn1020
 Study
 Info
Add to Folder
Share
CopyPrint
More Tools 

All 458
Flashcards

All 458

Learn

All 458

Speller

All 458

Test

All 458

Scatter
All 458

Space Race
Upgrade to
remove ads
This set has 458 terms
OriginalAlphabetical

Which of the following laboratory values would you expect in a
client experiencing prolonged immobility?
1. Elevated calcium
2. Decreased sodium
3. Elevated hemoglobin
4. Elevated potassium
1. Elevated calcium

A client has been on bed rest for several days. The client stands, and
the nurse notes that the client's systolic pressure drops 20 mm Hg.
Which of the following should the nurse document in the medical
record?
1. Rebound hypotension
2. Positional hypotension
3. Orthostatic hypotension
4. Central venous hypotension
3. Orthostatic hypotension

The nurse puts elastic stockings on a client following major
abdominal surgery. The nurse teaches the client that the stockings
are used after a surgical procedure to:
1. prevent varicose veins
2. prevent muscular atrophy
3. ensure joint mobility and prevent contractures
4. facilitate the return of venous blood to the heart
4. facilitate the return of venous blood to the heart

You are caring for a client who has osteoporosis. The nurse is
teaching her about ways to prevent fractures. Which of the following
client statements reflects a need for further education?
1. "I usually go swimming with my family at the YMCA 3 times a
week."
2. "I need to ask my doctor if i need to have a bone mineral density
check this year."
3. "If i don't drink milk at dinner, i will eat broccoli or cabbage to
get the calcium that i need in my diet."
4. "The more frequently i walk the more likely i will be to fall and
break my leg. I think i will get a wheelchair so i don't have to walk
any more."
4. "The more frequently i walk, the more likely i will be to fall and
break my leg. I think i will get a wheelchair so i don't have to walk any
more."

The client at greatest risk for developing adverse effects of
immobility is a:
1. 3-year-old child with a fractured femur
2. 78-year-old man in traction for a broken hip
3. 48-year-old woman following a thyroidectomy
4. 38-year-old woman undergoing a hysterectomy
2. 78-year-old man in traction for a broken hip

A client who was in a car accident and broke his femur has been
immobilized for 5 days. When the nurse gets this client out of bed
for the first time, a nursing diagnosis related to the safety of this
client will be:
1. Pain
2. Impaired skin integrity
3. Altered tissue perfusion
4. Risk for activity intolerance
4. Risk for activity intolerance

A client had a left- sided cerebral vascular accident 3 days ago and
is receiving 5000 units of heparin subcutaneously every 12 hours to
prevent thrombophlebitis. The client is receiving enternal feedings
through a small-bore nasogastric tube because of dysphagia. Which
of the following symptoms requires the nurse to call the health care
provider immediately?
1. Hematuria
2. Unilateral neglect
3. Limited ROM in the right hip
4. Coughing up moderate amount clear, thin sputum
1. Hematuria

A home care nurse is preparing the home for a client who is going
home following a left hip replacement. The client is cooperative and
can partially bear weight. What should the nurse order from the
home medical supply company to help the client move from the bed
to the chair?
1. A trapeze bar
2. A small transfer board
3. A powered standing-assist device
4. An ankle foot orthotic (AFO) for the affected foot
2. A small transfer board

The nurse is caring for a client who has right-sided weak-ness. The
nurse needs to help the client walk. What should the nurse order
from the home medical supply company to help the client move
from the bed to the chair?
1. Hold the client's left hand while walking
2. Hold the client's right hand while walking
3. Put a gait belt on the client and provide support on the left side
4. Put a gait belt on the client and provide support on the right side
4. Put a gait belt on the client and provide support on the right side

Before transferring a client from the bed to a stretcher, which
assessment data does the nurse need to gather?
(choose all that apply)
1. The client's weight
2. How cooperative the client is
3. The client's nutritional status
4. The presence of intravenous (IV) tubes
1. The client's weight
2. How cooperative the client is
4. The presence of intravenous (IV) tubes

THE NURSE SHOULD USE EXTREME CAUTION WHEN
APPLYING HEAT THERAPY TO WHICH OF THE
FOLLOWING PATIENTS:
A UNCONSCIOUS
B HIGH PAIN SENSITIVITY
C VENOUS ULCER
D RECEIVING STEROIDS
A

WHEN ADMINISTERING ORAL MEDICATIONS, WHICH OF
THE FOLLOWING PRACTICES SHOULD THE NURSE
FOLLOW(SELECT ALL THAT APPLYS)
A DISPENSE MULTIPLE LIQUID MEDICATIONS INTO A
SINGLE CUP TO REDUCE THE NUMBER OF CONTAINERS
THE PATIENT MUST HANDLE
B PERFORM HAND HYGIENE BEFORE AND AFTER
MEDICATION ADMINISTRATION
C STAY AT THE BEDSIDE UNTIL THE PATIENT HAS
FINISHED ALL MEDICATIONS
D KEEP THE PATIENTS MAR AT THE BEDSTIME AT ALL
TIMES
E VERIFY THE PATIENTS RESPONSE TO THE MEDICATION
30 MINUTES AFTER ADMINISTRATION, OR AS
APPROPRIATE FOR THE DRUG
BCE

THE NURSE IS PREPARING TO ADMINISTER A
MEDICATION VIA NG TUBE. WHAT GUIDELINE IS
APPROPRIATE FOR THE NURSE TO FOLLOW WHEN
ADMINISTERING A DRUG VIA THIS ROUTE?
FLUSH THE TUBE WITH WATER BETWEEN EACH MED
...

THE NURSE WOULD RECOGNIZE THAT AN OBESE MALE
PATIENT WHO HAS BEEN DIAGNOSED WITH
OBSTRUCTIVE SLEEP APNEA FACES AN INCREASED RISK
OF WHICH OF THE FOLLOWING?
A DEPRESSION
B RESPIRATORY ACIDOSIS
C HEART DISEASE
D SEIZURES
C

A GRADUATE NURSE IS ADMINISTERING SEVERAL
MEDICATIONS TO A NEWLY ADMITTED PATIENT. WHO IS
LEGALLY RESPONSIBLE FOR THE DRUGS ADMINISTERED
BY THIS NURSE?
A THE NURSE ADMINISTERING THE DRUG
B PHARMACIST WHO DISPENSED
C NURSE MANAGER
D PHYSICIAN WHO WROTE THE ORDER
A

WHICH ONE OF THE NUTRITIONAL GUIDELINES SHOULD
THE NURSE GIVE A WOMAN IN HER 2ND TRIMESTER OF
PREGNANCY
A EAT NORMAL NUMBER OF CALORIES BUT INCREASE
FRUITS AND VEGETABLES
B MAINTAIN REG CALORIE INTAKE, BUT TAKE
SUPPLEMENTS
C EAT AS MUCH AS YOU CAN
D MORE CALORIES AND HIGH IN NUTRIENTS
D

PATIENT TELLS NURSE "I CANT GET ANY SLEEP AROUND
HERE" NURSES FIRST RESPONSE:
A ADD MORE CARBS TO DINNER
B ASSESS FACTORS THAT PATIENT BELIEVES TO BE
PROBLEM
C TEACH PATIENT RELAXATION TECHNIQUES AND
REDUCE NOISE ON THE UNIT
D OBTAIN PRN ODER FOR SEDATIVE
B

THE DRESSING CHANGE ON A DEEP UPPER-ARM WOUND
IS PAINFUL FOR THE PATIENT. WHEN PREPARING A CARE
PLAN FOR THE PATIENT, THE NURSE WILL INCORPORATE
WHICH OF THE FOLLOWING MEASURES:
A ADMINISTER ANALGESIC IMMEDIATELY BEFORE
DRESSING CHANGE
B PERFORM DRESSING CHANGE WHEN PATIENT IS
FATIGUED FROM PT
C PERFORM DRESSING CHANGE DURING MEALTIME SO
PATIENT IS DISTRACTED
D ADMINISTER ANALGESIC 30-45 MIN BEFORE DRESSING
CHANGE
D

THE PHYSICIANS ADMITTING ORDERS INDICATE THAT
THE PATIENT IS TO BE PLACED IN A FOWLERS POSITION,
UPON POSITIONING THIS PATIENT, HOW MUCH WILL THE
NURSE ELEVATE THE HEAD?
A 15
B 90
C 45-60
D 30
C

PRIOR TO STARTING A TUBE FEEDING, THE NURSE
ASSESSES THE PH AND COLOR OF THE PATIENT'S
GASTRIC CONTENTS AND RECEIVES A PH READING OF 6.2
AND THE ASPIRATE IS OFF-WHITE COLOR.
A STOMACH
B SMALL INTESTINE
C COLON
D RESPIRATORY TRACT
D

DURING A SKIN ASSESSMENT, THE NURSE RECOGNIZES
THE 1ST INDICATION THAT A PRESSURE ULCER MAY BE
DEVELOPING WHEN SHE NOTICES THE SKIN IS WHICH
COLOR?
A BLUE
B WHITE
C YELLOW
D RED
B

WHICH MEDICATION WILL DELAY HEALING OF A POST-
OP WOUND
A LAXATIVE
B ANTIHYPERTENSIVE
C CORTICOSTEROID
D K+ SUPPLEMENT
C

THE NURSE WOULD RECOGNIZE WHICH OF THE
FOLLOWING PATIENTS TO HAVE IMPAIRED WOUND
HEALING
A NPO FOLLOWING SURGERY
B OBESE WOMAN WITH TYPE 1 DIABETES
C MAN WITH SEDENTARY LIFESTYLE AND LIFELONG
SMOKER
D A WOMAN WHO'S BREAST RECONSTRUCTION SURGERY
REQUIRED NUMEROUS INCISION
B

UPON RESPONDING TO A PATIENTS CALL BELL, THE
NURSE DISCOVERS THAT THE PATIENT'S WOUND HAS
DEHISCED. INITIAL NURSING MANAGEMENT INCLUDES
WHICH OF THE FOLLOWING
A COVERING THE WOUND AREA WITH STERILE TOWELS
MOISTENED WITH STERILE 0.9% SALINE
B CLOSING WOUND WITH STERI STRIPS
C HOLDING WOULD TOGETHER AND COVER WITH
BLANKET
D POURING H202 INTO ABDOMINAL CAVITY AND
PACKING WITH GAUZE
A

AT WHAT POINT SHOULD THE NURSE DO THE 3 CHECKS
OF MEDICATION ADMINISTRATION?
A AS THE NURSE REACHES FOR THE DRUG PACKAGE
B WHEN REVIEWING THE PATIENT;S MAR
C AT THE BEGINNING OF SHIFT
D AFTER RETRIEVING THE DRUG
A

What are the 10 Rights of Medication Administration
Medication
Assessment
Dose
Documentation
Route
Patient
Education
Timing
Evaluation
Refusal
(MADDRPETER)

blood+water
Serosanguinous

blood
Sanguinous

brown green or yellow
Purulent

True or False
The character of the exudate, in amount, color and odor, can help to
identify the exact nature of the infection
True

Stage I pressure Ulcer
Nonblanchable erythema of intact skin, the heralding lesion of skin
ulceration. In individuals with darker skin, discoloration of the skin,
warmth, edema, induration, or hardness may also be indicators

Stage II pressure Ulcer
Partial thickness skin loss involving epidermis, dermis, or both. The
ulcer is superficial and presents clinically as an abrasion, blister, or
shallow crater.

Stage III pressure Ulcer
Full thickness skin loss involving damage to or necrosis of
subcutaneous tissue that may extend down to, but not through,
underlying fascia. The ulcer presents clinically as a deep crater with or
without undermining of adjacent tissue.

Stage IV pressure Ulcer
Full thickness skin loss with extensive destruction, tissue necrosis, or
damage to muscle, bone, or supporting structures (e.g., tendon, joint
capsule). Undermining and sinus tracts also may be associated with
this type of pressure ulcer.

All of the following are examples of increased risk for pressure
ulcers (select all that apply)
A Wheelchair Bound
B Peripheral Vascular Disease
C Diabetes
D Malnourishment
E Incontinence
ABCDE

A nurse is performing wound care. Which of the following practices
violates surgical asepsis?
a. Holding sterile objects above the waist
b. Considering a 1″ edge around the sterile field as being
contaminated
c. Pouring solution onto a sterile field cloth
d. Opening the outermost flap of a sterile package away from the
body
C

Nurse Kate is changing a dressing and providing wound care. Which
activity should she perform first?
a. Assess the drainage in the dressing.
b. Slowly remove the soiled dressing
c. Wash hands thoroughly.
d. Put on latex gloves.
C

Which of the following clients would least likely be at risk of
developing skin breakdown?
a. A client incontinent of urine feces
b. A client with chronic nutritional deficiencies
c. A client with decreased sensory perception
d. A client who is unable to move about and is confined to bed
C

The evening nurse reviews the nursing documentation in the male
client's chart and notes that the day nurse has documented that the
client has a stage II pressure ulcer in the sacral area. Which of the
following would the nurse expect to note on assessment of the
client's sacral area?
a. Intact skin
b. Full-thickness skin loss
c. Exposed bone, tendon, or muscle
d. Partial-thickness skin loss of the dermis
D

The nurse has delegated administration of 10am medications to an
LPN/LVN. At 10:15am, the nurse notes none of the medications
have been administer yet. Which is the best action for the nurse to
take?

a) ask another LPN/LVN assigned to the unit to help administer


medications
b) begin administering the medications
c) report he situation to the head nurse
d) ask the LPN/LVN to give the nurse a status report
A

A nurse on a pediatric unit is preparing the assignment for the
evening shift. The unit employs unlicensed assistive personnel
(UAP). Which task is most appropriate for the nurse to delegate to
the UAP?

a) setting up Bryant's traction


b) completing the FACES pain scale for a child with sickle cell
crisis
c) obtaining post-operative vital signs on a client status post-
tonsillectomy
d) setting up an intravenous therapy pump
A

An operating room nurse has just finished setting up a sterile field
for a kidney transplant surgery. She gets word that the donor organ
will not be available for another thirty minutes. Which of the
following is the best course of action?

A)
Personally watch the sterile field to ensure that it is not broken.
B)
Place cones or barriers in front of the main OR doors.
C)
Place sterile drapes over all surfaces.
D)
Thirty minutes is too long. The sterile field will need to be broken
and reestablished later
A

When teaching a patient about wound healing, the nurse should tell
the patient:
A) Inadequate nutrition delays wound healing and increases risk of
infection.
B) Chronic wounds heal more efficiently in a dry, open
environment, so leave them open to air when possible.
C) Long-term steroid therapy diminishes the inflammatory response
and speeds wound healing.
D) Fat tissue heals more readily because there is less
vascularization.
A

The nurse is caring for a patient who had knee replacement surgery
5 days ago. The patient's knee appears red and is very warm to the
touch. The patient requests pain medication. Which of the following
would be a correct explanation of what the nurse is noticing?
A) These are expected findings for this postoperative time period.
B) The patient may becoming dependent upon pain medication.
C) The nurse should observe the patient more closely for wound
dehiscence.
D) The patient is demonstrating signs of a postoperative wound
infection.
D

The nurse is caring for a patient after major abdominal surgery.
Which of the following demonstrates correct understanding in
regard to wound dehiscence?
A) The nurse should be alert for an increase in serosanguineous
drainage from the wound.
B) Wound dehiscence is most likely to occur during the first 24 to
48 hours after surgery.
C) The nurse should administer cough suppressant to prevent wound
dehiscence.
D) The condition is an emergency that requires surgical repair.
A

A contaminated or traumatic wound may show signs of infection
within 24 hours. A surgical wound infection usually develops
postoperatively within 14 days.
True
False
false

Healing by primary intention is expected when the edges of a clean
surgical incision are sutured or stapled together, tissue loss is
minimal or absent, and the wound is uncontaminated by
microorganisms.
True
False
true

Which of the following patients have risk factors for developing a
wound infection? (Select all that apply.)
A) An 80-year-old man who has a burn
B) A 17-year-old patient who has a metal fragment lodged in his
thigh
C) A 30-year-old female who had an episiotomy after childbirth
D) A patient receiving chemotherapy who has a surgical incision
E) A patient with peripheral vascular disease and an ulcer on the
heel
ABDE

How can you determine a patient's history of allergies? (Select all
that apply.)
A) By looking at the patient's allergy bracelet
B) By looking at the MAR
C) By asking the patient
D) By looking at the front of the chart
E) By administering a dose and monitoring the patient's response
ABCD

Heat or Cold?
Increased Capillary Permeability
Heat

Heat or Cold?
Relieves Pain
Cold

Heat or Cold?
Increased Blood Viscosity
Cold

Heat or Cold?
Decreased Blood Viscosity
Heat

Heat or Cold?
Reduced cell metabolism
Cold

Heat or Cold?
Vasodilation
Heat

Heat or Cold?
Reduced muscle tension
Heat

Heat or Cold?
Vasoconstriction
Cold

Heat or Cold?
Local Anesthesia
Cold

Heat or Cold?
Increased Tissue metabolism
Heat

Heat or Cold?
Promotes movement of wastes and nutrients
Heat

Heat or Cold?
Promotes Muscle Relaxation
Heat

Heat or Cold?
Reduces Inflammation
Cold

Heat or Cold?
Improves delivery of leukocytes to wound
Heat

Heat or Cold?
Decreased blood flow to injured site
Cold

Heat or Cold?
Helps prevent edema from forming
Cold

Heat or Cold?
Increases Blood flow
Heat

Heat or Cold?
Reduces O2 needs of tissues
Cold

Heat or Cold?
Promotes blood coagulation at injury site
Cold

Heat or Cold?
Improves delivery of antibiotics to wound
Heat

Heat or Cold?
Decreases Spasmodic Pain
Heat

Advantages of Oral, Buccal and Sublingual Routes
--Conveinent and comfortable
--Economical
--Sometimes produce local or systemic effects
--rarely cause anxiety

Disadvantages of Oral, Buccal and Sublingual Routes
--GI irritation

Advantages of Parenteral Routes
--can be used when oral drugs are contraindicated
--more rapid absorption
--epidural provides excellent pain control

Disadvantages of Parenteral Routes
--introducing infection
--tissue damage
--more expensive
--quicker absorption=quicker adverse reactions
--more painful

Advantages for Skin
--local effect
--painless
--limited side effects

Disadvantages for Skin
--absorption occurs too rapid over abrasions
--medications overall absorb slowly through this route

Advantages for MM
--local application provides therapeutic effects
--aqueous solutions readily absorbed and capable of causing systemic
effects
--potential ROA when oral drugs are contraindicated

Disadvantages for MM
--highly sensitive
--awkward(vaginal and rectal)

Advantages for Inhalation
--rapid relief

Disadvantages for Inhalation
--serious systemic effects

Position of Ear for child
Down and Back

Position of ear for adult
Up and Back

Type of Syringe and Needle for ID
1ml
25-27 g
3/8-5/8"

Type of Syringe and Needle for SQ
3ml
25-29 g
1/2-5/8"

Type of Syringe and Needle for IM
3ml
20-25g
1-1.5"

Administration of intraocular disk
position convex side on fingertip
place on conjuctival sac btw iris and lower lid
gently pull eyelid over disk
carefully pinch disk to remove from patient's eye

What is the purpose of a spacer?
it helps the medication reach the lungs
used in children and elderly
helps avoid mouth fungus,nervousness and other side effects

NREM stage 1
A few minutes, light sleep, easily aroused, gradual reduction in vital
signs

NREM stage 2
10-20 min, can be awakened w/effort, deeper relaxation

NREM stage 3
15-30 min, early phase of deep sleep, snoring, relaxed muscle tone,
little/no physical movement, difficult to arouse

NREM stage 4
15-30 min, shortens toward morning, deep sleep, sleep-walking, sleep-
talking, bed-wetting may occur

REM sleep
a recurring sleep state during which dreaming occurs

Circadian Rhythm
the biological clock; regular bodily rhythms that occur on a 24-hour
cycle

Functions of Sleep
Restoration, reducing fatigue,stabilizing mood, improving blood flow
to the brain, increasing protein synthesis, maintaining the disease-
fighting mechanisms of the immune system,promoting cellular growth
and repair, improving the capacity for learning and memory storage

Factors affecting sleep
physical illness
drugs and substances
emotional stress
environment
lifestyle
exercise and fatigue
food and caloric intake
sound

transduction of pain
process that begins in the periphery when pain-producing stimulus
send an impulse across a peripheral nerve fiber

transmission of pain
movement of pain impulses from the periphery to the spinal cord &
then to the brain

perception of pain
protects the body from damage, and is stimulated by extremes of
pressure and temperature, as well as chemicals released from damaged
tissues(physical component)

pain modulation
hindering the transmission of pain by release of inhibitory
neurotransmitters (endorphins&enkephalins) that produce an analgesic
effect

NSAIDs inhibit the synthesis of
prostaglandins

A patient's age, gender,anxiety, culture, and __________ influence
the pain experience
previous experience&meaning of pain

Evaluation of pain therapy requires the consideration of the
________ character of pain, ____ to therapy, ___ to function, and
patient's perception of a therapy's effectivness
changing
response
ability

hypostatic pneumonia
inflammation of the lung from stasis or pooling of secretions from lack
of movement and exercise

reactive hyperemia
a bright red flush on the skin occurring after pressure is relieved

blanchable hyperemia
redness of the skin due to dilation of the superficial capillaries. When
pressure is applied to the skin, the area blanches, or turns a lighter
color

nonblanchable hyperemia
redness of the skin due to dilation of the superficial capillaries. The
redness persists when pressure is applied to the area, indicating tissue
damage

Unstageable
full thickness tissue loss in which the base of the ulcer is covered by
slough and/or eschar

partial thickness wound repair
- inflammatory response (24hrs.)
- epithelial proliferation/migration
- reestablishment of epidermal layers

full thickness wound repair
- inflammatory (up to 3 days)
- proliferative (3-24 days)
- remodeling (up to 1 yr.)

evisceration
wound separation with protrusion of organs

fistula
abnormal passageway between two organs or between an internal
organ and the body surface

dihiscence
rupture separion of one or more layers of a wound.

incision
surgical cut or wound produced by a sharp instrument

abrasion
Scraping or wearing away of the skin by friction; irritation

contusion
an injury to underlying tissues without breaking the skin and is
characterized by discoloration and pain

open wound
an injury in which the skin is interrupted, exposing the tissue beneath

closed wound
wound that involves underlying tissue without break in the skin

puncture wound
An open wound that tears through the skin and destroys underlaying
tissues. A penetrating puncture wound can be shallow or deep. A
perforating puncture wound has both an entrance and an exit wound.

hematoma
the collection of blood under the skin as the result of blood escaping
into the tissue from damaged blood vessels. bruise

crushing injury
an injury caused by compression that involves both direct tissue injury
caused by circulation disturbance resulting from pressure on blood
vessels

primary intention
wounds that heal under conditions of minimal tissue loss(partial
thickness)

secondary intention
complex healing of a larger wound involving sealing of the wound
through scab formation, granulation or filling of the wound, and
constriction of the wound. (full thickness)

Which phrase best describes the science of nursing?
a. The skilled application of knowledge
b. The knowledge base for care
c. Hands-on care, such as giving a bath
d. Respect for each individual patient
b. the science of nursing is the knowledge base for care that is
provided. In contrast, the skilled application of that knowledge is the
art of nursing.

Which nurse in history is credited with establishing nursing
education?
a. clara barton
b. lilian wald
c. lavinia dock
d. florence nightingale
d. Florence Nightingale established nursing education

What historic event in the 20th century led to an increased emphasis
on nursing and broadened the role of nurses?
a. religious reform
b. crimean war
c. world war II
d. Vietnam War
c. World War II

Which of the following phrases describes one of the purposes of the
ANA's nursing's social policy statement?
a. to describe the nurse as a dependent caregiver
b. To provide standards for nursing educational programs
c. to regulate nursing research
d. to describe nursing's values and social responsiblity
d. The nursing's social policy statement describes the values and social
responsibility of nursing

A school nurse is teaching a class of junior-high students about the
effects of smoking. This educational program will meet which of the
aims of nursing?
a. promoting health
b. preventing illness
c. restoring health
d. facilitating coping with disability or death
b. Educational programs can reduce the risk of illness by teaching good
health habits

Which of the following nursing degrees prepares a nurse for
advanced practice as a clinical specialist or nurse practitioner?
a. LPN
b. ADN
c. BSN
d. Master's
d. A Master's degree prepares advanced practice nurses.

Which nursing organization was the first international organization
of professional women?
a. ICN
b. ANA
c. NLN
d. NSNA
a. The ICN, founded in 1899, was the first international organization of
professional women.

What is the purpose of the ANA's Scope and Standards of Practice?
a. To describe the ethical responsibility of nurses
b. To define the activities that are special and unique to nursing
c. To establish nursing as an independent and free standing
profession
d. To regulate the practice of nursing
b. The ANA's Scope and Standards of Practice define the activities of
nurses that are specific and unique to nursing.

What type of authority regulates the practice of nursing?
a. International standards and codes
b. Federal guidelines and regulations
c. State nurse practice acts
d. Institutional policies
c. Nurse practice act are established in each state to regulate the
practice of nursing

Who are the largest group of healthcare providers in the United
States?
a. Registered Nurses
b. Physicians
c. Physical therapists
d. Social Workers
a. Registered nurses are the largest group of healthcare providers in the
United States

Which of the following phrases best defines culture?
a. A dominant group within a society
b. A shared system of beliefs, values, and behaviors
c. One's values are replaced by the values of the dominant culture
d. Categories are based on specific physical characteristics
b. Culture may be defined as a shared system of beliefs, values, and
behavioral expectations that provide social structure for daily living

Minority groups living within a dominant culture may lose the
cultural characteristics that made them different. What is this
process called?
a. cultural diversity
b. cultural imposition
c. cultural assimilation
d. ethnocentrism
c. When minority groups live within a dominant group, many members
lose the cultural characteristics that once made them different

Which of the following terms is defined as the sense of
identification with a collective cultural group?
a. ethnicity
b. race
c. cultural acquisition
d. culture shock
a. Ethnicity is the sense of identification with a collective cultural
group, largely based on the group's common heritage.

A nurse states, that woman is 78 years old-too old to learn how to
change a dressing. What is the nurse demonstrating?
a. cultural imposition
b. clustering
c. cultural competency
d. stereotyping
d. stereotyping is assuming that all members of a group are alike.

A young hispanic mother comes to the local clinic because her baby
is sick. She speaks only Spanish and the nurse speaks only English.
What should the nurse do?
a. Use short words and talk more loudly?
b. Ask an interpreter for help
c. Explain why care can't be provided
d. Provide instructions in writing.
b. Many agencies have a qualified interpreter who understands the
healthcare system and can reliably provide assistance.

A nurse is interviewing a newly admitted patient. Which question
would be considered culturally sensitive?
a. do you think you will be able to eat the food we have here?
b. Do you understand that we can't prepare special meals?
c. What types of food do you eat for meals?
d. Why cant you just eat our food while you are here?
c. Asking patients what types of foods they eat for meals is culturally
sensitive

What group is the largest subculture of the healthcare system?
a. nurses
b. physicians
c. social workers
d. physical therapists
a. nurses are the largest subculture of the healthcare system

A nurse states, I know I am cleaner than most of my patients. What
does this statement indicate?
a. cultural assimilation
b. racism
c. ethnocentrism
d. sterotyping
c. Ethnocentrism occurs when one believes that one's own ideas and
practices are superior to those of others.

A nurse wants to acquire knowledge of a specific culture. What
could be done first?
a. talk to coworkers
b. review literature
c. talk to family members of the patient
d. ask others with more experience for help
b. reviewing literature about a specific culture can provide the nurse
with a starting point for information about cultural values, dietary
practices, family lines of authority, and helth and illness beliefs and
practices.

Although all of the following are important to culturally competent
nursing care, which one is the most basic?
a. learning another language
b. having signifigant information
c. treating each person as an individual
d. recognizing the importance of family
c. In all aspects of nursing, it is important to treat each patient as an
individual. This is also true in providing culturally competent care.

A nurse is providing care based on Maslow's hierarch of basic
human needs. For which of the following nursing activities is the
approach useful?
a. making accurate nursing diagnoses
b. establishing priorities of care
c. communicating concerns more concisely
d. integrating science into nursing care
b. Maslow's hierarch of basic human needs is useful for establishing
priorities of care.

Which of the following levels of basic human needs is most basic?
a. physiologic
b. safety and security
c. love and belonging
d. self-actualization
a. physiologic

Of all physiologic needs, which one is the most essential?
a. food
b. water
c. elimination
d. oxygen
d. oxygen

Practicing careful hand hygiene and using sterile techniques are
ways in which nurses meet which basic human need?
a. physiologic
b. safety and security
c. self esteem
d. love and belonging
b. safety and security

Of the following statements, which one is true of self actualization?
a. Humans are born with fully developed self-actualization
b. self-actualization needs are met by having confidence and
indepence
c. The self-actualization process continues throughout life.
d. loneliness and isolation occur when self-actualization needs are
unmet
c. self actualization, or reaching one's full potential is a process that
continues through life

What is the best broad definition of family?
a. a father, a mother, and children
b. a gruop whose members are biologically related
c. a unit that includes aunts, uncles, and cousins
d. a group of people who live together
d. although all of the responses may be true, the best definition is a
group of people who live together

Where do individuals learn their health beliefs and values?
a. in the family
b. in school
c. from school nurses
d. from peers
a. healthcare activities, health beliefs, and values are learned within
one's family.
See More
Please allow access to your computer’s microphone to use Voice
Recording.
Having trouble? Click here for help.

The nurse is having difficulty reading a physician's order for a


medication. The nurse knows the physician is very busy and does
not like to be called. The nurse should
Call the physician to have the order clarified

The client has an order for 2 tablespoons of Milk of Magnesia. The
nurse converts this dose to the metric system and gives the client:
30 mL

Most medication errors occur when the nurse:
Fails to follow routine procedures

A client is to receive cephalexin (Keflex) 500 mg PO. The pharmacy
has sent 250-mg tablets. The nurse gives:
2 tablets

When identifying a new client before administering medications, the
nurse asks the client to state his name. The client does not state the
correct name. The nurse asks again, and the client states still another
name. What is the nurse's next action?
Investigate the client's mental status before administering any further
medications.

A client is transitioning from the hospital to the home environment.
A home care referral is obtained. What is a priority, in relation to
safe medication administration, for the discharge nurse?
Ensure the home care agency is aware of medication and health
teaching needs.

A nursing student takes a client's antibiotic to his room. The client
asks the nursing student what it is and why he should take it. The
nursing student's reply includes the following information:
The name of the medication and a description of its desired effect

The nurse is administering a sustained-release capsule to a new
client. The client insists that he cannot swallow pills. The best
course of action for the nurse is to:
Ask the physician to change the order

The nurse takes a medication to a client, and the client tells the nurse
to take it away because she is not going to take it. The nurse's first
action should be to:
Ask the client's reason for refusal

The nurse selects the route for administering medication according
to:
The prescriber's orders

A client is receiving an IV push medication. If this type of drug
infiltrates into the outer tissues, the nurse will:
Follow facility policy or drug manufacturer's directions

If a client who is receiving IV fluids develops tenderness, warmth,
erythema, and pain at the site, the nurse suspects:
Phlebitis

A nurse administering medications has many responsibilities.
Among these responsibilities is a knowledge of pharmacokinetics.
Which statement is the best description of pharmacokinetics?
The study of how medications enter the body, reach their site of action,
metabolize, and exit the bod

The following orders were written by a prescriber (physician,
advanced practice nurse, physician's assistant). Which order is
written correctly?
CORRECT ANSWER IS C
A) Aspirin 2 tablets prn
B) Haloperidol (Haldol) ½ tablet at bedtime
C) Zolpidem (Ambien) 5 mg PO at bedtime prn
D) Levothyroxine (Synthroid) 0.05 mg 1 tablet

To better control the client's blood glucose level, the physician
orders a high regular insulin dosage of 20 units of U-500 insulin.
The nurse has only a U-100 syringe. How many units will be given
4
U-500 insulin is 5 times as strong as U-100 insulin. Therefore the
amount of U-500 insulin should be divided by 5; 20 units ÷ 5 = 4 units.

The nurse is administering an intramuscular (IM) injection. The Z-
track method is recommended for IM injections because:
It minimizes local skin irritation by sealing the medication in muscle
tissue.

What is the best nursing practice for administrating a controlled
substance if part of the medication must be discarded?
The nurse documents on the medication administration record and the
control inventory form, and has a second nurse witness the medication
being discarded.

When administering medications, it is essential for the nurse to have
an understanding of basic arithmetic to calculate doses. The
physician has ordered 250 mg of a medication that is available in 1-
g amount. The vial reads 2 ml = 1 g. What dose would be given by
the nurse?
0.5 ml = 250 mg of this medication.
(Dose ordered/dose on hand) × amount on hand = amount administered
[250 mg/1000 mg (1 g)] × 2 ml = 500/1000 = ½ ml or, in decimals, 0.5
ml

While the nurse is administering medication, the client says, "This
pill looks different from what I usually take." What is the nurse's
best action?
Go recheck the medication order, taking along the medication.

The client is a 40-year-old man who weighs 160 lb and is 5 feet 9
inches tall. The order is for 5 ml of a medication to be given as a
deep intramuscular (IM) injection. What size of syringe and gauge
and length of needle should the nurse use for best practice?
Two 3-ml syringes, 20- to 23-gauge, 1½-inch needle

A site that was a traditional location for intramuscular (IM)
injections in the past is no longer recommended because its use
carries the risk of striking the underlying sciatic nerve or major
blood vessel. What is the name of this site?
Dorsogluteal
Please allow access to your computer’s microphone to use Voice
Recording.
Having trouble? Click here for help.

1: Mr. Cooper is in the critical care unit and requires


sedation for an emergency procedure. The prescription
reads midazolam (Versed) 10 mcg/kg intravenous now.
The patient weighs 220 pounds. How many micrograms
of Versed do you give Mr. Cooper?
A: 22 mcg.
B: 1,000 mcg.
C: 100 mcg.
D: 2,200 mcg.
2: You are preparing to give Mr. Hamptou his Lopressor
(metoprolol tartrate) 100 mg PO daily as prescribed. The
pharmacy sends up Lipitor 50-mg tablets. How many
tablets do you give to Mr. Hamptou?
A: 2 tablets.
B: � tablet.
C: None; the pharmacy sent up the wrong
medication.
D: None; the prescription is for capsules and the
pharmacy sent up tablets.

3: Prescription: Synthroid, generic name levothyroxine,


100 mcg PO daily. 50 mcg = 0.05 mg. The pharmacy
sends up a 0.1-mg tablet of Synthroid. Is this the correct
dose?
A: No, it should be 0.5 mg.
B: Yes, this is the correct dose.
C: Yes, but you should give one-half of the tablet
daily.
D: No, you cannot convert micrograms to milligrams.

4: Prescription: Lamivudine 30 mg by mouth. Have:


Lamivudine oral solution = 10 mg/mL. How much will
you give?
A: 300 mL.
B: 30 mL.
C: 3.33 mL.
D: 3 mL.

5: Prescription: Indomethacin 20 mg by mouth every


morning. Have: Indomethacin oral suspension = 25 mg/5
mL. How much Indomethacin will you give each
morning?
A: 3 mL.
B: 4 mL.
C: 5 mL.
D: 100 mL.

6: Prescription: Valproic acid 400 mg by mouth now.


Have: Valproic acid oral syrup = 250mg/5 mL. How much
valproic acid will you give?
A: 8 mL.
B: 3.125 mL.
C: 20 mL.
D: 5 mL.

7: Prescription: Albuterol 1.6 mg by mouth. Have:


Albuterol oral syrup = 2 mg/5 mL. How much will you
give?
A: 6 mL.
B: 5 mL.
C: 4 mL.
D: 3 mL.

8: Prescription: Amprenivir 450 mg by mouth. Have:


Amprenivir 150-mg capsules. How many capsules of
Amprenivir will you give?
A: 2 capsules.
B: 3 capsules.
C: 4 capsules.
D: 5 capsules.

9: Nine-year-old Peter is diagnosed with otitis media and


receives the following prescription: Cefprozil oral
suspension 350 mg by mouth. Have: Cefprozil oral
suspension = 250 mg/5 mL. How much Cefprozil will you
give?
A: 7 mL.
B: 5 mL.
C: 3.57 mL.
D: 120 mL.
10: Mrs. Jennings has herpes zoster due to
immunosuppression as a result of chemotherapy given to
treat her breast cancer. She receives the following
prescription: Acyclovir 200 mg IV. Have: Acyclovir
solution for IV infusion = 500 mg/10 mL. How much will
you give?
A: 25 mL.
B: 4 mL.
C: 6 mL.
D: 70 mL.

11: Mr. Markwood is diagnosed with severe


adrenocortical insufficiency and is prescribed
hydrocortisone sodium succinate (Solu-Cortef) 250 mg IV
now. The pharmacy sends up multiple vials of Solu-Cortef
intravenous solution 125 mg/2 mL. How much Solu-
Cortef will you give?
A: 75 mL.
B: 187.5 mL.
C: 4 mL.
D: 2 mL.

12: Erica is seven years old and needs to be premedicated


to prevent nausea from the chemotherapy that she is set to
receive in 1 hour. The standing order reads as follows:
Metaclopramide syrup 8 mg by mouth 1 hour prior to
chemotherapy. The pharmacy sends up multiple
containers of metaclopramide syrup 5 mg/5 mL. How
much metaclopramide will you give 1 hour prior to
chemotherapy?
A: 40 mL.
B: 8 mL.
C: 3.125 mL.
D: 5 mL.

13: Mr. Walberg has a history of seizures, and his


laboratory work indicates that he has a therapeutic blood
level of phenytoin. Therefore, the nurse practitioner
prescribes the usual dose of phenytoin Mr. Walberg takes
at home: phenytoin 75 mg orally by suspension every
evening. The pharmacy sends up phenytoin 125 mg/5 mL.
How much phenytoin will you give Mr. Walberg this
evening?
A: 7 mL.
B: 8.33 mL.
C: 2.5 mL.
D: 3 mL.

14: Mrs. Corliss was in Mexico for vacation 3 months


ago and has stomach complaints. She has just been
diagnosed with intestinal amebiasis. The physician
prescribes the following: erythromycin (Estolate
suspension) 400 mg by mouth now. The pharmacy sends
up Estolate suspension 250 mg/5 mL. How much Estolate
suspension do you give now?
A: 8 mL.
B: 3.125 mL.
C: 1.6 mL.
D: 200 mL.

15: Mr. Johanson just arrived on the unit and is


prescribed loracarbef (Lorabid) 400 mg orally now. The
pharmacy dispenses Lorabid 200 mg/5 mL suspension.
How much Lorabid do you give Mr. Johanson?
A: 2.5 mL.
B: 5 mL.
C: 10 mL.
D: 20 mL.

16: Mrs. Bailey says she cannot sleep without her


sleeping pill. The nurse practitioner prescribes what Mrs.
Bailey has been taking at home, which is lorazepam
(Ativan) 1 mg orally every evening at bedtime. The
pharmacy sends up scored 2-mg tablets of lorazepam.
How much lorazepam do you give Mrs. Bailey tonight?
A: 0.5 tablet.
B: 1 tablet.
C: 2 tablets.
D: 4 tablets.

17: Mr. Doe comes into your clinic with an extreme case
of poison ivy. He says the last time he had this problem,
you gave him a �shot� that worked miracles. His
medical records show that he received dexamethasone
(Decadron) 10 mg IM, so you prescribe the same
medication and dose. The pharmacy dispenses
dexamethasone 4 mg/mL per vial. How many milliliters
of Decadron do you draw up?
A: 0.4 mL.
B: 1.5 mL.
C: 2 mL.
D: 2.5 mL.

18: Ms. Turner in prescribed fluoxetine hydrochloride


(Prozac) oral solution 10 mg orally daily in the morning
by her psychiatric Clinical Nurse Specialist. The
pharmacy sends up Prozac liquid 20 mg/5 mL. How much
Prozac do you draw up to give Ms. Turner?
A: 4 mL.
B: 2 mL.
C: 2.5 mL.
D: 10 mL.

19: The physician prescribes phenobarbital elixir 60 mg


PO now for the patient. The pharmacy dispenses a 120-
mL bottle of phenobarbital elixir labeled 20 mg/5 mL.
How much of the elixir should you administer to this
patient?
A: 5 mL.
B: 15 mL.
C: 30 mL.
D: 1.2 mL.
20: Randy has an infection in his foot from a cut he
received at the beach. His nurse practitioner prescribes
cephalothin (Keflin) 0.5 g orally every 8 hours � 10 days.
The pharmacist has dispensed Keflin in 250-mg capsules.
How many capsules should you educate Randy to take
every 8 hours?
A: 1 capsule.
B: 2 capsules.
C: 4 capsules.
D: 5 capsules.

21: Mr. Klingon is prescribed Demerol 150 mg


intramuscularly as a preoperative medication. The
pharmacy dispenses meperidine hydrochloride (Demerol)
100 mg/mL for IM use. How much Demerol do you draw
up to administer?
A: 1.5 mL.
B: 2 mL.
C: 2.5 mL.
D: 0.66 mL.

22: Your patient in the intensive care unit has an


infection that the culture and sensitivity indicate is
sensitive to fluconazole (Diflucan). The physician
prescribes 4 mg of fluconazole intravenously now. The
pharmacy sends up multiple vials of fluconazole 2 mg/mL
for IV infusion. How much Diflucan do you draw up in
preparation for mixing the medication with the proper
intravenous fluids per policy?
A: 0.5 mL.
B: 2 mL.
C: 3 mL.
D: 4 mL.

23: Ten-year-old Seraphima ate some peanuts about 5


minutes ago; she is allergic to nuts. She comes to the
Emergency Department with hives. The nurse practitioner
prescribes diphenhydramine hydrochloride (Benadryl) 25
mg intravenously now. The pharmacy dispenses a vial of
Benadryl at 50 mg/mL for IM or IV use. How much
Benadryl do you draw up to administer to Seraphima?
A: 0.5 mL.
B: 1 mL.
C: 0.75 mL.
D: 2 mL.

24: Daniel is a 9-year-old patient who broken his ankle


while playing football. He is prescribed atropine sulfate
0.2 mg IV as a preoperative medication. The pharmacy
dispenses a vial of atropine sulfate 0.4 mg/mL. How much
atropine do you draw up to give Daniel preoperatively?
A: 0.25 mL.
B: 0.5 mL.
C: 2 mL.
D: 0.8 mL.

25: Mr. Cooper is prescribed midazolam HCl (Versed) 3


mg intramuscularly 30 minutes prior to colonoscopy. The
pharmacy dispenses a multidose vial of Versed 1 mg/mL.
How many milliliters do you draw up to give Mr. Cooper?
A: 1 mL.
B: 2 mL.
C: 3 mL.
D: 4 mL.

26: Mrs. Skiba has developed heart failure and is


prescribed Lanoxin (digoxin) 250 mcg IV push now. The
pharmacy dispenses digoxin 500 mcg/2 mL. How much
digoxin do you administer?
A: 0.25 mL.
B: 0.5 mL.
C: 1 mL.
D: 4 mL.

27: Ms. Calugar has severe nausea related to a stomach


virus. The physician in the Emergency Department
prescribes cimetidine (Tagamet) 300 mg IM now. The
pharmacy sends up vials of Tagamet 300 mg/2 mL for IM
use. How much Tagamet do you draw up to give Ms.
Calugar?
A: 0.75 mL.
B: 0.5 mL.
C: 1 mL.
D: 2 mL.

28: Mr. Holden is prescribed Fragmin (dalteparin


sodium) 7,500 units SC daily for 5 days. The pharmacy
dispenses Fragmin 5,000 units/0.2 mL. How much
Fragmin do you draw up per day?
A: 0.2 mL.
B: 0.3 mL.
C: 0.75mL.
D: 1.5 mL.

29: Mrs. Fowler is prescribed Kenalog (triamcinolone) 60


mg IM � 1. The pharmacy dispenses Kenalog 40 mg/mL
for intramuscular use. How much Kenalog do you draw
up to administer?
A: 1.5 mL.
B: 0.75 mL.
C: 2 mL.
D: 3.5 mL.

30: Mr. Havens comes to the Emergency Department


with intractable hiccups lasting for 2 days. The physician
prescribes chlorpromazine (Thorazine) 50 mg IM now.
The pharmacy dispenses vials of Thorazine 25 mg/mL for
IM use. How much medication do you draw up?
A: 1 mL.
B: 2 mL.
C: 3 mL.
D: 4 mL.
31: Your patient needs anticoagulation therapy, and the
physician prescribes heparin sodium injection of 12,500
units SC now. The pharmacy sends up vials of heparin
sodium 5,000 units/mL for subcutaneous injection. How
much heparin sodium do you draw up?
A: 0.4 mL.
B: 2 mL.
C: 2.5 mL.
D: 5 mL.

32: Your patient is 10 days post-chemotherapy and is


prescribed epoetin alfa (Procrit) 12,000 units SC three
times per week. The pharmacy dispenses a vial of Procrit
20,000 units/mL. How much epoetin alfa do you draw up
for each dose?
A: 1.66 mL.
B: 0.6 mL.
C: 2 mL.
D: 0.8 mL.

33: To keep Ms. Grimmitt's cardiac output within normal


range, she must receive Lanoxin (digoxin) 750 mcg
intravenously daily. The medication strength received
from pharmacy is Lanoxin 500 mcg/2mL. How much
Lanoxin do you draw up daily to give Ms. Grimmitt?
A: 5 mL.
B: 0.33 mL.
C: 1.33 mL.
D: 3 mL.

You might also like